Степенные выражения примеры: Степенные выражения (выражения со степенями) и их преобразование

Содержание

Степенные выражения (выражения со степенями) и их преобразование

Рассмотрим тему преобразования выражений со степенями, но прежде остановимся на ряде преобразований, которые можно проводить с любыми выражениями, в том числе со степенными. Мы научимся раскрывать скобки, приводить подобные слагаемые, работать с основанием и показателем степени, использовать свойства степеней.

Что представляют собой степенные выражения?

В школьном курсе мало кто использует словосочетание «степенные выражения», зато этот термин постоянно встречается в сборниках для подготовки к ЕГЭ. В большинства случаев словосочетанием обозначаются выражения, которые содержат в своих записях степени. Это мы и отразим в нашем определении.

Определение 1

Степенное выражение – это выражение, которое содержит степени.

Приведем несколько примеров степенных выражений, начиная со степени с натуральным показателем и заканчивая степенью с действительным показателем.

Самыми простыми степенными выражениями можно считать степени числа с натуральным показателем: 32, 75+1, (2+1)5, (−0,1)4, 2233, 3·a2−a+a2, x3−1, (a2)3. А также степени с нулевым показателем: 50, (a+1)0, 3+52−3,20. И степени с целыми отрицательными степенями: (0,5)2+(0,5)-22.

Чуть сложнее работать со степенью, имеющей рациональный  и иррациональный показатели: 26414-3·3·312, 23,5·2-22-1,5, 1a14·a12-2·a-16·b12, xπ·x1-π, 233+5.

В качестве показателя может выступать переменная 3x-54-7·3x-58 или логарифм x2·lgx−5·xlgx.

С вопросом о том, что такое степенные выражения, мы разобрались. Теперь займемся их преобразованием.

Основные виды преобразований степенных выражений

В первую очередь мы рассмотрим основные тождественные преобразования выражений, которые можно выполнять со степенными выражениями.

Пример 1

Вычислите значение степенного выражения 23·(42−12).

Решение

Все преобразования мы будем проводить с соблюдением порядка выполнения действий. В данном случае начнем мы с выполнения действий в скобках: заменим степень на цифровое значение и вычислим разность двух чисел. Имеем 23·(42−12)=23·(16−12)=23·4.

Нам остается заменить степень 23 ее значением 8 и вычислить произведение 8·4=32. Вот наш ответ.

Ответ: 23·(42−12)=32.

Пример 2

Упростите выражение со степенями 3·a4·b−7−1+2·a4·b−7.

Решение

Данное нам в условии задачи выражение содержит подобные слагаемые, которые мы можем привести: 3·a4·b−7−1+2·a4·b−7=5·a4·b−7−1.

Ответ: 3·a4·b−7−1+2·a4·b−7=5·a4·b−7−1.

Пример 3

Представьте выражение со степенями 9-b3·π-12 в виде произведения.

Решение

Представим число 9 как степень 32 и применим формулу сокращенного умножения:

9-b3·π-12=32-b3·π-12==3-b3·π-13+b3·π-1

Ответ: 9-b3·π-12=3-b3·π-13+b3·π-1.

А теперь перейдем к разбору тождественных преобразований, которые могут применяться именно в отношении степенных выражений.  

Работа с основанием и показателем степени

Степень в основании или показателе может иметь и числа, и переменные, и некоторые выражения. Например, (2+0,3·7)5−3,7 и (a·(a+1)−a2)2·(x+1). Работать с такими записями сложно. Намного проще заменить выражение в основании степени или выражение в показателе тождественно равным выражением.

Проводятся преобразования степени и показателя по известным нам правилам отдельно друг от друга. Самое главное, чтобы в результате преобразований получилось выражение, тождественное исходному.

Цель преобразований – упростить исходное выражение или получить решение задачи. Например, в примере, который мы привели выше, (2+0,3·7)5−3,7 можно выполнить действия для перехода к степени 4,11,3. Раскрыв скобки, мы можем привести подобные слагаемые в основании степени (a·(a+1)−a2)2·(x+1) и получить степенное выражение более простого вида a2·(x+1).

Использование свойств степеней

Свойства степеней, записанные в виде равенств, являются одним из главных инструментов преобразования выражений со степенями. Приведем здесь основные из них, учитывая, что a и b – это любые положительные числа, а r и s — произвольные действительные числа:

Определение 2

  • ar·as=ar+s;
  • ar:as=ar−s;
  • (a·b)r=ar·br;
  • (a:b)r=ar:br;
  • (ar)s=ar·s.

В тех случаях, когда мы имеем дело с натуральными, целыми, положительными показателями степени, ограничения на числа a и b могут быть гораздо менее строгими. Так, например, если рассмотреть равенство am·an=am+n, где m и n – натуральные числа, то оно будет верно для любых значений a, как положительных, так и отрицательных, а также для a=0.

Применять свойства степеней без ограничений можно в тех случаях, когда основания степеней положительные или содержат переменные, область допустимых значений которых такова, что на ней основания принимают лишь положительные значения. Фактически, в рамках школьной программы по математике задачей учащегося является выбор подходящего свойства и правильное его применение.

При подготовке к поступлению в Вузы могут встречаться задачи, в которых неаккуратное применение свойств будет приводить к сужению ОДЗ и другим сложностям с решением. В данном разделе мы разберем всего два таких случая. Больше информации по вопросу можно найти в теме «Преобразование выражений с использованием свойств степеней».

Пример 4

Представьте выражение a2,5·(a2)−3:a−5,5 в виде степени с основанием a.

Решение

Для начала используем свойство возведения в степень и преобразуем по нему второй множитель (a2)−3 . Затем используем свойства умножения и деления степеней с одинаковым основанием:

a2,5·a−6:a−5,5= a2,5−6:a−5,5=a−3,5:a−5,5= a−3,5−(−5,5)=a2.

Ответ: a2,5·(a2)−3:a−5,5=a2.

Преобразование степенных выражений согласно свойству степеней может производиться как слева направо, так и в обратном направлении.

Пример 5

Найти значение степенного выражения 313·713·2123.

Решение

Если мы применим равенство (a·b)r=ar·br, справа налево, то получим произведение вида 3·713·2123 и дальше 2113·2123. Сложим показатели при умножении степеней с одинаковыми основаниями: 2113·2123=2113+23=211=21.

Есть еще один способ провести преобразования:

313·713·2123=313·713·(3·7)23=313·713·323·723==313·323·713·723=313+23·713+23=31·71=21

Ответ: 313·713·2123=31·71=21

Нужна помощь преподавателя?

Опиши задание — и наши эксперты тебе помогут!

Описать задание

 
Пример 6

Дано степенное выражение a1,5−a0,5−6, введите новую переменную t=a0,5.

Решение

Представим степень a1,5 как a0,5·3 . Используем свойство степени в степени (ar)s=ar·s справа налево и получим (a0,5)3: a1,5−a0,5−6=(a0,5)3−a0,5−6. В полученное выражение можно без проблем вводить новую переменную t=a0,5: получаем t3−t−6.

Ответ: t3−t−6.

Преобразование дробей, содержащих степени

Обычно мы имеем дело с двумя вариантами степенных выражений с дробями: выражение представляет собой дробь со степенью или содержит такую дробь. К таким выражениям применимы все основные преобразования дробей без ограничений. Их можно сокращать, приводить к новому знаменателю, работать отдельно с числителем и знаменателем. Проиллюстрируем это примерами.

Пример 7

Упростить степенное выражение 3·523·513-5-231+2·x2-3-3·x2.

Решение

Мы имеем дело с дробью, поэтому проведем преобразования и в числителе, и в знаменателе:

3·523·513-5-231+2·x2-3-3·x2=3·523·513-3·523·5-23-2-x2==3·523+13-3·523+-23-2-x2=3·51-3·50-2-x2

Поместим минус перед дробью для того, чтобы изменить знак знаменателя: 12-2-x2=-122+x2

Ответ:  3·523·513-5-231+2·x2-3-3·x2=-122+x2

Дроби, содержащие степени, приводятся к новому знаменателю точно также, как и рациональные дроби. Для этого необходимо найти дополнительный множитель и умножить на него числитель и знаменатель дроби. Подбирать дополнительный множитель необходимо таким образом, чтобы он не обращался в нуль ни при каких значениях переменных из ОДЗ переменных для исходного выражения.

Пример 8

 

Приведите дроби к новому знаменателю: а) a+1a0,7 к знаменателю a, б) 1×23-2·x13·y16+4·y13 к знаменателю x+8·y12.

Решение

а) Подберем множитель, который позволит нам произвести приведение к новому знаменателю. a0,7·a0,3=a0,7+0,3=a, следовательно, в качестве дополнительного множителя мы возьмем a0,3. Область допустимых значений переменной а включает множество всех положительных действительных чисел. В этой области степень a0,3 не обращается в нуль.

Выполним умножение числителя и знаменателя дроби на a0,3:

a+1a0,7=a+1·a0,3a0,7·a0,3=a+1·a0,3a

б) Обратим внимание на знаменатель:

x23-2·x13·y16+4·y13==x132-x13·2·y16+2·y162

Умножим это выражение на x13+2·y16, получим сумму кубов x13 и 2·y16, т.е. x+8·y12. Это наш новый знаменатель, к которому нам надо привести исходную дробь.

 

Так мы нашли дополнительный множитель x13+2·y16. На области допустимых значений переменных x и y выражение x13+2·y16 не обращается в нуль, поэтому, мы можем умножить на него числитель и знаменатель дроби:
1×23-2·x13·y16+4·y13==x13+2·y16x13+2·y16x23-2·x13·y16+4·y13==x13+2·y16x133+2·y163=x13+2·y16x+8·y12

Ответ: а) a+1a0,7=a+1·a0,3a , б) 1×23-2·x13·y16+4·y13=x13+2·y16x+8·y12.  

Пример 9

Сократите дробь: а) 30·x3·(x0,5+1)·x+2·x113-5345·x0,5+12·x+2·x113-53, б) a14-b14a12-b12.

Решение

а) Используем наибольший общий знаменатель (НОД), на который можно сократить числитель и знаменатель. Для чисел 30 и 45 это 15. Также мы можем произвести сокращение на x0,5+1 и на x+2·x113-53.

Получаем:

30·x3·(x0,5+1)·x+2·x113-5345·x0,5+12·x+2·x113-53=2·x33·(x0,5+1)

б) Здесь наличие одинаковых множителей неочевидно. Придется выполнить некоторые преобразования для того, чтобы получить одинаковые множители в числителе и знаменателе. Для этого разложим знаменатель, используя формулу разности квадратов:

a14-b14a12-b12=a14-b14a142-b122==a14-b14a14+b14·a14-b14=1a14+b14

Ответ:  а)30·x3·(x0,5+1)·x+2·x113-5345·x0,5+12·x+2·x113-53=2·x33·(x0,5+1), б) a14-b14a12-b12=1a14+b14.

К числу основных действий с дробями относится приведение к новому знаменателю и сокращение дробей. Оба действия выполняют с соблюдением ряда правил. При сложении и вычитании дробей сначала дроби приводятся к общему знаменателю, после чего проводятся действия (сложение или вычитание) с числителями. Знаменатель остается прежним. Результатом наших действий является новая дробь, числитель которой является произведением числителей, а знаменатель есть произведение знаменателей.

Пример 10

Выполните действия x12+1×12-1-x12-1×12+1·1×12.

Решение

Начнем с вычитания дробей, которые располагаются в скобках. Приведем их к общему знаменателю:

x12-1·x12+1

Вычтем числители:

x12+1×12-1-x12-1×12+1·1×12==x12+1·x12+1×12-1·x12+1-x12-1·x12-1×12+1·x12-1·1×12==x12+12-x12-12×12-1·x12+1·1×12==x122+2·x12+1-x122-2·x12+1×12-1·x12+1·1×12==4·x12x12-1·x12+1·1×12

Теперь умножаем дроби:

4·x12x12-1·x12+1·1×12==4·x12x12-1·x12+1·x12

Произведем сокращение на степень x12, получим 4×12-1·x12+1.

Дополнительно можно упростить степенное выражение в знаменателе, используя формулу разности квадратов: квадратов: 4×12-1·x12+1=4×122-12=4x-1.

Ответ: x12+1×12-1-x12-1×12+1·1×12=4x-1

Пример 11

Упростите степенное выражение x34·x2,7+12x-58·x2,7+13.
Решение

Мы можем произвести сокращение дроби на (x2,7+1)2. Получаем дробь x34x-58·x2,7+1.

Продолжим преобразования степеней икса x34x-58·1×2,7+1. Теперь можно использовать свойство деления степеней с одинаковыми основаниями:  x34x-58·1×2,7+1=x34—58·1×2,7+1=x118·1×2,7+1.

Переходим от последнего произведения к дроби x138x2,7+1.

Ответ: x34·x2,7+12x-58·x2,7+13=x138x2,7+1.

Множители с отрицательными показателями степени в большинстве случаев удобнее переносить из числителя в знаменатель и обратно, изменяя знак показателя. Это действие позволяет упростить дальнейшее решение. Приведем пример: степенное выражение (x+1)-0,23·x-1 можно заменить  на x3·(x+1)0,2.

Преобразование выражений с корнями и степенями

В задачах встречаются степенные выражения, которые содержат не только степени с дробными показателями, но и корни. Такие выражения желательно привести только к корням или только к степеням. Переход к степеням предпочтительнее, так как с ними проще работать. Такой переход является особенно предпочтительным, когда ОДЗ переменных для исходного выражения позволяет заменить корни степенями без необходимости обращаться к модулю или разбивать ОДЗ на несколько промежутков.

Пример 12

Представьте выражение x19·x·x36 в виде степени.

Решение

Область допустимых значений переменной x определяется двумя неравенствами  x≥0  и x·x3≥0 ,  которые задают множество [0, +∞).

На этом множестве мы имеем право перейти от корней к степеням: 

x19·x·x36=x19·x·x1316

Используя свойства степеней, упростим полученное степенное выражение.

x19·x·x1316=x19·x16·x1316=x19·x16·x1·13·6==x19·x16·x118=x19+16+118=x13

Ответ: x19·x·x36=x13.

Преобразование степеней с переменными в показателе

Данные преобразования достаточно просто произвести, если грамотно использовать свойства степени. Например, 52·x+1−3·5x·7x−14·72·x−1=0.

Мы можем заменить произведением степени, в показателях которых находится сумма некоторой переменной и числа. В левой части это можно проделать с первым и последним слагаемыми левой части выражения:

52·x·51−3·5x·7x−14·72·x·7−1=0, 5·52·x−3·5x·7x−2·72·x=0.

Теперь поделим обе части равенства на 72·x. Это выражение на ОДЗ переменной x принимает только положительные значения:

5·5-3·5x·7x-2·72·x72·x=072·x,5·52·x72·x-3·5x·7×72·x-2·72·x72·x=0,5·52·x72·x-3·5x·7x7x·7x-2·72·x72·x=0

Сократим дроби со степенями, получим: 5·52·x72·x-3·5x7x-2=0.

Наконец, отношение степеней с одинаковыми показателями заменяется степенями отношений, что приводит к уравнению 5·572·x-3·57x-2=0 , которое равносильно 5·57×2-3·57x-2=0.

Введем новую переменную t=57x, что сводит решение исходного показательного уравнения к решению квадратного уравнения 5·t2−3·t−2=0.

Преобразование выражений со степенями и логарифмами

Выражения, содержащие с записи степени и логарифмы, также встречаются в задачах. Примером таких выражений могут служить: 141-5·log23 или log3279+5(1-log35)·log53. Преобразование подобных выражений проводится с использованием разобранных выше подходов и свойств логарифмов, которые мы подробно разобрали в теме «Преобразование логарифмических выражений».

Решение примеров со степенями. Степенные выражения (выражения со степенями) и их преобразование

Разделы:
Математика

Тип урока:
урок обобщения и систематизации знаний

Цели:

  • обучающие
    – повторить определение степени, правила умножения и деления степеней, возведения степени в степень, закрепить умения решения примеров, содержащих степени,
  • развивающие
    – развитие логического мышления учащихся, интереса к изучаемому материалу,
  • воспитывающие
    – воспитание ответственного отношения к учебе, культуры общения, чувства коллективизма.
  • Оборудование:
    компьютер, мультимедийный проектор, интерактивная доска, презентация “Степени” для устного счета, карточки с заданиями, раздаточный материал.

    План урока:

  • Организационный момент.
  • Повторение правил
  • Устный счет.
  • Историческая справка.
  • Работа у доски.
  • Физкультминутка.
  • Работа на интерактивной доске.
  • Самостоятельная работа.
  • Домашнее задание.
  • Подведение итогов урока.
  • Ход урока

    I. Организационный момент

    Сообщение темы и целей урока.

    На предыдущих уроках вы открыли для себя удивительный мир степеней, научились умножать и делить степени, возводить их в степень. Сегодня мы должны закрепить полученные знания при решении примеров.

    II. Повторение правил
    (устно)

    1. Дайте определение степени с натуральным показателем? (Степенью числа а
      с натуральным показателем, большим 1, называется произведение n
      множителей, каждый из которых равен а
      .)
    2. Как умножить две степени? (Чтобы умножить степени с одинаковыми основаниями, надо основание оставить тем же, а показатели сложить. )
    3. Как разделить степень на степень? (Чтобы разделить степени с одинаковыми основаниями, надо основание оставить тем же, а показатели вычесть.)
    4. Как возвести произведение в степень? (Чтобы возвести произведение в степень, надо каждый множитель возвести в эту степень)
    5. Как возвести степень в степень? (Чтобы возвести степень в степень, надо основание оставить тем же, а показатели перемножить)
    6. III. Устный счет
      (по мультимедиа)

      IV. Историческая справка

      Все задачи из папируса Ахмеса, который записан около 1650 года до н. э. связаны с практикой строительства, размежеванием земельных наделов и т. п. Задачи сгруппированы по тематике. По преимуществу это задачи на нахождение площадей треугольника, четырёхугольников и круга, разнообразные действия с целыми числами и дробями, пропорциональное деление, нахождение отношений, здесь присутствует и возведение в разные степени, решение уравнений первой и второй степени с одним неизвестным.

      Полностью отсутствуют какие бы то ни было объяснения или доказательства. Искомый результат либо даётся прямо, либо приводится краткий алгоритм его вычисления. Такой способ изложения, типичный для науки стран древнего Востока, наводит на мысль о том, что математика там развивалась путём обобщений и догадок, не образующих никакой общей теории. Тем не менее, в папирусе есть целый ряд свидетельств того, что египетские математики умели извлекать корни и возводить в степень, решать уравнения, и даже владели зачатками алгебры.

      V. Работа у доски

      Найдите значение выражения рациональным способом:

      Вычислите значение выражения:

      VI. Физкультминутка

    7. для глаз
    8. для шеи
    9. для рук
    10. для туловища
    11. для ног
    12. VII. Решение задач
      (с показом на интерактивной доске)

      Является ли корень уравнения положительным числом?

      xn--i1abbnckbmcl9fb.xn--p1ai

      Формулы степеней и корней.

      Формулы степеней
      используют в процессе сокращения и упрощения сложных выражений, в решении уравнений и неравенств.

      Число c
      является n
      -ной степенью числа a
      когда:

      Операции со степенями.

      1. Умножая степени с одинаковым основанием их показатели складываются:

      2. В делении степеней с одинаковым основанием их показатели вычитаются:

      3. Степень произведения 2-х либо большего числа множителей равняется произведению степеней этих сомножителей:

      (abc…) n = a n · b n · c n …

      4. Степень дроби равняется отношению степеней делимого и делителя:

      5. Возводя степень в степень, показатели степеней перемножают:

      Каждая вышеприведенная формула верна в направлениях слева направо и наоборот.

      Операции с корнями.

      1. Корень из произведения нескольких сомножителей равняется произведению корней из этих сомножителей:

      2. Корень из отношения равен отношению делимого и делителя корней:

      3. При возведении корня в степень довольно возвести в эту степень подкоренное число:

      4. Если увеличить степень корня в n
      раз и в тоже время возвести в n
      -ую степень подкоренное число, то значение корня не поменяется:

      5. Если уменьшить степень корня в n
      раз и в тоже время извлечь корень n
      -ой степени из подкоренного числа, то значение корня не поменяется:

      Степень некоторого числа с неположительным (целым) показателем определяют как единицу, деленную на степень того же числа с показателем, равным абсолютной величине неположительного показателя:

      Формулу a m
      :a n =a m — n
      можно использовать не только при m
      > n
      , но и при m
      4:a 7 = a 4 — 7 = a -3 .

      Чтобы формула a m
      :a n =a m — n
      стала справедливой при m=n
      , нужно присутствие нулевой степени.

      Степень всякого числа, не равного нулю, с нулевым показателем равняется единице.

      Чтобы возвести действительное число а
      в степень m/n
      , необходимо извлечь корень n
      –ой степени из m
      -ой степени этого числа а
      :

      Формулы степеней.


      6.
      a

      n
      =
      — деление степеней;

      7.
      — деление степеней;

      8. a 1/n =
      ;

      Степени правила действия со степенями

      1. Степень произведения двух или нескольких сомножителей равна произведению степеней этих сомножителей (с тем же показателем):

      (abc…) n = a n b n c n …

      Пример 1. (7 2 10) 2 = 7 2 2 2 10 2 = 49 4 100 = 19600. Пример 2. (x 2 –a 2) 3 = [(x +a)(x — a)] 3 =(x +a) 3 (x — a) 3

      Практически более важно обратное преобразование:

      a n b n c n … = (abc…) n

      т.е. произведение одинаковых степеней нескольких величин равно той же степени произведения этих величин.

      Пример 3. Пример 4. (a +b) 2 (a 2 – ab +b 2) 2 =[(a +b)(a 2 – ab +b 2)] 2 =(a 3 +b 3) 2

      2. Степень частного (дроби) равна частному от деления той же степени делимого на ту же степень делителя:

      Пример 5. Пример 6.

      Обратное преобразование:. Пример 7.. Пример 8..

      3. При умножении степеней с одинаковыми основаниями показатели степеней складываются:

      Пример 9.2 2 2 5 =2 2+5 =2 7 =128. Пример 10. (a – 4c +x) 2 (a – 4c +x) 3 =(a – 4c + x) 5 .

      4. При делении степеней с одинаковыми основаниями показатель степени делителя вычитается из показателя степени делимого

      Пример 11. 12 5:12 3 =12 5-3 =12 2 =144. Пример 12. (x-y) 3:(x-y) 2 =x-y.

      5. При возведении степени в степень показатели степеней перемножаются:

      Пример 13. (2 3) 2 =2 6 =64. Пример 14.

      www.maths.yfa1.ru

      Степени и корни

      Операции со степенями и корнями. Степень с отрицательным
      ,

      нулевым и дробным
      показателем. О выражениях, не имеющих смысла.

      Операции со степенями.

      1. При умножении степеней с одинаковым основанием их показатели складываются:

      a m
      · a n = a m + n .

      2. При делении степеней с одинаковым основанием их показатели
      вычитаются
      .

      3. Степень произведения двух или нескольких сомножителей равна произведению степеней этих сомножителей.

      4. Степень отношения (дроби) равна отношению степеней делимого (числителя) и делителя (знаменателя):

      (a / b
      ) n = a n / b n .

      5. При возведении степени в степень их показатели перемножаются:

      Все вышеприведенные формулы читаются и выполняются в обоих направлениях слева направо и наоборот.

      П р и м е р. (2 · 3 · 5 / 15) ² =
      2 ² · 3 ² · 5 ² / 15 ² = 900 / 225 = 4 .

      Операции с корнями.

      Во всех нижеприведенных формулах символ означает арифметический корень
      (подкоренное выражение положительно).

      1. Корень из произведения нескольких сомножителей равен произведению корней из этих сомножителей:

      2. Корень из отношения равен отношению корней делимого и делителя:

      3. При возведении корня в степень достаточно возвести в эту степень
      подкоренное число:

      4. Если увеличить степень корня в m раз и одновременно возвести в m -ую степень подкоренное число, то значение корня не изменится:

      5. Если уменьшить степень корня в m раз и одновременно извлечь корень m -ой степени из подкоренного числа, то значение корня не изменится:


      Расширение понятия степени.

      До сих пор мы рассматривали степени только с натуральным показателем; но действия со степенями и корнями могут приводить также к отрицательным
      , нулевым
      и дробным
      показателям. Все эти показатели степеней требуют дополнительного определения.

      Степень с отрицательным показателем.

      Степень некоторого числа с отрицательным (целым) показателем определяется как единица, делённая на степень того же числа с показателем, равным абсолютной велечине отрицательного показателя:

      Т еперь формула a m
      : a n
      = a m — n
      может быть использована не только при m
      , большем, чем n
      , но и при m
      , меньшем, чем n
      .

      П р и м е р. a
      4: a
      7 = a
      4 — 7 = a
      — 3 .

      Если мы хотим, чтобы формула a m
      : a n
      = a m
      n
      была справедлива при m = n
      , нам необходимо определение нулевой степени.

      Степень с нулевым показателем.

      Степень любого ненулевого числа с нулевым показателем равна 1.

      П р и м е р ы. 2 0 = 1, (
      5) 0 = 1, (
      3 / 5) 0 = 1.

      Степень с дробным показателем.

      Для того, чтобы возвести действительное число а в степень m / n , нужно извлечь корень n –ой степени из m -ой степени этого числа а:

      О выражениях, не имеющих смысла.

      Есть несколько таких выражений.

      где a
      ≠ 0 , не существует.

      В самом деле, если предположить, что x
      – некоторое число, то в соответствии с определением операции деления имеем: a
      = 0· x
      , т.e. a
      = 0, что противоречит условию: a
      ≠ 0

      любое число.

      В самом деле, если предположить, что это выражение равно некоторому числу x
      , то согласно определению операции деления имеем: 0 = 0 · x
      . Но это равенство имеет место при любом числе x
      , что и требовалось доказать.

      0 0 — любое число.

      Р е ш е н и е. Рассмотрим три основных случая:

      1) x
      = 0
      это значение не удовлетворяет данному уравнению

      2) при x
      > 0 получаем: x / x
      = 1, т.e. 1 = 1, откуда следует,

      что x
      – любое число; но принимая во внимание, что в

      нашем случае x
      > 0 , ответом является x
      > 0 ;

      Свойства степени

      Напоминаем, что в данном уроке разбираются свойства степеней
      с натуральными показателями и нулём. Степени с рациональными показателями и их свойства будут рассмотрены в уроках для 8 классов.

      Степень с натуральным показателем обладает несколькими важными свойствами, которые позволяют упрощать вычисления в примерах со степенями.

      Свойство № 1

      Произведение степеней

      При умножении степеней с одинаковыми основаниями основание остаётся без изменений, а показатели степеней складываются.

      a m · a n = a m + n , где « a » — любое число, а « m », « n » — любые натуральные числа.

      Данное свойство степеней также действует на произведение трёх и более степеней.

    • Упростить выражение.
      b · b 2 · b 3 · b 4 · b 5 = b 1 + 2 + 3 + 4 + 5 = b 15
    • Представить в виде степени.
      6 15 · 36 = 6 15 · 6 2 = 6 15 · 6 2 = 6 17
    • Представить в виде степени.
      (0,8) 3 · (0,8) 12 = (0,8) 3 + 12 = (0,8) 15
    • Обратите внимание, что в указанном свойстве речь шла только об умножении степеней с одинаковыми основаниями
      . Оно не относится к их сложению.

      Нельзя заменять сумму (3 3 + 3 2) на 3 5 . Это понятно, если
      посчитать (3 3 + 3 2) = (27 + 9) = 36 , а 3 5 = 243

      Свойство № 2

      Частное степеней

      При делении степеней с одинаковыми основаниями основание остаётся без изменений, а из показателя степени делимого вычитают показатель степени делителя.

    • Записать частное в виде степени
      (2b) 5: (2b) 3 = (2b) 5 − 3 = (2b) 2
    • Вычислить.

    11 3 − 2 · 4 2 − 1 = 11 · 4 = 44
    Пример. Решить уравнение. Используем свойство частного степеней.
    3 8: t = 3 4

    Ответ: t = 3 4 = 81

    Пользуясь свойствами № 1 и № 2, можно легко упрощать выражения и производить вычисления.

    Пример. Упростить выражение.
    4 5m + 6 · 4 m + 2: 4 4m + 3 = 4 5m + 6 + m + 2: 4 4m + 3 = 4 6m + 8 − 4m − 3 = 4 2m + 5

    Пример. Найти значение выражения, используя свойства степени.

    2 11 − 5 = 2 6 = 64

    Обратите внимание, что в свойстве 2 речь шла только о делении степеней с одинаковыми основаниями.

    Нельзя заменять разность (4 3 −4 2) на 4 1 . Это понятно, если посчитать (4 3 −4 2) = (64 − 16) = 48 , а 4 1 = 4

    Свойство № 3

    Возведение степени в степень

    При возведении степени в степень основание степени остаётся без изменения, а показатели степеней перемножаются.

    (a n) m = a n · m , где « a » — любое число, а « m », « n » — любые натуральные числа.

  • Пример.
    (a 4) 6 = a 4 · 6 = a 24
  • Пример. Представить 3 20 в виде степени с основанием 3 2 .
  • По свойству возведения степени в степень
    известно, что при возведении в степень показатели перемножаются, значит:

    Свойства 4

    Степень произведения

    При возведении степени в степень произведения в эту степень возводится каждый множитель и результаты перемножаются.

    (a · b) n = a n · b n , где « a », « b » — любые рациональные числа; « n » — любое натуральное число.

    • Пример 1.
      (6 · a 2 · b 3 · c) 2 = 6 2 · a 2 · 2 · b 3 · 2 · с 1 · 2 = 36 a 4 · b 6 · с 2
    • Пример 2.
      (−x 2 · y) 6 = ((−1) 6 · x 2 · 6 · y 1 · 6) = x 12 · y 6
    • Обратите внимание, что свойство № 4, как и другие свойства степеней, применяют и в обратном порядке.

      (a n · b n)= (a · b) n

      То есть, чтобы перемножить степени с одинаковыми показателями можно перемножить основания, а показатель степени оставить неизменным.

    • Пример. Вычислить.
      2 4 · 5 4 = (2 · 5) 4 = 10 4 = 10 000
    • Пример. Вычислить.
      0,5 16 · 2 16 = (0,5 · 2) 16 = 1
    • В более сложных примерах могут встретиться случаи, когда умножение и деление надо выполнить над степенями с разными основаниями и разными показателями. В этом случае советуем поступать следующим образом.

      Например, 4 5 · 3 2 = 4 3 · 4 2 · 3 2 = 4 3 · (4 · 3) 2 = 64 · 12 2 = 64 · 144 = 9216

      Пример возведения в степень десятичной дроби.

      4 21 · (−0,25) 20 = 4 · 4 20 · (−0,25) 20 = 4 · (4 · (−0,25)) 20 = 4 · (−1) 20 = 4 · 1 = 4

      Свойства 5

      Степень частного (дроби)

      Чтобы возвести в степень частное, можно возвести в эту степень отдельно делимое и делитель, и первый результат разделить на второй.

      (a: b) n = a n: b n , где « a », « b » — любые рациональные числа, b ≠ 0, n — любое натуральное число.

    • Пример. Представить выражение в виде частного степеней.
      (5: 3) 12 = 5 12: 3 12
    • Напоминаем, что частное можно представить в виде дроби. Поэтому на теме возведение дроби в степень мы остановимся более подробно на следующей странице.

    Рассмотрим тему преобразования выражений со степенями, но прежде остановимся на ряде преобразований, которые можно проводить с любыми выражениями, в том числе со степенными. Мы научимся раскрывать скобки, приводить подобные слагаемые, работать с основанием и показателем степени, использовать свойства степеней.

    Yandex.RTB R-A-339285-1

    Что представляют собой степенные выражения?

    В школьном курсе мало кто использует словосочетание «степенные выражения», зато этот термин постоянно встречается в сборниках для подготовки к ЕГЭ. В большинства случаев словосочетанием обозначаются выражения, которые содержат в своих записях степени. Это мы и отразим в нашем определении.

    Определение 1

    Степенное выражение
    – это выражение, которое содержит степени.

    Приведем несколько примеров степенных выражений, начиная со степени с натуральным показателем и заканчивая степенью с действительным показателем.

    Самыми простыми степенными выражениями можно считать степени числа с натуральным показателем: 3 2 , 7 5 + 1 , (2 + 1) 5 , (− 0 , 1) 4 , 2 2 3 3 , 3 · a 2 − a + a 2 , x 3 − 1 , (a 2) 3 . А также степени с нулевым показателем: 5 0 , (a + 1) 0 , 3 + 5 2 − 3 , 2 0 . И степени с целыми отрицательными степенями: (0 , 5) 2 + (0 , 5) — 2 2 .

    Чуть сложнее работать со степенью, имеющей рациональный и иррациональный показатели: 264 1 4 — 3 · 3 · 3 1 2 , 2 3 , 5 · 2 — 2 2 — 1 , 5 , 1 a 1 4 · a 1 2 — 2 · a — 1 6 · b 1 2 , x π · x 1 — π , 2 3 3 + 5 .

    В качестве показателя может выступать переменная 3 x — 54 — 7 · 3 x — 58 или логарифм x 2 · l g x − 5 · x l g x
    .

    С вопросом о том, что такое степенные выражения, мы разобрались. Теперь займемся их преобразованием.

    Основные виды преобразований степенных выражений

    В первую очередь мы рассмотрим основные тождественные преобразования выражений, которые можно выполнять со степенными выражениями.

    Пример 1

    Вычислите значение степенного выражения 2 3 · (4 2 − 12)
    .

    Решение

    Все преобразования мы будем проводить с соблюдением порядка выполнения действий. В данном случае начнем мы с выполнения действий в скобках: заменим степень на цифровое значение и вычислим разность двух чисел. Имеем 2 3 · (4 2 − 12) = 2 3 · (16 − 12) = 2 3 · 4
    .

    Нам остается заменить степень 2 3
    ее значением 8
    и вычислить произведение 8 · 4 = 32
    . Вот наш ответ.

    Ответ:
    2 3 · (4 2 − 12) = 32 .

    Пример 2

    Упростите выражение со степенями 3 · a 4 · b − 7 − 1 + 2 · a 4 · b − 7
    .

    Решение

    Данное нам в условии задачи выражение содержит подобные слагаемые, которые мы можем привести: 3 · a 4 · b − 7 − 1 + 2 · a 4 · b − 7 = 5 · a 4 · b − 7 − 1
    .

    Ответ:
    3 · a 4 · b − 7 − 1 + 2 · a 4 · b − 7 = 5 · a 4 · b − 7 − 1 .

    Пример 3

    Представьте выражение со степенями 9 — b 3 · π — 1 2 в виде произведения.

    Решение

    Представим число 9 как степень 3 2
    и применим формулу сокращенного умножения:

    9 — b 3 · π — 1 2 = 3 2 — b 3 · π — 1 2 = = 3 — b 3 · π — 1 3 + b 3 · π — 1

    Ответ:
    9 — b 3 · π — 1 2 = 3 — b 3 · π — 1 3 + b 3 · π — 1 .

    А теперь перейдем к разбору тождественных преобразований, которые могут применяться именно в отношении степенных выражений.

    Работа с основанием и показателем степени

    Степень в основании или показателе может иметь и числа, и переменные, и некоторые выражения. Например, (2 + 0 , 3 · 7) 5 − 3 , 7
    и . Работать с такими записями сложно. Намного проще заменить выражение в основании степени или выражение в показателе тождественно равным выражением.

    Проводятся преобразования степени и показателя по известным нам правилам отдельно друг от друга. Самое главное, чтобы в результате преобразований получилось выражение, тождественное исходному.

    Цель преобразований – упростить исходное выражение или получить решение задачи. Например, в примере, который мы привели выше, (2 + 0 , 3 · 7) 5 − 3 , 7 можно выполнить действия для перехода к степени 4 , 1 1 , 3
    . Раскрыв скобки, мы можем привести подобные слагаемые в основании степени (a · (a + 1) − a 2) 2 · (x + 1)
    и получить степенное выражение более простого вида a 2 · (x + 1)
    .

    Использование свойств степеней

    Свойства степеней, записанные в виде равенств, являются одним из главных инструментов преобразования выражений со степенями. Приведем здесь основные из них, учитывая, что a
    и b
    – это любые положительные числа, а r
    и s
    — произвольные действительные числа:

    Определение 2

    • a r · a s = a r + s ;
    • a r: a s = a r − s ;
    • (a · b) r = a r · b r ;
    • (a: b) r = a r: b r ;
    • (a r) s = a r · s .

    В тех случаях, когда мы имеем дело с натуральными, целыми, положительными показателями степени, ограничения на числа a и b могут быть гораздо менее строгими. Так, например, если рассмотреть равенство a m · a n = a m + n
    , где m
    и n
    – натуральные числа, то оно будет верно для любых значений a , как положительных, так и отрицательных, а также для a = 0
    .

    Применять свойства степеней без ограничений можно в тех случаях, когда основания степеней положительные или содержат переменные, область допустимых значений которых такова, что на ней основания принимают лишь положительные значения. Фактически, в рамках школьной программы по математике задачей учащегося является выбор подходящего свойства и правильное его применение.

    При подготовке к поступлению в Вузы могут встречаться задачи, в которых неаккуратное применение свойств будет приводить к сужению ОДЗ и другим сложностям с решением. В данном разделе мы разберем всего два таких случая. Больше информации по вопросу можно найти в теме «Преобразование выражений с использованием свойств степеней».

    Пример 4

    Представьте выражение a 2 , 5 · (a 2) − 3: a − 5 , 5
    в виде степени с основанием a
    .

    Решение

    Для начала используем свойство возведения в степень и преобразуем по нему второй множитель (a 2) − 3
    . Затем используем свойства умножения и деления степеней с одинаковым основанием:

    a 2 , 5 · a − 6: a − 5 , 5 = a 2 , 5 − 6: a − 5 , 5 = a − 3 , 5: a − 5 , 5 = a − 3 , 5 − (− 5 , 5) = a 2 .

    Ответ:
    a 2 , 5 · (a 2) − 3: a − 5 , 5 = a 2 .

    Преобразование степенных выражений согласно свойству степеней может производиться как слева направо, так и в обратном направлении.

    Пример 5

    Найти значение степенного выражения 3 1 3 · 7 1 3 · 21 2 3 .

    Решение

    Если мы применим равенство (a · b) r = a r · b r
    , справа налево, то получим произведение вида 3 · 7 1 3 · 21 2 3 и дальше 21 1 3 · 21 2 3 . Сложим показатели при умножении степеней с одинаковыми основаниями: 21 1 3 · 21 2 3 = 21 1 3 + 2 3 = 21 1 = 21 .

    Есть еще один способ провести преобразования:

    3 1 3 · 7 1 3 · 21 2 3 = 3 1 3 · 7 1 3 · (3 · 7) 2 3 = 3 1 3 · 7 1 3 · 3 2 3 · 7 2 3 = = 3 1 3 · 3 2 3 · 7 1 3 · 7 2 3 = 3 1 3 + 2 3 · 7 1 3 + 2 3 = 3 1 · 7 1 = 21

    Ответ:
    3 1 3 · 7 1 3 · 21 2 3 = 3 1 · 7 1 = 21

    Пример 6

    Дано степенное выражение a 1 , 5 − a 0 , 5 − 6
    , введите новую переменную t = a 0 , 5
    .

    Решение

    Представим степень a 1 , 5
    как a 0 , 5 · 3
    . Используем свойство степени в степени (a r) s = a r · s
    справа налево и получим (a 0 , 5) 3: a 1 , 5 − a 0 , 5 − 6 = (a 0 , 5) 3 − a 0 , 5 − 6 . В полученное выражение можно без проблем вводить новую переменную t = a 0 , 5
    : получаем t 3 − t − 6
    .

    Ответ:
    t 3 − t − 6 .

    Преобразование дробей, содержащих степени

    Обычно мы имеем дело с двумя вариантами степенных выражений с дробями: выражение представляет собой дробь со степенью или содержит такую дробь. К таким выражениям применимы все основные преобразования дробей без ограничений. Их можно сокращать, приводить к новому знаменателю, работать отдельно с числителем и знаменателем. Проиллюстрируем это примерами.

    Пример 7

    Упростить степенное выражение 3 · 5 2 3 · 5 1 3 — 5 — 2 3 1 + 2 · x 2 — 3 — 3 · x 2 .

    Решение

    Мы имеем дело с дробью, поэтому проведем преобразования и в числителе, и в знаменателе:

    3 · 5 2 3 · 5 1 3 — 5 — 2 3 1 + 2 · x 2 — 3 — 3 · x 2 = 3 · 5 2 3 · 5 1 3 — 3 · 5 2 3 · 5 — 2 3 — 2 — x 2 = = 3 · 5 2 3 + 1 3 — 3 · 5 2 3 + — 2 3 — 2 — x 2 = 3 · 5 1 — 3 · 5 0 — 2 — x 2

    Поместим минус перед дробью для того, чтобы изменить знак знаменателя: 12 — 2 — x 2 = — 12 2 + x 2

    Ответ:
    3 · 5 2 3 · 5 1 3 — 5 — 2 3 1 + 2 · x 2 — 3 — 3 · x 2 = — 12 2 + x 2

    Дроби, содержащие степени, приводятся к новому знаменателю точно также, как и рациональные дроби. Для этого необходимо найти дополнительный множитель и умножить на него числитель и знаменатель дроби. Подбирать дополнительный множитель необходимо таким образом, чтобы он не обращался в нуль ни при каких значениях переменных из ОДЗ переменных для исходного выражения.

    Пример 8

    Приведите дроби к новому знаменателю: а) a + 1 a 0 , 7 к знаменателю a
    , б) 1 x 2 3 — 2 · x 1 3 · y 1 6 + 4 · y 1 3 к знаменателю x + 8 · y 1 2 .

    Решение

    а) Подберем множитель, который позволит нам произвести приведение к новому знаменателю. a 0 , 7 · a 0 , 3 = a 0 , 7 + 0 , 3 = a ,
    следовательно, в качестве дополнительного множителя мы возьмем a 0 , 3
    . Область допустимых значений переменной а включает множество всех положительных действительных чисел. В этой области степень a 0 , 3
    не обращается в нуль.

    Выполним умножение числителя и знаменателя дроби на a 0 , 3
    :

    a + 1 a 0 , 7 = a + 1 · a 0 , 3 a 0 , 7 · a 0 , 3 = a + 1 · a 0 , 3 a

    б) Обратим внимание на знаменатель:

    x 2 3 — 2 · x 1 3 · y 1 6 + 4 · y 1 3 = = x 1 3 2 — x 1 3 · 2 · y 1 6 + 2 · y 1 6 2

    Умножим это выражение на x 1 3 + 2 · y 1 6 , получим сумму кубов x 1 3 и 2 · y 1 6 , т. е. x + 8 · y 1 2 . Это наш новый знаменатель, к которому нам надо привести исходную дробь.

    Так мы нашли дополнительный множитель x 1 3 + 2 · y 1 6 . На области допустимых значений переменных x
    и y
    выражение x 1 3 + 2 · y 1 6 не обращается в нуль, поэтому, мы можем умножить на него числитель и знаменатель дроби:
    1 x 2 3 — 2 · x 1 3 · y 1 6 + 4 · y 1 3 = = x 1 3 + 2 · y 1 6 x 1 3 + 2 · y 1 6 x 2 3 — 2 · x 1 3 · y 1 6 + 4 · y 1 3 = = x 1 3 + 2 · y 1 6 x 1 3 3 + 2 · y 1 6 3 = x 1 3 + 2 · y 1 6 x + 8 · y 1 2

    Ответ:
    а) a + 1 a 0 , 7 = a + 1 · a 0 , 3 a , б) 1 x 2 3 — 2 · x 1 3 · y 1 6 + 4 · y 1 3 = x 1 3 + 2 · y 1 6 x + 8 · y 1 2 .

    Пример 9

    Сократите дробь: а) 30 · x 3 · (x 0 , 5 + 1) · x + 2 · x 1 1 3 — 5 3 45 · x 0 , 5 + 1 2 · x + 2 · x 1 1 3 — 5 3 , б) a 1 4 — b 1 4 a 1 2 — b 1 2 .

    Решение

    а) Используем наибольший общий знаменатель (НОД), на который можно сократить числитель и знаменатель. Для чисел 30 и 45 это 15 . Также мы можем произвести сокращение на x 0 , 5 + 1
    и на x + 2 · x 1 1 3 — 5 3 .

    Получаем:

    30 · x 3 · (x 0 , 5 + 1) · x + 2 · x 1 1 3 — 5 3 45 · x 0 , 5 + 1 2 · x + 2 · x 1 1 3 — 5 3 = 2 · x 3 3 · (x 0 , 5 + 1)

    б) Здесь наличие одинаковых множителей неочевидно. Придется выполнить некоторые преобразования для того, чтобы получить одинаковые множители в числителе и знаменателе. Для этого разложим знаменатель, используя формулу разности квадратов:

    a 1 4 — b 1 4 a 1 2 — b 1 2 = a 1 4 — b 1 4 a 1 4 2 — b 1 2 2 = = a 1 4 — b 1 4 a 1 4 + b 1 4 · a 1 4 — b 1 4 = 1 a 1 4 + b 1 4

    Ответ:
    а) 30 · x 3 · (x 0 , 5 + 1) · x + 2 · x 1 1 3 — 5 3 45 · x 0 , 5 + 1 2 · x + 2 · x 1 1 3 — 5 3 = 2 · x 3 3 · (x 0 , 5 + 1) , б) a 1 4 — b 1 4 a 1 2 — b 1 2 = 1 a 1 4 + b 1 4 .

    К числу основных действий с дробями относится приведение к новому знаменателю и сокращение дробей. Оба действия выполняют с соблюдением ряда правил. При сложении и вычитании дробей сначала дроби приводятся к общему знаменателю, после чего проводятся действия (сложение или вычитание) с числителями. Знаменатель остается прежним. Результатом наших действий является новая дробь, числитель которой является произведением числителей, а знаменатель есть произведение знаменателей.

    Пример 10

    Выполните действия x 1 2 + 1 x 1 2 — 1 — x 1 2 — 1 x 1 2 + 1 · 1 x 1 2 .

    Решение

    Начнем с вычитания дробей, которые располагаются в скобках. Приведем их к общему знаменателю:

    x 1 2 — 1 · x 1 2 + 1

    Вычтем числители:

    x 1 2 + 1 x 1 2 — 1 — x 1 2 — 1 x 1 2 + 1 · 1 x 1 2 = = x 1 2 + 1 · x 1 2 + 1 x 1 2 — 1 · x 1 2 + 1 — x 1 2 — 1 · x 1 2 — 1 x 1 2 + 1 · x 1 2 — 1 · 1 x 1 2 = = x 1 2 + 1 2 — x 1 2 — 1 2 x 1 2 — 1 · x 1 2 + 1 · 1 x 1 2 = = x 1 2 2 + 2 · x 1 2 + 1 — x 1 2 2 — 2 · x 1 2 + 1 x 1 2 — 1 · x 1 2 + 1 · 1 x 1 2 = = 4 · x 1 2 x 1 2 — 1 · x 1 2 + 1 · 1 x 1 2

    Теперь умножаем дроби:

    4 · x 1 2 x 1 2 — 1 · x 1 2 + 1 · 1 x 1 2 = = 4 · x 1 2 x 1 2 — 1 · x 1 2 + 1 · x 1 2

    Произведем сокращение на степень x 1 2
    , получим 4 x 1 2 — 1 · x 1 2 + 1 .

    Дополнительно можно упростить степенное выражение в знаменателе, используя формулу разности квадратов: квадратов: 4 x 1 2 — 1 · x 1 2 + 1 = 4 x 1 2 2 — 1 2 = 4 x — 1 .

    Ответ:
    x 1 2 + 1 x 1 2 — 1 — x 1 2 — 1 x 1 2 + 1 · 1 x 1 2 = 4 x — 1

    Пример 11

    Упростите степенное выражение x 3 4 · x 2 , 7 + 1 2 x — 5 8 · x 2 , 7 + 1 3 .
    Решение

    Мы можем произвести сокращение дроби на (x 2 , 7 + 1) 2
    . Получаем дробь x 3 4 x — 5 8 · x 2 , 7 + 1 .

    Продолжим преобразования степеней икса x 3 4 x — 5 8 · 1 x 2 , 7 + 1 . Теперь можно использовать свойство деления степеней с одинаковыми основаниями: x 3 4 x — 5 8 · 1 x 2 , 7 + 1 = x 3 4 — — 5 8 · 1 x 2 , 7 + 1 = x 1 1 8 · 1 x 2 , 7 + 1 .

    Переходим от последнего произведения к дроби x 1 3 8 x 2 , 7 + 1 .

    Ответ:
    x 3 4 · x 2 , 7 + 1 2 x — 5 8 · x 2 , 7 + 1 3 = x 1 3 8 x 2 , 7 + 1 .

    Множители с отрицательными показателями степени в большинстве случаев удобнее переносить из числителя в знаменатель и обратно, изменяя знак показателя. Это действие позволяет упростить дальнейшее решение. Приведем пример: степенное выражение (x + 1) — 0 , 2 3 · x — 1 можно заменить на x 3 · (x + 1) 0 , 2 .

    Преобразование выражений с корнями и степенями

    В задачах встречаются степенные выражения, которые содержат не только степени с дробными показателями, но и корни. Такие выражения желательно привести только к корням или только к степеням. Переход к степеням предпочтительнее, так как с ними проще работать. Такой переход является особенно предпочтительным, когда ОДЗ переменных для исходного выражения позволяет заменить корни степенями без необходимости обращаться к модулю или разбивать ОДЗ на несколько промежутков.

    Пример 12

    Представьте выражение x 1 9 · x · x 3 6 в виде степени.

    Решение

    Область допустимых значений переменной x
    определяется двумя неравенствами x ≥ 0
    и x · x 3 ≥ 0 , которые задают множество [ 0 , + ∞)
    .

    На этом множестве мы имеем право перейти от корней к степеням:

    x 1 9 · x · x 3 6 = x 1 9 · x · x 1 3 1 6

    Используя свойства степеней, упростим полученное степенное выражение.

    x 1 9 · x · x 1 3 1 6 = x 1 9 · x 1 6 · x 1 3 1 6 = x 1 9 · x 1 6 · x 1 · 1 3 · 6 = = x 1 9 · x 1 6 · x 1 18 = x 1 9 + 1 6 + 1 18 = x 1 3

    Ответ:
    x 1 9 · x · x 3 6 = x 1 3 .

    Преобразование степеней с переменными в показателе

    Данные преобразования достаточно просто произвести, если грамотно использовать свойства степени. Например, 5 2 · x + 1 − 3 · 5 x · 7 x − 14 · 7 2 · x − 1 = 0
    .

    Мы можем заменить произведением степени, в показателях которых находится сумма некоторой переменной и числа. В левой части это можно проделать с первым и последним слагаемыми левой части выражения:

    5 2 · x · 5 1 − 3 · 5 x · 7 x − 14 · 7 2 · x · 7 − 1 = 0 , 5 · 5 2 · x − 3 · 5 x · 7 x − 2 · 7 2 · x = 0 .

    Теперь поделим обе части равенства на 7 2 · x
    . Это выражение на ОДЗ переменной x принимает только положительные значения:

    5 · 5 — 3 · 5 x · 7 x — 2 · 7 2 · x 7 2 · x = 0 7 2 · x , 5 · 5 2 · x 7 2 · x — 3 · 5 x · 7 x 7 2 · x — 2 · 7 2 · x 7 2 · x = 0 , 5 · 5 2 · x 7 2 · x — 3 · 5 x · 7 x 7 x · 7 x — 2 · 7 2 · x 7 2 · x = 0

    Сократим дроби со степенями, получим: 5 · 5 2 · x 7 2 · x — 3 · 5 x 7 x — 2 = 0 .

    Наконец, отношение степеней с одинаковыми показателями заменяется степенями отношений, что приводит к уравнению 5 · 5 7 2 · x — 3 · 5 7 x — 2 = 0 , которое равносильно 5 · 5 7 x 2 — 3 · 5 7 x — 2 = 0 .

    Введем новую переменную t = 5 7 x , что сводит решение исходного показательного уравнения к решению квадратного уравнения 5 · t 2 − 3 · t − 2 = 0 .

    Преобразование выражений со степенями и логарифмами

    Выражения, содержащие с записи степени и логарифмы, также встречаются в задачах. Примером таких выражений могут служить: 1 4 1 — 5 · log 2 3 или log 3 27 9 + 5 (1 — log 3 5) · log 5 3 . Преобразование подобных выражений проводится с использованием разобранных выше подходов и свойств логарифмов, которые мы подробно разобрали в теме «Преобразование логарифмических выражений».

    Если вы заметили ошибку в тексте, пожалуйста, выделите её и нажмите Ctrl+Enter

    Степень используется для упрощения записи операции умножения числа само на себя. Например, вместо записи можно написать 4 5 {\displaystyle 4^{5}}

    (объяснение такому переходу дано в первом разделе этой статьи). {a}ix=cosax+isinax}
    , где i = (− 1) {\displaystyle i={\sqrt {(}}-1)}
    ; е — константа, примерно равная 2,7; а — произвольная постоянная. Доказательство этого равенства можно найти в любом учебнике по высшей математике.

    Предупреждения

    • При увеличении показателя степени ее значение сильно возрастает. Поэтому если ответ кажется вам неправильным, на самом деле он может оказаться верным. Вы можете проверить это, построив график любой показательной функции, например, 2 x .

    Одной из главных характеристик в алгебре, да и во всей математике является степень. Конечно, в 21 веке все расчеты можно проводить на онлайн-калькуляторе, но лучше для развития мозгов научиться делать это самому.

    В данной статье рассмотрим самые важные вопросы, касающиеся этого определения. А именно, поймем что это вообще такое и каковы основные его функции, какие имеются свойства в математике.

    Рассмотрим на примерах то, как выглядит расчет, каковы основные формулы. Разберем основные виды величины и то, чем они отличаются от других функций.

    Поймем, как решать с помощью этой величины различные задачи. Покажем на примерах, как возводить в нулевую степень, иррациональную, отрицательную и др.

    Онлайн-калькулятор возведения в степень

    Что такое степень числа

    Что же подразумевают под выражением «возвести число в степень»?

    Степенью n числа а является произведение множителей величиной а n-раз подряд.

    Математически это выглядит следующим образом:

    a n = a * a * a * …a n .

    Например:

    • 2 3 = 2 в третьей степ. = 2 * 2 * 2 = 8;
    • 4 2 = 4 в степ. два = 4 * 4 = 16;
    • 5 4 = 5 в степ. четыре = 5 * 5 * 5 * 5 = 625;
    • 10 5 = 10 в 5 степ. = 10 * 10 * 10 * 10 * 10 = 100000;
    • 10 4 = 10 в 4 степ. = 10 * 10 * 10 * 10 = 10000.

    Ниже будет представлена таблица квадратов и кубов от 1 до 10.

    Таблица степеней от 1 до 10

    Ниже будут приведены результаты возведения натуральных чисел в положительные степени – «от 1 до 100».

    Ч-ло
    2-ая ст-нь
    3-я ст-нь
    1
    1
    1
    2
    4
    8
    3
    9
    27
    4
    16
    64
    5
    25
    125
    6
    36
    216
    7
    49
    343
    8
    64
    512
    9
    81
    279
    10
    100
    1000

    Свойства степеней

    Что же характерно для такой математической функции? Рассмотрим базовые свойства.

    Учеными установлено следующие признаки, характерные для всех степеней:

    • a n * a m = (a) (n+m) ;
    • a n: a m = (a) (n-m) ;
    • (a b) m =(a) (b*m) .

    Проверим на примерах:

    2 3 * 2 2 = 8 * 4 = 32. С другой стороны 2 5 = 2 * 2 * 2 * 2 * 2 =32.

    Аналогично: 2 3: 2 2 = 8 / 4 =2. Иначе 2 3-2 = 2 1 =2.

    (2 3) 2 = 8 2 = 64. А если по-другому? 2 6 = 2 * 2 * 2 * 2 * 2 * 2 = 32 * 2 = 64.

    Как видим, правила работают.

    А как же быть со сложением и вычитанием
    ? Всё просто. Выполняется сначала возведение в степень, а уж потом сложение и вычитание.

    Посмотрим на примерах:

    • 3 3 + 2 4 = 27 + 16 = 43;
    • 5 2 – 3 2 = 25 – 9 = 16. Обратите внимание: правило не будет выполняться, если сначала произвести вычитание: (5 — 3) 2 = 2 2 = 4.

    А вот в этом случае надо вычислять сначала сложение, поскольку присутствуют действия в скобках: (5 + 3) 3 = 8 3 = 512.

    Как производить вычисления в более сложных случаях
    ? Порядок тот же:

    • при наличии скобок – начинать нужно с них;
    • затем возведение в степень;
    • потом выполнять действия умножения, деления;
    • после сложение, вычитание.

    Есть специфические свойства, характерные не для всех степеней:

    1. Корень n-ой степени из числа a в степени m запишется в виде: a m / n .
    2. При возведении дроби в степень: этой процедуре подвержены как числитель, так и ее знаменатель.
    3. При возведении произведения разных чисел в степень, выражение будет соответствовать произведению этих чисел в заданной степени. То есть: (a * b) n = a n * b n .
    4. При возведении числа в отрицательную степ., нужно разделить 1 на число в той же ст-ни, но со знаком «+».
    5. Если знаменатель дроби находится в отрицательной степени, то это выражение будет равно произведению числителя на знаменатель в положительной степени.
    6. Любое число в степени 0 = 1, а в степ. 1 = самому себе.

    Эти правила важны в отдельных случаях, их рассмотрим подробней ниже.

    Степень с отрицательным показателем

    Что делать при минусовой степени, т. е. когда показатель отрицательный?

    Исходя из свойств 4 и 5
    (смотри пункт выше), получается
    :

    A (- n) = 1 / A n , 5 (-2) = 1 / 5 2 = 1 / 25.

    И наоборот:

    1 / A (- n) = A n , 1 / 2 (-3) = 2 3 = 8.

    А если дробь?

    (A / B) (- n) = (B / A) n , (3 / 5) (-2) = (5 / 3) 2 = 25 / 9.

    Степень с натуральным показателем

    Под ней понимают степень с показателями, равными целым числам.

    Что нужно запомнить:

    A 0 = 1, 1 0 = 1; 2 0 = 1; 3.15 0 = 1; (-4) 0 = 1…и т. д.

    A 1 = A, 1 1 = 1; 2 1 = 2; 3 1 = 3…и т. д.

    Кроме того, если (-a) 2 n +2 , n=0, 1, 2…то результат будет со знаком «+». Если отрицательное число возводится в нечетную степень, то наоборот.

    Общие свойства, да и все специфические признаки, описанные выше, также характерны для них.

    Дробная степень

    Этот вид можно записать схемой: A m / n . Читается как: корень n-ой степени из числа A в степени m.

    С дробным показателем можно делать, что угодно: сокращать, раскладывать на части, возводить в другую степень и т. д.

    Степень с иррациональным показателем

    Пусть α – иррациональное число, а А ˃ 0.

    Чтобы понять суть степени с таким показателем, рассмотрим разные возможные случаи:

    • А = 1. Результат будет равен 1. Поскольку существует аксиома – 1 во всех степенях равна единице;

    А r 1 ˂ А α ˂ А r 2 , r 1 ˂ r 2 – рациональные числа;

    В этом случае наоборот: А r 2 ˂ А α ˂ А r 1 при тех же условиях, что и во втором пункте.

    Например, показатель степени число π.
    Оно рациональное.

    r 1 – в этом случае равно 3;

    r 2 – будет равно 4.

    Тогда, при А = 1, 1 π = 1.

    А = 2, то 2 3 ˂ 2 π ˂ 2 4 , 8 ˂ 2 π ˂ 16.

    А = 1/2, то (½) 4 ˂ (½) π ˂ (½) 3 , 1/16 ˂ (½) π ˂ 1/8.

    Для таких степеней характерны все математические операции и специфические свойства, описанные выше.

    Заключение

    Подведём итоги — для чего же нужны эти величины, в чем преимущество таких функций? Конечно, в первую очередь они упрощают жизнь математиков и программистов при решении примеров, поскольку позволяют минимизировать расчеты, сократить алгоритмы, систематизировать данные и многое другое.

    Где еще могут пригодиться эти знания? В любой рабочей специальности: медицине, фармакологии, стоматологии, строительстве, технике, инженерии, конструировании и т. д.

    На канал на youtube нашего сайта сайт, чтобы быть в курсе всех новых видео уроков.

    Для начала вспомним основные формулы степеней и их свойства.

    Произведение числа a
    само на себя происходит n раз, это выражение мы можем записать как a a … a=a n

    1. a 0 = 1 (a ≠ 0)

    3. a n a m = a n + m

    4. (a n) m = a nm

    5. a n b n = (ab) n

    7. a n /a m = a n — m

    Степенные или показательные уравнения
    – это уравнения в которых переменные находятся в степенях (или показателях), а основанием является число.

    Примеры показательных уравнений:

    В данном примере число 6 является основанием оно всегда стоит внизу, а переменная x
    степенью или показателем.

    Приведем еще примеры показательных уравнений.
    2 x *5=10
    16 x — 4 x — 6=0

    Теперь разберем как решаются показательные уравнения?

    Возьмем простое уравнение:

    2 х = 2 3

    Такой пример можно решить даже в уме. Видно, что x=3. Ведь чтобы левая и правая часть были равны нужно вместо x поставить число 3.
    А теперь посмотрим как нужно это решение оформить:

    2 х = 2 3
    х = 3

    Для того, чтобы решить такое уравнение, мы убрали одинаковые основания
    (то есть двойки) и записали то что осталось, это степени. Получили искомый ответ.

    Теперь подведем итоги нашего решения.

    Алгоритм решения показательного уравнения:

    1. Нужно проверить одинаковые
    ли основания у уравнения справа и слева. Если основания не одинаковые ищем варианты для решения данного примера.
    2. После того как основания станут одинаковыми, приравниваем
    степени и решаем полученное новое уравнение.

    Теперь прорешаем несколько примеров:

    Начнем с простого.

    Основания в левой и правой части равны числу 2, значит мы можем основание отбросить и приравнять их степени.

    x+2=4 Получилось простейшее уравнение.
    x=4 — 2
    x=2
    Ответ: x=2

    В следующем примере видно, что основания разные это 3 и 9.

    3 3х — 9 х+8 = 0

    Для начала переносим девятку в правую сторону, получаем:

    Теперь нужно сделать одинаковые основания. Мы знаем что 9=3 2 . Воспользуемся формулой степеней (a n) m = a nm .

    3 3х = (3 2) х+8

    Получим 9 х+8 =(3 2) х+8 =3 2х+16

    3 3х = 3 2х+16 теперь видно что в левой и правой стороне основания одинаковые и равные тройке, значит мы их можем отбросить и приравнять степени.

    3x=2x+16 получили простейшее уравнение
    3x — 2x=16
    x=16
    Ответ: x=16.

    Смотрим следующий пример:

    2 2х+4 — 10 4 х = 2 4

    В первую очередь смотрим на основания, основания разные два и четыре. А нам нужно, чтобы были — одинаковые. Преобразовываем четверку по формуле (a n) m = a nm .

    4 х = (2 2) х = 2 2х

    И еще используем одну формулу a n a m = a n + m:

    2 2х+4 = 2 2х 2 4

    Добавляем в уравнение:

    2 2х 2 4 — 10 2 2х = 24

    Мы привели пример к одинаковым основаниям. Но нам мешают другие числа 10 и 24. Что с ними делать? Если приглядеться видно, что в левой части у нас повторяется 2 2х,вот и ответ — 2 2х мы можем вынести за скобки:

    2 2х (2 4 — 10) = 24

    Посчитаем выражение в скобках:

    2 4 — 10 = 16 — 10 = 6

    Все уравнение делим на 6:

    Представим 4=2 2:

    2 2х = 2 2 основания одинаковые, отбрасываем их и приравниваем степени.
    2х = 2 получилось простейшее уравнение. Делим его на 2 получаем
    х = 1
    Ответ: х = 1.

    Решим уравнение:

    9 х – 12*3 х +27= 0

    Преобразуем:
    9 х = (3 2) х = 3 2х

    Получаем уравнение:
    3 2х — 12 3 х +27 = 0

    Основания у нас одинаковы равны трем.В данном примере видно, что у первой тройки степень в два раза (2x) больше, чем у второй (просто x). В таком случаем можно решить методом замены
    . Число с наименьшей степенью заменяем:

    Тогда 3 2х = (3 х) 2 = t 2

    Заменяем в уравнении все степени с иксами на t:

    t 2 — 12t+27 = 0
    Получаем квадратное уравнение. Решаем через дискриминант, получаем:
    D=144-108=36
    t 1 = 9
    t 2 = 3

    Возвращаемся к переменной x
    .

    Берем t 1:
    t 1 = 9 = 3 х

    Стало быть,

    3 х = 9
    3 х = 3 2
    х 1 = 2

    Один корень нашли. Ищем второй, из t 2:
    t 2 = 3 = 3 х
    3 х = 3 1
    х 2 = 1
    Ответ: х 1 = 2; х 2 = 1.

    На сайте Вы можете в разделе ПОМОГИТЕ РЕШИТЬ задавать интересующие вопросы мы Вам обязательно ответим.

    Вступайте в группу

    Как найти значение выражения со степенями

    Формулировка задачи: Найдите значение выражения (степени, с разными основаниями).

    Задача входит в состав ЕГЭ по математике базового уровня для 11 класса под номером 2 (Действия со степенями).

    Рассмотрим, как решаются подобные задачи на примере.

    Пример задачи 1:

    Найдите значение выражения 8 0,76 ∙ 64 0,12 .

    Найдем значение выражения. Для этого приведем числа к одинаковому основанию и выполним необходимые действия:

    Пример задачи 2:

    Найдите значение выражения:

    Найдем значение выражения. Для этого приведем числа к одинаковому основанию и выполним необходимые действия:

    Пример задачи 3:

    Найдите значение выражения 35 -4,7 ∙ 7 5,7 : 5 -3,7 .

    Найдем значение выражения. Для этого приведем числа к одинаковому основанию и выполним необходимые действия:

    Пример задачи 4:

    Найдите значение выражения:

    Найдем значение выражения. Для этого приведем числа к одинаковому основанию и выполним необходимые действия:

    Поделитесь статьей с одноклассниками «Найдите значение выражения (степени, с разными основаниями) – как решать».

    Есть другой способ решения?

    Предложите другой способ решения задачи «Найдите значение выражения (степени, с разными основаниями)». Возможно, он окажется более понятным для кого-нибудь:

    Рассмотрим тему преобразования выражений со степенями, но прежде остановимся на ряде преобразований, которые можно проводить с любыми выражениями, в том числе со степенными. Мы научимся раскрывать скобки, приводить подобные слагаемые, работать с основанием и показателем степени, использовать свойства степеней.

    Что представляют собой степенные выражения?

    В школьном курсе мало кто использует словосочетание «степенные выражения», зато этот термин постоянно встречается в сборниках для подготовки к ЕГЭ. В большинства случаев словосочетанием обозначаются выражения, которые содержат в своих записях степени. Это мы и отразим в нашем определении.

    Степенное выражение – это выражение, которое содержит степени.

    Приведем несколько примеров степенных выражений, начиная со степени с натуральным показателем и заканчивая степенью с действительным показателем.

    Самыми простыми степенными выражениями можно считать степени числа с натуральным показателем: 3 2 , 7 5 + 1 , ( 2 + 1 ) 5 , ( − 0 , 1 ) 4 , 2 2 3 3 , 3 · a 2 − a + a 2 , x 3 − 1 , ( a 2 ) 3 . А также степени с нулевым показателем: 5 0 , ( a + 1 ) 0 , 3 + 5 2 − 3 , 2 0 . И степени с целыми отрицательными степенями: ( 0 , 5 ) 2 + ( 0 , 5 ) – 2 2 .

    Чуть сложнее работать со степенью, имеющей рациональный и иррациональный показатели: 264 1 4 – 3 · 3 · 3 1 2 , 2 3 , 5 · 2 – 2 2 – 1 , 5 , 1 a 1 4 · a 1 2 – 2 · a – 1 6 · b 1 2 , x π · x 1 – π , 2 3 3 + 5 .

    В качестве показателя может выступать переменная 3 x – 54 – 7 · 3 x – 58 или логарифм x 2 · l g x − 5 · x l g x .

    С вопросом о том, что такое степенные выражения, мы разобрались. Теперь займемся их преобразованием.

    Основные виды преобразований степенных выражений

    В первую очередь мы рассмотрим основные тождественные преобразования выражений, которые можно выполнять со степенными выражениями.

    Вычислите значение степенного выражения 2 3 · ( 4 2 − 12 ) .

    Решение

    Все преобразования мы будем проводить с соблюдением порядка выполнения действий. В данном случае начнем мы с выполнения действий в скобках: заменим степень на цифровое значение и вычислим разность двух чисел. Имеем 2 3 · ( 4 2 − 12 ) = 2 3 · ( 16 − 12 ) = 2 3 · 4 .

    Нам остается заменить степень 2 3 ее значением 8 и вычислить произведение 8 · 4 = 32 . Вот наш ответ.

    Ответ: 2 3 · ( 4 2 − 12 ) = 32 .

    Упростите выражение со степенями 3 · a 4 · b − 7 − 1 + 2 · a 4 · b − 7 .

    Решение

    Данное нам в условии задачи выражение содержит подобные слагаемые, которые мы можем привести: 3 · a 4 · b − 7 − 1 + 2 · a 4 · b − 7 = 5 · a 4 · b − 7 − 1 .

    Ответ: 3 · a 4 · b − 7 − 1 + 2 · a 4 · b − 7 = 5 · a 4 · b − 7 − 1 .

    Представьте выражение со степенями 9 – b 3 · π – 1 2 в виде произведения.

    Решение

    Представим число 9 как степень 3 2 и применим формулу сокращенного умножения:

    9 – b 3 · π – 1 2 = 3 2 – b 3 · π – 1 2 = = 3 – b 3 · π – 1 3 + b 3 · π – 1

    Ответ: 9 – b 3 · π – 1 2 = 3 – b 3 · π – 1 3 + b 3 · π – 1 .

    А теперь перейдем к разбору тождественных преобразований, которые могут применяться именно в отношении степенных выражений.

    Работа с основанием и показателем степени

    Степень в основании или показателе может иметь и числа, и переменные, и некоторые выражения. Например, ( 2 + 0 , 3 · 7 ) 5 − 3 , 7 и ( a · ( a + 1 ) − a 2 ) 2 · ( x + 1 ) . Работать с такими записями сложно. Намного проще заменить выражение в основании степени или выражение в показателе тождественно равным выражением.

    Проводятся преобразования степени и показателя по известным нам правилам отдельно друг от друга. Самое главное, чтобы в результате преобразований получилось выражение, тождественное исходному.

    Цель преобразований – упростить исходное выражение или получить решение задачи. Например, в примере, который мы привели выше, ( 2 + 0 , 3 · 7 ) 5 − 3 , 7 можно выполнить действия для перехода к степени 4 , 1 1 , 3 . Раскрыв скобки, мы можем привести подобные слагаемые в основании степени ( a · ( a + 1 ) − a 2 ) 2 · ( x + 1 ) и получить степенное выражение более простого вида a 2 · ( x + 1 ) .

    Использование свойств степеней

    Свойства степеней, записанные в виде равенств, являются одним из главных инструментов преобразования выражений со степенями. Приведем здесь основные из них, учитывая, что a и b – это любые положительные числа, а r и s – произвольные действительные числа:

    • a r · a s = a r + s ;
    • a r : a s = a r − s ;
    • ( a · b ) r = a r · b r ;
    • ( a : b ) r = a r : b r ;
    • ( a r ) s = a r · s .

    В тех случаях, когда мы имеем дело с натуральными, целыми, положительными показателями степени, ограничения на числа a и b могут быть гораздо менее строгими. Так, например, если рассмотреть равенство a m · a n = a m + n , где m и n – натуральные числа, то оно будет верно для любых значений a , как положительных, так и отрицательных, а также для a = 0 .

    Применять свойства степеней без ограничений можно в тех случаях, когда основания степеней положительные или содержат переменные, область допустимых значений которых такова, что на ней основания принимают лишь положительные значения. Фактически, в рамках школьной программы по математике задачей учащегося является выбор подходящего свойства и правильное его применение.

    При подготовке к поступлению в Вузы могут встречаться задачи, в которых неаккуратное применение свойств будет приводить к сужению ОДЗ и другим сложностям с решением. В данном разделе мы разберем всего два таких случая. Больше информации по вопросу можно найти в теме «Преобразование выражений с использованием свойств степеней».

    Представьте выражение a 2 , 5 · ( a 2 ) − 3 : a − 5 , 5 в виде степени с основанием a .

    Решение

    Для начала используем свойство возведения в степень и преобразуем по нему второй множитель ( a 2 ) − 3 . Затем используем свойства умножения и деления степеней с одинаковым основанием:

    a 2 , 5 · a − 6 : a − 5 , 5 = a 2 , 5 − 6 : a − 5 , 5 = a − 3 , 5 : a − 5 , 5 = a − 3 , 5 − ( − 5 , 5 ) = a 2 .

    Ответ: a 2 , 5 · ( a 2 ) − 3 : a − 5 , 5 = a 2 .

    Преобразование степенных выражений согласно свойству степеней может производиться как слева направо, так и в обратном направлении.

    Найти значение степенного выражения 3 1 3 · 7 1 3 · 21 2 3 .

    Решение

    Если мы применим равенство ( a · b ) r = a r · b r , справа налево, то получим произведение вида 3 · 7 1 3 · 21 2 3 и дальше 21 1 3 · 21 2 3 . Сложим показатели при умножении степеней с одинаковыми основаниями: 21 1 3 · 21 2 3 = 21 1 3 + 2 3 = 21 1 = 21 .

    Есть еще один способ провести преобразования:

    3 1 3 · 7 1 3 · 21 2 3 = 3 1 3 · 7 1 3 · ( 3 · 7 ) 2 3 = 3 1 3 · 7 1 3 · 3 2 3 · 7 2 3 = = 3 1 3 · 3 2 3 · 7 1 3 · 7 2 3 = 3 1 3 + 2 3 · 7 1 3 + 2 3 = 3 1 · 7 1 = 21

    Ответ: 3 1 3 · 7 1 3 · 21 2 3 = 3 1 · 7 1 = 21

    Дано степенное выражение a 1 , 5 − a 0 , 5 − 6 , введите новую переменную t = a 0 , 5 .

    Решение

    Представим степень a 1 , 5 как a 0 , 5 · 3 . Используем свойство степени в степени ( a r ) s = a r · s справа налево и получим ( a 0 , 5 ) 3 : a 1 , 5 − a 0 , 5 − 6 = ( a 0 , 5 ) 3 − a 0 , 5 − 6 . В полученное выражение можно без проблем вводить новую переменную t = a 0 , 5 : получаем t 3 − t − 6 .

    Ответ: t 3 − t − 6 .

    Преобразование дробей, содержащих степени

    Обычно мы имеем дело с двумя вариантами степенных выражений с дробями: выражение представляет собой дробь со степенью или содержит такую дробь. К таким выражениям применимы все основные преобразования дробей без ограничений. Их можно сокращать, приводить к новому знаменателю, работать отдельно с числителем и знаменателем. Проиллюстрируем это примерами.

    Упростить степенное выражение 3 · 5 2 3 · 5 1 3 – 5 – 2 3 1 + 2 · x 2 – 3 – 3 · x 2 .

    Решение

    Мы имеем дело с дробью, поэтому проведем преобразования и в числителе, и в знаменателе:

    3 · 5 2 3 · 5 1 3 – 5 – 2 3 1 + 2 · x 2 – 3 – 3 · x 2 = 3 · 5 2 3 · 5 1 3 – 3 · 5 2 3 · 5 – 2 3 – 2 – x 2 = = 3 · 5 2 3 + 1 3 – 3 · 5 2 3 + – 2 3 – 2 – x 2 = 3 · 5 1 – 3 · 5 0 – 2 – x 2

    Поместим минус перед дробью для того, чтобы изменить знак знаменателя: 12 – 2 – x 2 = – 12 2 + x 2

    Ответ: 3 · 5 2 3 · 5 1 3 – 5 – 2 3 1 + 2 · x 2 – 3 – 3 · x 2 = – 12 2 + x 2

    Дроби, содержащие степени, приводятся к новому знаменателю точно также, как и рациональные дроби. Для этого необходимо найти дополнительный множитель и умножить на него числитель и знаменатель дроби. Подбирать дополнительный множитель необходимо таким образом, чтобы он не обращался в нуль ни при каких значениях переменных из ОДЗ переменных для исходного выражения.

    Приведите дроби к новому знаменателю: а) a + 1 a 0 , 7 к знаменателю a , б) 1 x 2 3 – 2 · x 1 3 · y 1 6 + 4 · y 1 3 к знаменателю x + 8 · y 1 2 .

    Решение

    а) Подберем множитель, который позволит нам произвести приведение к новому знаменателю. a 0 , 7 · a 0 , 3 = a 0 , 7 + 0 , 3 = a , следовательно, в качестве дополнительного множителя мы возьмем a 0 , 3 . Область допустимых значений переменной а включает множество всех положительных действительных чисел. В этой области степень a 0 , 3 не обращается в нуль.

    Выполним умножение числителя и знаменателя дроби на a 0 , 3 :

    a + 1 a 0 , 7 = a + 1 · a 0 , 3 a 0 , 7 · a 0 , 3 = a + 1 · a 0 , 3 a

    б) Обратим внимание на знаменатель:

    x 2 3 – 2 · x 1 3 · y 1 6 + 4 · y 1 3 = = x 1 3 2 – x 1 3 · 2 · y 1 6 + 2 · y 1 6 2

    Умножим это выражение на x 1 3 + 2 · y 1 6 , получим сумму кубов x 1 3 и 2 · y 1 6 , т. е. x + 8 · y 1 2 . Это наш новый знаменатель, к которому нам надо привести исходную дробь.

    Так мы нашли дополнительный множитель x 1 3 + 2 · y 1 6 . На области допустимых значений переменных x и y выражение x 1 3 + 2 · y 1 6 не обращается в нуль, поэтому, мы можем умножить на него числитель и знаменатель дроби:
    1 x 2 3 – 2 · x 1 3 · y 1 6 + 4 · y 1 3 = = x 1 3 + 2 · y 1 6 x 1 3 + 2 · y 1 6 x 2 3 – 2 · x 1 3 · y 1 6 + 4 · y 1 3 = = x 1 3 + 2 · y 1 6 x 1 3 3 + 2 · y 1 6 3 = x 1 3 + 2 · y 1 6 x + 8 · y 1 2

    Ответ: а) a + 1 a 0 , 7 = a + 1 · a 0 , 3 a , б) 1 x 2 3 – 2 · x 1 3 · y 1 6 + 4 · y 1 3 = x 1 3 + 2 · y 1 6 x + 8 · y 1 2 .

    Сократите дробь: а) 30 · x 3 · ( x 0 , 5 + 1 ) · x + 2 · x 1 1 3 – 5 3 45 · x 0 , 5 + 1 2 · x + 2 · x 1 1 3 – 5 3 , б) a 1 4 – b 1 4 a 1 2 – b 1 2 .

    Решение

    а) Используем наибольший общий знаменатель (НОД), на который можно сократить числитель и знаменатель. Для чисел 30 и 45 это 15 . Также мы можем произвести сокращение на x 0 , 5 + 1 и на x + 2 · x 1 1 3 – 5 3 .

    30 · x 3 · ( x 0 , 5 + 1 ) · x + 2 · x 1 1 3 – 5 3 45 · x 0 , 5 + 1 2 · x + 2 · x 1 1 3 – 5 3 = 2 · x 3 3 · ( x 0 , 5 + 1 )

    б) Здесь наличие одинаковых множителей неочевидно. Придется выполнить некоторые преобразования для того, чтобы получить одинаковые множители в числителе и знаменателе. Для этого разложим знаменатель, используя формулу разности квадратов:

    a 1 4 – b 1 4 a 1 2 – b 1 2 = a 1 4 – b 1 4 a 1 4 2 – b 1 2 2 = = a 1 4 – b 1 4 a 1 4 + b 1 4 · a 1 4 – b 1 4 = 1 a 1 4 + b 1 4

    Ответ: а) 30 · x 3 · ( x 0 , 5 + 1 ) · x + 2 · x 1 1 3 – 5 3 45 · x 0 , 5 + 1 2 · x + 2 · x 1 1 3 – 5 3 = 2 · x 3 3 · ( x 0 , 5 + 1 ) , б) a 1 4 – b 1 4 a 1 2 – b 1 2 = 1 a 1 4 + b 1 4 .

    К числу основных действий с дробями относится приведение к новому знаменателю и сокращение дробей. Оба действия выполняют с соблюдением ряда правил. При сложении и вычитании дробей сначала дроби приводятся к общему знаменателю, после чего проводятся действия (сложение или вычитание) с числителями. Знаменатель остается прежним. Результатом наших действий является новая дробь, числитель которой является произведением числителей, а знаменатель есть произведение знаменателей.

    Выполните действия x 1 2 + 1 x 1 2 – 1 – x 1 2 – 1 x 1 2 + 1 · 1 x 1 2 .

    Решение

    Начнем с вычитания дробей, которые располагаются в скобках. Приведем их к общему знаменателю:

    x 1 2 – 1 · x 1 2 + 1

    x 1 2 + 1 x 1 2 – 1 – x 1 2 – 1 x 1 2 + 1 · 1 x 1 2 = = x 1 2 + 1 · x 1 2 + 1 x 1 2 – 1 · x 1 2 + 1 – x 1 2 – 1 · x 1 2 – 1 x 1 2 + 1 · x 1 2 – 1 · 1 x 1 2 = = x 1 2 + 1 2 – x 1 2 – 1 2 x 1 2 – 1 · x 1 2 + 1 · 1 x 1 2 = = x 1 2 2 + 2 · x 1 2 + 1 – x 1 2 2 – 2 · x 1 2 + 1 x 1 2 – 1 · x 1 2 + 1 · 1 x 1 2 = = 4 · x 1 2 x 1 2 – 1 · x 1 2 + 1 · 1 x 1 2

    Теперь умножаем дроби:

    4 · x 1 2 x 1 2 – 1 · x 1 2 + 1 · 1 x 1 2 = = 4 · x 1 2 x 1 2 – 1 · x 1 2 + 1 · x 1 2

    Произведем сокращение на степень x 1 2 , получим 4 x 1 2 – 1 · x 1 2 + 1 .

    Дополнительно можно упростить степенное выражение в знаменателе, используя формулу разности квадратов: квадратов: 4 x 1 2 – 1 · x 1 2 + 1 = 4 x 1 2 2 – 1 2 = 4 x – 1 .

    Ответ: x 1 2 + 1 x 1 2 – 1 – x 1 2 – 1 x 1 2 + 1 · 1 x 1 2 = 4 x – 1

    Упростите степенное выражение x 3 4 · x 2 , 7 + 1 2 x – 5 8 · x 2 , 7 + 1 3 .
    Решение

    Мы можем произвести сокращение дроби на ( x 2 , 7 + 1 ) 2 . Получаем дробь x 3 4 x – 5 8 · x 2 , 7 + 1 .

    Продолжим преобразования степеней икса x 3 4 x – 5 8 · 1 x 2 , 7 + 1 . Теперь можно использовать свойство деления степеней с одинаковыми основаниями: x 3 4 x – 5 8 · 1 x 2 , 7 + 1 = x 3 4 – – 5 8 · 1 x 2 , 7 + 1 = x 1 1 8 · 1 x 2 , 7 + 1 .

    Переходим от последнего произведения к дроби x 1 3 8 x 2 , 7 + 1 .

    Ответ: x 3 4 · x 2 , 7 + 1 2 x – 5 8 · x 2 , 7 + 1 3 = x 1 3 8 x 2 , 7 + 1 .

    Множители с отрицательными показателями степени в большинстве случаев удобнее переносить из числителя в знаменатель и обратно, изменяя знак показателя. Это действие позволяет упростить дальнейшее решение. Приведем пример: степенное выражение ( x + 1 ) – 0 , 2 3 · x – 1 можно заменить на x 3 · ( x + 1 ) 0 , 2 .

    Преобразование выражений с корнями и степенями

    В задачах встречаются степенные выражения, которые содержат не только степени с дробными показателями, но и корни. Такие выражения желательно привести только к корням или только к степеням. Переход к степеням предпочтительнее, так как с ними проще работать. Такой переход является особенно предпочтительным, когда ОДЗ переменных для исходного выражения позволяет заменить корни степенями без необходимости обращаться к модулю или разбивать ОДЗ на несколько промежутков.

    Представьте выражение x 1 9 · x · x 3 6 в виде степени.

    Решение

    Область допустимых значений переменной x определяется двумя неравенствами x ≥ 0 и x · x 3 ≥ 0 , которые задают множество [ 0 , + ∞ ) .

    На этом множестве мы имеем право перейти от корней к степеням:

    x 1 9 · x · x 3 6 = x 1 9 · x · x 1 3 1 6

    Используя свойства степеней, упростим полученное степенное выражение.

    x 1 9 · x · x 1 3 1 6 = x 1 9 · x 1 6 · x 1 3 1 6 = x 1 9 · x 1 6 · x 1 · 1 3 · 6 = = x 1 9 · x 1 6 · x 1 18 = x 1 9 + 1 6 + 1 18 = x 1 3

    Ответ: x 1 9 · x · x 3 6 = x 1 3 .

    Преобразование степеней с переменными в показателе

    Данные преобразования достаточно просто произвести, если грамотно использовать свойства степени. Например, 5 2 · x + 1 − 3 · 5 x · 7 x − 14 · 7 2 · x − 1 = 0 .

    Мы можем заменить произведением степени, в показателях которых находится сумма некоторой переменной и числа. В левой части это можно проделать с первым и последним слагаемыми левой части выражения:

    5 2 · x · 5 1 − 3 · 5 x · 7 x − 14 · 7 2 · x · 7 − 1 = 0 , 5 · 5 2 · x − 3 · 5 x · 7 x − 2 · 7 2 · x = 0 .

    Теперь поделим обе части равенства на 7 2 · x . Это выражение на ОДЗ переменной x принимает только положительные значения:

    5 · 5 – 3 · 5 x · 7 x – 2 · 7 2 · x 7 2 · x = 0 7 2 · x , 5 · 5 2 · x 7 2 · x – 3 · 5 x · 7 x 7 2 · x – 2 · 7 2 · x 7 2 · x = 0 , 5 · 5 2 · x 7 2 · x – 3 · 5 x · 7 x 7 x · 7 x – 2 · 7 2 · x 7 2 · x = 0

    Сократим дроби со степенями, получим: 5 · 5 2 · x 7 2 · x – 3 · 5 x 7 x – 2 = 0 .

    Наконец, отношение степеней с одинаковыми показателями заменяется степенями отношений, что приводит к уравнению 5 · 5 7 2 · x – 3 · 5 7 x – 2 = 0 , которое равносильно 5 · 5 7 x 2 – 3 · 5 7 x – 2 = 0 .

    Введем новую переменную t = 5 7 x , что сводит решение исходного показательного уравнения к решению квадратного уравнения 5 · t 2 − 3 · t − 2 = 0 .

    Преобразование выражений со степенями и логарифмами

    Выражения, содержащие с записи степени и логарифмы, также встречаются в задачах. Примером таких выражений могут служить: 1 4 1 – 5 · log 2 3 или log 3 27 9 + 5 ( 1 – log 3 5 ) · log 5 3 . Преобразование подобных выражений проводится с использованием разобранных выше подходов и свойств логарифмов, которые мы подробно разобрали в теме «Преобразование логарифмических выражений».

    Напоминаем, что в данном уроке разбираются свойства степеней с натуральными показателями и нулём. Степени с рациональными показателями и их свойства будут рассмотрены в уроках для 8 классов.

    Степень с натуральным показателем обладает несколькими важными свойствами, которые позволяют упрощать вычисления в примерах со степенями.

    Свойство № 1

    Произведение степеней

    При умножении степеней с одинаковыми основаниями основание остаётся без изменений, а показатели степеней складываются.

    a m · a n = a m + n , где « a » — любое число, а « m », « n » — любые натуральные числа.

    Данное свойство степеней также действует на произведение трёх и более степеней.

    • Упростить выражение.
      b · b 2 · b 3 · b 4 · b 5 = b 1 + 2 + 3 + 4 + 5 = b 15
    • Представить в виде степени.
      6 15 · 36 = 6 15 · 6 2 = 6 15 · 6 2 = 6 17
    • Представить в виде степени.
      (0,8) 3 · (0,8) 12 = (0,8) 3 + 12 = (0,8) 15

    Обратите внимание, что в указанном свойстве речь шла только об умножении степеней с одинаковыми основаниями . Оно не относится к их сложению.

    Нельзя заменять сумму (3 3 + 3 2 ) на 3 5 . Это понятно, если
    посчитать (3 3 + 3 2 ) = (27 + 9) = 36 , а 3 5 = 243

    Свойство № 2

    Частное степеней

    При делении степеней с одинаковыми основаниями основание остаётся без изменений, а из показателя степени делимого вычитают показатель степени делителя.

    m : a n =–>

    = a m − n , где « a » — любое число, не равное нулю, а « m », « n » — любые натуральные числа такие, что « m > n ».

    Ответ: t = 3 4 = 81

    Пользуясь свойствами № 1 и № 2, можно легко упрощать выражения и производить вычисления.

      Пример. Упростить выражение.
      4 5m + 6 · 4 m + 2 : 4 4m + 3 = 4 5m + 6 + m + 2 : 4 4m + 3 = 4 6m + 8 − 4m − 3 = 4 2m + 5

    Пример. Найти значение выражения, используя свойства степени.

    =

    =

    =

    =

    = 2 11 − 5 = 2 6 = 64

    Обратите внимание, что в свойстве 2 речь шла только о делении степеней с одинаковыми основаниями.

    Нельзя заменять разность (4 3 −4 2 ) на 4 1 . Это понятно, если посчитать (4 3 −4 2 ) = (64 − 16) = 48 , а 4 1 = 4

    Свойство № 3

    Возведение степени в степень

    При возведении степени в степень основание степени остаётся без изменения, а показатели степеней перемножаются.

    (a n ) m = a n · m , где « a » — любое число, а « m », « n » — любые натуральные числа.

    • Пример.
      (a 4 ) 6 = a 4 · 6 = a 24
    • Пример. Представить 3 20 в виде степени с основанием 3 2 .

    По свойству возведения степени в степень известно, что при возведении в степень показатели перемножаются, значит:

    Свойства 4

    Степень произведения

    При возведении в степень произведения каждый из множителей возводится в степень. Затем полученные результаты перемножаются.

    (a · b) n = a n · b n , где « a », « b » — любые рациональные числа; « n » — любое натуральное число.

    • Пример 1.
      (6 · a 2 · b 3 · c ) 2 = 6 2 · a 2 · 2 · b 3 · 2 · с 1 · 2 = 36 a 4 · b 6 · с 2
    • Пример 2.
      (−x 2 · y) 6 = ( (−1) 6 · x 2 · 6 · y 1 · 6 ) = x 12 · y 6

    Обратите внимание, что свойство № 4, как и другие свойства степеней, применяют и в обратном порядке.

    (a n · b n )= (a · b) n

    То есть, чтобы перемножить степени с одинаковыми показателями можно перемножить основания, а показатель степени оставить неизменным.

    • Пример. Вычислить.
      2 4 · 5 4 = (2 · 5) 4 = 10 4 = 10 000
    • Пример. Вычислить.
      0,5 16 · 2 16 = (0,5 · 2) 16 = 1

    В более сложных примерах могут встретиться случаи, когда умножение и деление надо выполнить над степенями с разными основаниями и разными показателями. В этом случае советуем поступать следующим образом.

    Например, 4 5 · 3 2 = 4 3 · 4 2 · 3 2 = 4 3 · (4 · 3) 2 = 64 · 12 2 = 64 · 144 = 9216

    Пример возведения в степень десятичной дроби.

    4 21 · (−0,25) 20 = 4 · 4 20 · (−0,25) 20 = 4 · (4 · (−0,25)) 20 = 4 · (−1) 20 = 4 · 1 = 4

    Свойства 5

    Степень частного (дроби)

    Чтобы возвести в степень частное, можно возвести в эту степень отдельно делимое и делитель, и первый результат разделить на второй.

    (a : b) n = a n : b n , где « a », « b » — любые рациональные числа, b ≠ 0, n — любое натуральное число.

    • Пример. Представить выражение в виде частного степеней.
      (5 : 3) 12 = 5 12 : 3 12

    Напоминаем, что частное можно представить в виде дроби. Поэтому на теме возведение дроби в степень мы остановимся более подробно на следующей странице.

    правила вычисления степеней с разными основаниями или натуральными показателями по математике и порядок этого

    Одной из главных характеристик в алгебре, да и во всей математике является степень. Конечно, в 21 веке все расчеты можно проводить на онлайн-калькуляторе, но лучше для развития мозгов научиться делать это самому.

    В данной статье рассмотрим самые важные вопросы, касающиеся этого определения. А именно, поймем что это вообще такое и каковы основные его функции, какие имеются свойства в математике.

    Степень, свойства и действия со степенями, сложение, умножение, деление отрицательных степеней, степень с натуральным показателем, правила и формулы

    Рассмотрим на примерах то, как выглядит расчет, каковы основные формулы. Разберем основные виды величины и то, чем они отличаются от других функций.

    Поймем, как решать с помощью этой величины различные задачи. Покажем на примерах, как возводить в нулевую степень, иррациональную, отрицательную и др.

    Что такое степень числа

    Что же подразумевают под выражением «возвести число в степень»?

    Степенью n числа а является произведение множителей величиной а n-раз подряд.

    Математически это выглядит следующим образом: an = a * a * a * …an.

    Причем, левая часть уравнения будет читаться, как a в степ. n.

    Например:

    • 23 = 2 в третьей степ. = 2 * 2 * 2 = 8,
    • 42 = 4 в степ. два = 4 * 4 = 16,
    • 54 = 5 в степ. четыре = 5 * 5 * 5 * 5 = 625,
    • 105 = 10 в 5 степ. = 10 * 10 * 10 * 10 * 10 = 100000,
    • 104 = 10 в 4 степ. = 10 * 10 * 10 * 10 = 10000.

    Ниже будет представлена таблица квадратов и кубов от 1 до 10.

    Таблица степеней от 1 до 10

    Ниже будут приведены результаты возведения натуральных чисел в положительные степени – «от 1 до 100».

    Ч-ло2-ая ст-нь3-я ст-нь
    111
    248
    3927
    41664
    525125
    636216
    749343
    864512
    981279
    101001000

    Свойства степеней

    Что же характерно для такой математической функции? Рассмотрим базовые свойства.

    Учеными установлено следующие признаки, характерные для всех степеней:

    • an * am = (a)(n+m),
    • an : am = (a)(n-m),
    • (ab ) m=(a)(b*m).

    Проверим на примерах:

    • 23 * 22 = 8 * 4 = 32. С другой стороны 25 = 2 * 2 * 2 * 2 * 2 =32.

    Аналогично:

    • 23 : 22 = 8 / 4 =2. Иначе 23-2 = 21 =2.
    • (23)2 = 82 = 64. А если по-другому? 26 = 2 * 2 * 2 * 2 * 2 * 2 = 32 * 2 = 64.

    Как видим, правила работают.

    А как же быть со сложением и вычитанием? Всё просто. Выполняется сначала возведение в степень, а уж потом сложение и вычитание.

    Посмотрим на примерах:

    • 33 + 24 = 27 + 16 = 43,
    • 52 – 32 = 25 – 9 = 16. Обратите внимание: правило не будет выполняться, если сначала произвести вычитание: (5 3)2 = 22 = 4.
    • А вот в этом случае надо вычислять сначала сложение, поскольку присутствуют действия в скобках: (5 + 3)3 = 83 = 512.

    Как производить вычисления в более сложных случаях? Порядок тот же:

    • при наличии скобок – начинать нужно с них,
    • затем возведение в степень,
    • потом выполнять действия умножения, деления,
    • после сложение, вычитание.

    Есть специфические свойства, характерные не для всех степеней:

    1. Корень n-ой степени из числа a в степени m запишется в виде: am/n.
    2. При возведении дроби в степень: этой процедуре подвержены как числитель, так и ее знаменатель.
    3. При возведении произведения разных чисел в степень, выражение будет соответствовать произведению этих чисел в заданной степени. То есть: (a * b)n = an * bn.
    4. При возведении числа в отрицательную степ., нужно разделить 1 на число в той же ст-ни, но со знаком «+».
    5. Если знаменатель дроби находится в отрицательной степени, то это выражение будет равно произведению числителя на знаменатель в положительной степени.
    6. Любое число в степени 0 = 1, а в степ. 1 = самому себе.

    Степень с отрицательным показателем

    Что делать при минусовой степени, т. е. когда показатель отрицательный?

    Исходя из свойств 4 и 5 (смотри пункт выше), получается:

    • A(-n) = 1 / An, 5(-2) = 1 / 52 = 1 / 25.

    И наоборот:

    • 1 / A(-n) = An, 1 / 2(-3) = 23 = 8.

    А если дробь?

    • (A / B)(-n) = (B / A)n, (3 / 5)(-2) = (5 / 3)2 = 25 / 9.

    Степень с натуральным показателем

    Под ней понимают степень с показателями, равными целым числам.

    Что нужно запомнить:

    • A0 = 1, 10 = 1, 20 = 1, 3.150 = 1, (-4)0 = 1… и т. д.
    • A1 = A, 11 = 1, 21 = 2, 31 = 3 … и т. д.

    Кроме того, если (-a)2n+2, n=0, 1, 2…то результат будет со знаком «+». Если отрицательное число возводится в нечетную степень, то наоборот. Общие свойства, да и все специфические признаки, описанные выше, также характерны для них.

    Дробная степень

    Этот вид можно записать схемой: Am/n. Читается как: корень n-ой степени из числа A в степени m.

    С дробным показателем можно делать, что угодно: сокращать, раскладывать на части, возводить в другую степень и т. д.

    Степень с иррациональным показателем

    Пусть α – иррациональное число, а А ˃ 0.

    Чтобы понять суть степени с таким показателем, рассмотрим разные возможные случаи:

    • А = 1. Результат будет равен 1. Поскольку существует аксиома – 1 во всех степенях равна единице,
    • А˃1.
    • Аr1 ˂ Аα ˂ Аr2, r1 ˂ r2 – рациональные числа.

    В этом случае наоборот: Аr2 ˂ Аα ˂ Аr1 при тех же условиях, что и во втором пункте.

    Например, показатель степени число π. Оно рациональное.

    1. r1 – в этом случае равно 3,
    2. r2 – будет равно 4.
    3. Тогда, при А = 1, 1π = 1.
    4. А = 2, то 23 ˂ 2π ˂ 24, 8 ˂ 2π ˂ 16.
    5. А = 1/2, то (½)4 ˂ (½)π ˂ (½)3, 1/16 ˂ (½)π ˂ 1/8.

    Для таких степеней характерны все математические операции и специфические свойства, описанные выше.

    Заключение

    Подведём итоги для чего же нужны эти величины, в чем преимущество таких функций? Конечно, в первую очередь они упрощают жизнь математиков и программистов при решении примеров, поскольку позволяют минимизировать расчеты, сократить алгоритмы, систематизировать данные и многое другое.

    Где еще могут пригодиться эти знания? В любой рабочей специальности: медицине, фармакологии, стоматологии, строительстве, технике, инженерии, конструировании и т. д.

    Источник: https://tvercult.ru/nauka/stepen-svoystva-pravila-deystviya-i-formulyi

    Степенные выражения (выражения со степенями) и их преобразование

    Рассмотрим тему преобразования выражений со степенями, но прежде остановимся на ряде преобразований, которые можно проводить с любыми выражениями, в том числе со степенными. Мы научимся раскрывать скобки, приводить подобные слагаемые, работать с основанием и показателем степени, использовать свойства степеней.

    Что представляют собой степенные выражения?

    В школьном курсе мало кто использует словосочетание «степенные выражения», зато этот термин постоянно встречается в сборниках для подготовки к ЕГЭ. В большинства случаев словосочетанием обозначаются выражения, которые содержат в своих записях степени. Это мы и отразим в нашем определении.

    Степенное выражение – это выражение, которое содержит степени.

    Приведем несколько примеров степенных выражений, начиная со степени с натуральным показателем и заканчивая степенью с действительным показателем.

    • Самыми простыми степенными выражениями можно считать степени числа с натуральным показателем: 32, 75+1, (2+1)5, (−0,1)4, 2233, 3·a2−a+a2, x3−1, (a2)3.
    • А также степени с нулевым показателем: 50, (a+1)0, 3+52−3,20. И степени с целыми отрицательными степенями: (0,5)2+(0,5)-22.
    • Чуть сложнее работать со степенью, имеющей рациональный  и иррациональный показатели: 26414-3·3·312, 23,5·2-22-1,5, 1a14·a12-2·a-16·b12, xπ·x1-π, 233+5.
    • В качестве показателя может выступать переменная 3x-54-7·3x-58 или логарифм x2·lgx−5·xlgx.

    С вопросом о том, что такое степенные выражения, мы разобрались. Теперь займемся их преобразованием.

    Основные виды преобразований степенных выражений

    В первую очередь мы рассмотрим основные тождественные преобразования выражений, которые можно выполнять со степенными выражениями.

    Вычислите значение степенного выражения 23·(42−12).

    Решение

    Все преобразования мы будем проводить с соблюдением порядка выполнения действий. В данном случае начнем мы с выполнения действий в скобках: заменим степень на цифровое значение и вычислим разность двух чисел. Имеем 23·(42−12)=23·(16−12)=23·4.

    Нам остается заменить степень 23 ее значением 8 и вычислить произведение 8·4=32. Вот наш ответ.

    Ответ: 23·(42−12)=32.

    Упростите выражение со степенями 3·a4·b−7−1+2·a4·b−7.

    Решение

    Данное нам в условии задачи выражение содержит подобные слагаемые, которые мы можем привести: 3·a4·b−7−1+2·a4·b−7=5·a4·b−7−1.

    Ответ: 3·a4·b−7−1+2·a4·b−7=5·a4·b−7−1.

    Источник: https://Zaochnik.com/spravochnik/matematika/vyrazhenija/stepennye-vyrazhenija/

    Возведение в степень

    Возведение в степень — это арифметическая операция повторяющегося умножения. Если требуется перемножить число n-ное количество раз, то достаточно возвести его в n-ную степень.

    Основные действия со степенями

    В первую очередь степень — это повторяющееся умножение. Число 134 — это 13 × 13 × 13 × 13, где перемножаются четыре одинаковых сомножителя. Если умножить 134 на 132, то мы получим (13 × 13 × 13 × 13) × (13 × 13), что логично превращается в 136.

    Это и есть первое правило возведения в степень, которое гласит: при умножении чисел, возведенных в степень, их показатели суммируются. Математически это записывается как:

    Если разделить 134 на 132, то нам потребуется вычислить дробь вида:

    • (13 × 13 × 13 × 13) / (13 × 13).

    Мы можем просто сократить числа в числителе и знаменателе, и в результате останется 13 × 13 = 132. Очевидно, деление чисел, возведенных в степень, соответствует вычитанию их показателей. Второе правило действий со степенями математически выглядит так: am / an = a(m – n).

    Теперь давайте возведем 114 в куб, то есть в третью степень. Для этого нам потребуется вычислить выражение (11 × 11 × 11 × 11) × (11 × 11 × 11 × 11) × (11 × 11 × 11 × 11). Получилось 12 сомножителей, следовательно, при возведении в n-ную степень числа в степени m, показатели перемножаются. Третье правило записывается так: (am)n = a(m × n).

    Это основные правила работы со степенными выражениями. Однако число можно возвести в отрицательную степень, дробную и нулевую. Какой результат даст выражение 150? Давайте воспользуемся вторым правилом действий степенями и попробуем разделить 154 на 154, что запишется как дробь: 154 / 154.

    Очевидно, что в числителе и знаменателе стоят одни и те же числа, а когда число делится само на себя, оно превращается в единицу. Но согласно правилу действий со степенными числами это будет эквивалентно 150.

    Следовательно: 154 / 154 = 150 = 1.

    Таким образом, четвертое правило гласит, что любое положительное число в нулевой степени равняется единице. Выглядит это правило так: a0 = 1.

    При помощи второго правила легко объяснить и работу с отрицательными степенями. К примеру, давайте разделим 82 на 84 и запишем выражение в виде дроби.

    (8 × 8) / (8 × 8 × 8 × 8).

    Мы можем сократить две восьмерки в числителе и знаменателе и преобразовать дробь в 1 / (8 × 8). Но согласно правилу в ответе мы должны получить 8-2. В знаменателе у нас как раз стоит восьмерка в квадрате. Таким образом:

    При этом для значения -1 правило трансформируется в элегантную формулу:

    И последнее правило, которое пригодится вам при работе со степенными функциями, гласит о дробных степенях. Что мы можем сделать с выражением 7(1/2). Очевидно, что возвести его в квадрат, и тогда по третьему правилу в результате у нас останется только семерка.

    Степень 1/2 — это извлечение квадратного корня, так как при возведении его в квадрат мы получаем целое число. Степень 1/3 соответствует извлечению кубического корня, но как быть с показателем 2/3? Логично, что это кубический корень из числа, возведенного в квадрат.

    Последнее правило гласит, что знаменатель дробного показателя означает извлечение корня, а числитель — возведение в степень. Математически это выглядит как: a(m/n) есть корень n-ной степени из am. Теперь вы знаете, как проводить любые арифметические операции со степенными выражениями.

    Вы можете использовать наш калькулятор для вычисления степенных функций. Программа позволяет определить основание, показатель и результат операции. Кроме того, калькулятор сопровождается иллюстрацией графика функций: параболы, кубической параболы и параболы в n-ной степени. Рассмотрим пару примеров.

    Примеры из реальной жизни

    Депозит в банке

    Если мы положим на банковский депозит $1 000 под годовую ставку в размере 9% годовых, то сколько денег на счету будет через 20 лет? Рост с течением времени рассчитываются по экспоненциальной формуле вида:

    Рост = a × e(kt),

    • где a – начальное значение,
    •  e – константа, равная 2,718;
    • k – коэффициент роста;
    • t – время.

    Для решения банковской задачи нам потребуется возвести 2,718 в степень, равную 20 × 0,09 = 1,8. Воспользуемся нашим калькулятором и введем в ячейку «Число, x =» значение 2,718, а в ячейку «Степень, n =» значение 1,8. Мы получим ответ, равный 6,049. Теперь, для подсчета суммы на банковском счету нам необходимо умножить начальное значение $1 000 на прирост в размере 6,049. В итоге, через 20 лет на депозите будет $6 049.

    Школьная задача

    Пусть в школьной задаче требуется построить график функции y = x2,5. Это алгебраическая задача, для решения которой требуется задаться тремя значениями «x» и вычислить соответствующие ему значения «y». После чего по найденным точкам построить график функции.

    Введите в ячейку «Степень, n =» значение 2,5. После этого последовательно рассчитайте значения «y», вводя в «Число, x =» аргументы 1, 2, 3. Вы получите соответствующие значения функции 1; 5,657; 15,588. Вам останется только нарисовать кривую по найденным точкам.

    Источник: https://BBF.ru/calculators/73/

    выражение со степенями

    Вы искали выражение со степенями? На нашем сайте вы можете получить ответ на любой математический вопрос здесь. Подробное
    решение с описанием и пояснениями поможет вам разобраться даже с самой сложной задачей и выражения со степенями, не
    исключение. Мы поможем вам подготовиться к домашним работам, контрольным, олимпиадам, а так же к поступлению
    в вуз.
    И какой бы пример, какой бы запрос по математике вы не ввели — у нас уже есть решение.
    Например, «выражение со степенями».

    Применение различных математических задач, калькуляторов, уравнений и функций широко распространено в нашей
    жизни. Они используются во многих расчетах, строительстве сооружений и даже спорте. Математику человек
    использовал еще в древности и с тех пор их применение только возрастает. Однако сейчас наука не стоит на
    месте и мы можем наслаждаться плодами ее деятельности, такими, например, как онлайн-калькулятор, который
    может решить задачи, такие, как выражение со степенями,выражения со степенями,выражения со степенями примеры решения,вычисление со степенями,как найти значение выражения со степенями,как решать выражения со степенями,как решать с степенями примеры,как решать со степенью примеры,как решать со степенями,как решать со степенями примеры,как решать степенные выражения,как решать степень,как решить выражение со степенями,как решить пример со степенями,как упростить выражение со степенями,найдите значение выражения со степенями,найти значение выражения с дробями и степенями,найти значение выражения со степенями,найти значения выражения со степенями,пример решить со степенями,примеров со степенью решение,примеры с степенями как решать,примеры со степенями 7 класс с решениями,примеры со степенями 9 класс с решениями,примеры со степенями как решать,решение выражений со степенями,решение дробей с степенями,решение дробей со степенями,решение примеров с дробными степенями,решение примеров с степенями,решение примеров со степенью,решение примеров со степенями,решение примеров со степенями 7 класс,решение с степенями примеров,решение со степенями,решение степеней,решить пример со степенями,сложение дробей со степенями,сократить выражение со степенями,степенные выражения как решать,степень как решать,упростите выражение 7 класс алгебра примеры со степенями,упростите выражение 7 класс алгебра со степенями примеры,упростите выражение с дробями и степенями,упростите выражение со степенями,упростить выражение со степенями,упрощение выражений со степенями,упрощение дробей со степенями. На этой странице вы найдёте калькулятор,
    который поможет решить любой вопрос, в том числе и выражение со степенями. Просто введите задачу в окошко и нажмите
    «решить» здесь (например, выражения со степенями примеры решения).

    Где можно решить любую задачу по математике, а так же выражение со степенями Онлайн?

    Решить задачу выражение со степенями вы можете на нашем сайте https://pocketteacher.ru. Бесплатный
    онлайн решатель позволит решить онлайн задачу любой сложности за считанные секунды. Все, что вам необходимо
    сделать — это просто
    ввести свои данные в решателе. Так же вы можете посмотреть видео инструкцию и узнать, как правильно ввести
    вашу задачу на нашем сайте. А если у вас остались вопросы, то вы можете задать их в чате снизу слева на странице
    калькулятора.

    Подайте в виде степени выражение. Степенные выражения (выражения со степенями) и их преобразование

    Десятичная дробь используется, когда нужно выполнять действия с нецелыми числами. Это может показаться нерациональным. Но такой вид чисел существенно облегчает математические операции, которые с ними необходимо выполнять. Это понимание приходит со временем, когда их запись становится привычной, а прочтение не вызывает трудностей, и освоены правила десятичных дробей. Тем более что все действия повторяют уже известные, которые усвоены с натуральными числами. Только нужно запомнить некоторые особенности.

    Определение десятичной дроби

    Десятичная дробь — это особое представление нецелого числа со знаменателем, который делится на 10, а ответ получается в виде единицы и, возможно, нулей. Другими словами, если в знаменателе 10, 100, 1000 и так далее, то удобнее переписать число с использованием запятой. Тогда до нее будет расположена целая часть, а потом — дробная. Причем запись второй половины числа будет зависеть от знаменателя. Количество цифр, которые находятся в дробной части, должно быть равно разряду знаменателя.

    Проиллюстрировать вышесказанное можно этими числами:

    9/10=0,9; 178/10000=0,0178; 3,05; 56 003,7006.

    Причины, по которым понадобилось применение десятичных дробей

    Математикам потребовались десятичные дроби по нескольким основаниям:

      Упрощение записи. Такая дробь расположена вдоль одной линии без черточки между знаменателем и числителем, при этом наглядность не страдает.

      Простота в сравнении. Достаточно просто соотнести цифры, находящиеся в одинаковых позициях, в то время как с обыкновенными дробями пришлось бы приводить их к общему знаменателю.

      Упрощение вычислений.

      Калькуляторы не рассчитаны на введение обыкновенных дробей, они для всех операций используют десятичную запись чисел.

    Как правильно прочитать такие числа?

    Ответ прост: так же, как обыкновенное смешанное число со знаменателем, кратным 10. Исключение составляют только дроби без целого значения, тогда при чтении нужно произносить «ноль целых».

    Например, 45/1000 нужно произнести как сорок пять тысячных
    , в то же время 0,045 будет звучать как ноль целых сорок пять тысячных
    .

    Смешанное число с целой частью равной 7 и дробью 17/100, что запишется как 7,17, в обоих случаях будет прочитано как семь целых семнадцать сотых
    .

    Роль разрядов в записи дробей

    Верно отметить разряд — это то, что требует математика. Десятичные дроби и их значение могут существенно измениться, если записать цифру не в том месте. Впрочем, это было справедливо и раньше.

    Для прочтения разрядов целой части десятичной дроби нужно просто воспользоваться правилами, известными для натуральных чисел. А в правой части они зеркально отражаются и по-другому читаются. Если в целой части звучало «десятки», то после запятой это будут уже «десятые».

    Наглядно это можно увидеть в этой таблице.

    Таблица разрядов десятичной дроби

    класстысячиединицы,дробная часть
    разрядсот.дес.ед.сот.дес.ед.десятаясотаятысячнаядесятитысячная

    Как правильно записать смешанное число десятичной дробью?

    Если в знаменателе стоит число, равное 10 или 100, и прочие, то вопрос о том, как дробь перевести в десятичную, несложен. Для этого достаточно по-другому переписать все ее составные части. В этом помогут такие пункты:

      немного в стороне написать числитель дроби, в этот момент десятичная запятая располагается справа, после последней цифры;

      переместить запятую влево, здесь самое главное — правильно сосчитать цифры — передвинуть ее нужно на столько позиций, сколько нолей в знаменателе;

      если их не хватает, то на пустых позициях должны оказаться нули;

      нули, которые были в конце числителя, теперь не нужны, и их можно зачеркнуть;

      перед запятой приписать целую часть, если ее не было, то здесь тоже окажется нуль.

    Внимание. Нельзя зачеркивать нули, которые оказались окружены другими цифрами.

    О том, как быть в ситуации, когда в знаменателе число не только из единицы и нулей, как дробь переводить в десятичную, можно прочитать чуть ниже. Это важная информация, с которой обязательно стоит ознакомиться.

    Как дробь перевести в десятичную, если знаменатель — произвольное число?

    Здесь возможны два варианта:

      Когда знаменатель можно представить в виде числа, которое равно десяти в любой степени.

      Если такую операцию проделать нельзя.

    Как это проверить? Нужно разложить знаменатель на множители. Если в произведении присутствуют только 2 и 5, то все хорошо, и дробь легко преобразуется в конечную десятичную. В противном случае, если появляются 3, 7 и другие простые числа, то результат будет бесконечным. Такую десятичную дробь для удобства использования в математических операциях принято округлять. Об этом будет речь немного ниже.

    Изучает, как получаются такие десятичные дроби, 5 класс. Примеры здесь будут очень кстати.

    Пусть в знаменателях находятся числа: 40, 24 и 75. Разложение на простые множители для них будет такое:

    • 40=2·2·2·5;
    • 24=2·2·2·3;
    • 75=5·5·3.

    В этих примерах только первая дробь может быть представлена в виде конечной.

    Алгоритм перевода обыкновенной дроби в конечную десятичную

      Проверить разложение знаменателя на простые множители и убедиться в том, что оно будет состоять из 2 и 5.

      Добавить к этим числам столько 2 и 5, чтобы их стало равное количество. Они дадут значение дополнительного множителя.

      Произвести умножение знаменателя и числителя на это число. В результате получится обыкновенная дробь, под чертой у которой стоит 10 в некоторой степени.

    Если в задаче эти действия выполняются со смешанным числом, то его сначала нужно представить в виде неправильной дроби. А уже потом действовать по описанному сценарию.

    Представление обыкновенной дроби в виде округленной десятичной

    Этот способ того, как дробь переводить в десятичную, кому-то покажется даже проще. Потому что в нем нет большого количества действий. Нужно только разделить значение числителя на знаменатель.

    К любому числу с десятичной частью справа от запятой можно приписать бесконечное количество нулей. Этим свойством и нужно воспользоваться.

    Сначала записать целую часть и поставить после нее запятую. Если дробь правильная, то написать ноль.

    Потом полагается выполнить деление числителя на знаменатель. Так, чтобы количество цифр у них было одинаковым. То есть приписать справа у числителя нужное количество нолей.

    Выполнять деление в столбик до тех пор, пока не будет набрано нужное количество цифр. Например, если округлить нужно будет до сотых, то в ответе их должно быть 3. В общем, цифр должно быть на одну больше, чем нужно получить в итоге.

    Записать промежуточный ответ после запятой и округлить по правилам. Если последняя цифра — от 0 до 4, то ее нужно просто отбросить. А когда она равна 5-9, то стоящую перед ней нужно увеличить на единицу, отбросив последнюю.

    Возврат от десятичной дроби к обыкновенной

    В математике встречаются задачи, когда десятичные дроби удобнее представить в виде обыкновенных, в которых есть числитель со знаменателем. Можно вздохнуть с облегчением: эта операция возможна всегда.

    Для этой процедуры нужно сделать следующее:

      записать целую часть, если она равна нулю, то ничего писать не надо;

      провести дробную черту;

      над ней записать цифры из правой части, если первыми идут нули, то их нужно зачеркнуть;

      под чертой написать единицу с таким количеством нолей, сколько цифр стоит после запятой в первоначальной дроби.

      Это все, что нужно сделать, чтобы перевести десятичную дробь в обыкновенную.

      Что можно делать с десятичными дробями?

      В математике это будут определенные действия с десятичными дробями, которые ранее выполнялись для других чисел.

      Ими являются:

        сравнение;

        сложение и вычитание;

        умножение и деление.

      Первое действие, сравнение, похоже на то, как это делалось для натуральных чисел. Чтобы определить, какое больше, нужно сравнивать разряды целой части. Если они окажутся равными, то переходят к дробной и так же по разрядам сравнивают их. То число, где окажется большая цифра в старшем разряде, и будет ответом.

      Сложение и вычитание десятичных дробей

      Это, пожалуй, самые простые действия. Потому что выполняются по правилам для натуральных чисел.

      Так, чтобы выполнить сложение десятичных дробей, их нужно записать друг под другом, разместив запятые в столбик. При такой записи слева от запятых оказываются целые части, а справа — дробные. И теперь нужно сложить цифры поразрядно, как это делается с натуральными числами, снеся вниз запятую. Начинать сложение нужно с самого маленького разряда дробной части числа. Если в правой половине не хватает цифр, то дописывают нули.

      При вычитании действуют так же. И здесь действует правило, которое описывает возможность занять единицу у старшего разряда. Если в уменьшаемой дроби после запятой меньше цифр, чем у вычитаемого, то в ней просто приписывают нули.

      Немного сложнее обстоит дело с заданиями, где нужно выполнить умножение и деление десятичных дробей.

      Как умножить десятичную дробь в разных примерах?

      Правило, по которому производится умножение десятичных дробей на натуральное число, такое:

        записать их в столбик, не обращая внимания на запятую;

        перемножить, как если бы они были натуральными;

        отделить запятой столько цифр, сколько их было в дробной части исходного числа.

      Частным случаем является пример, в котором натуральное число равно 10 в любой степени. Тогда для получения ответа нужно просто передвинуть запятую вправо на столько позиций, сколько нулей в другом множителе. Иными словами, при умножении на 10 запятая сдвигается на одну цифру, на 100 — их будет уже две, и так далее. Если цифр в дробной части не хватает, то нужно записать на пустых позициях нули.

      Правило, которым пользуются, когда в задании нужно произвести умножение десятичных дробей на другое такое же число:

        записать их друг под другом, не обращая внимания на запятые;

        умножить, как если бы они были натуральными;

        отделить запятой столько цифр, сколько их было в дробных частях обеих исходных дробях вместе.

      Частным случаем выделяются примеры, в которых один из множителей равен 0,1 или 0,01 и далее. В них нужно выполнить перемещение запятой влево на количество цифр в представленных множителях. То есть если умножается на 0,1, то запятая сдвигается на одну позицию.

      Как разделить десятичную дробь в разных заданиях?

      Деление десятичных дробей на натуральное число выполняется по такому правилу:

        записать их для деления в столбик, как если бы они были натуральными;

        делить по привычному правилу до тех пор, пока не закончится целая часть;

        поставить в ответ запятую;

        продолжить деление дробной составляющей до получения в остатке нуля;

        если нужно, то можно приписать нужное количество нулей.

      Если целая часть равна нулю, то и в ответе ее тоже не будет.

      Отдельно стоит деление на числа, равные десятке, сотне и так далее. В таких задачах нужно передвинуть запятую влево на количество нулей в делителе. Бывает, что цифр в целой части не хватает, тогда вместо них используют нули. Можно заметить, что эта операция подобна умножению на 0,1 и подобным ей числам.

      Чтобы выполнить деление десятичных дробей, нужно воспользоваться этим правилом:

        превратить делитель в натуральное число, а для этого перенести в нем запятую вправо до конца;

        выполнить перемещение запятой и в делимом на такое же число цифр;

        действовать по предыдущему сценарию.

      Выделяется деление на 0,1; 0,01 и прочие подобные числа. В таких примерах запятая сдвигается вправо на число цифр в дробной части. Если они закончились, то нужно приписать недостающее количество нулей. Стоит отметить, что это действие повторяет деление на 10 и подобные ему числа.

      Заключение: все дело в практике

      Ничто в учебе не дается легко и без усилий. Для надежного освоения нового материала требуются время и тренировка. Математика не исключение.

      Чтобы тема про десятичные дроби не вызывала затруднений, нужно решать с ними примеров как можно больше. Ведь было время, когда и сложение натуральных чисел ставило в тупик. А теперь все нормально.

      Поэтому, перефразируя известную фразу: решать, решать и еще раз решать. Тогда и задания с такими числами будут выполняться легко и непринужденно, как очередная головоломка.

      Кстати, и головоломки поначалу решаются сложно, а потом нужно делать привычные движения. Так же и в математических примерах: пройдя по одному пути несколько раз, потом уже не будешь задумываться над тем, куда повернуть.

    В этом уроке мы рассмотрим каждую из этих операций по отдельности.

    Содержание урока


    Сложение десятичных дробей

    Как мы знаем, десятичная дробь имеет целую и дробную часть. При сложении десятичных дробей, целые и дробные части складываются по отдельности.

    Например, сложим десятичные дроби 3,2 и 5,3. Десятичные дроби удобнее складывать в столбик.

    Запишем сначала эти две дроби в столбик, при этом целые части обязательно должны быть под целыми, а дробные под дробными. В школе это требование называют «запятая под запятой»
    .

    Запишем дроби в столбик так, чтобы запятая оказалась под запятой:

    Начинаем складывать дробные части: 2 + 3= 5. Записываем пятёрку в дробной части нашего ответа:

    Теперь складываем целые части: 3 + 5 = 8. Записываем восьмёрку в целой части нашего ответа:

    Теперь отделяем запятой целую часть от дробной. Для этого опять же соблюдаем правило «запятая под запятой»
    :

    Получили ответ 8,5. Значит выражения 3,2 + 5,3 равно 8,5

    На самом деле, не всё так просто, как кажется на первый взгляд. Здесь тоже имеются свои подводные камни, о которых мы сейчас поговорим.

    Разряды в десятичных дробях

    У десятичных дробей, как и у обычных чисел, есть свои разряды. Это разряды десятых, разряды сотых, разряды тысячных. При этом разряды начинаются после запятой.

    Первая цифра после запятой отвечает за разряд десятых, вторая цифра после запятой за разряд сотых, третья цифра после запятой за разряд тысячных.

    Разряды в десятичных дробях хранят в себе некоторую полезную информацию. В частности, они сообщают сколько в десятичной дроби десятых частей, сотых частей и тысячных частей.

    Например, рассмотрим десятичную дробь 0,345

    Позиция, где находится тройка, называется разрядом десятых

    Позиция, где находится четвёрка, называется разрядом сотых

    Позиция, где находится пятёрка, называется разрядом тысячных

    Посмотрим на данный рисунок. Видим, что в разряде десятых располагается тройка. Это говорит о том, что в десятичной дроби 0,345 содержится три десятых .

    Если мы сложим дроби , и то получим изначальную десятичную дробь 0,345

    Видно, что сначала мы получили ответ , но перевели его в десятичную дробь и получили 0,345.

    При сложении десятичных дробей соблюдаются те же принципы и правила, что и при сложении обычных чисел. Сложение десятичных дробей происходит по разрядам: десятые части складываются с десятыми частями, сотые с сотыми, тысячные с тысячными.

    Поэтому при сложении десятичных дробей требуют соблюдать правило «запятая под запятой»
    . Запятая под запятой обеспечивает тот самый порядок, в котором десятые части складываются с десятыми, сотые с сотыми, тысячные с тысячными.

    Пример 1.
    Найти значение выражения 1,5 + 3,4

    В первую очередь складываем дробные части 5 + 4 = 9. Записываем девятку в дробной части нашего ответа:

    Теперь складываем целые части 1 + 3 = 4. Записываем четвёрку в целой части нашего ответа:

    Теперь отделяем запятой целую часть от дробной. Для этого опять же соблюдаем правило «запятая под запятой»:

    Получили ответ 4,9. Значит значение выражения 1,5 + 3,4 равно 4,9

    Пример 2.
    Найти значение выражения: 3,51 + 1,22

    Записываем в столбик данное выражение, соблюдая правило «запятая под запятой»

    В первую очередь складываем дробную часть, а именно сотые части 1+2=3. Записываем тройку в сотой части нашего ответа:

    Теперь складываем десятые части 5+2=7. Записываем семёрку в десятой части нашего ответа:

    Теперь складываем целые части 3+1=4. Записываем четвёрку в целой части нашего ответа:

    Отделяем запятой целую часть от дробной, соблюдая правило «запятая под запятой»:

    Получили ответ 4,73. Значит значение выражения 3,51 + 1,22 равно 4,73

    3,51 + 1,22 = 4,73

    Как и в обычных числах, при сложении десятичных дробей может произойти . В этом случае в ответе записывается одна цифра, а остальные переносят на следующий разряд.

    Пример 3.
    Найти значение выражения 2,65 + 3,27

    Записываем в столбик данное выражение:

    Складываем сотые части 5+7=12. Число 12 не поместится в сотой части нашего ответа. Поэтому в сотой части записываем цифру 2, а единицу переносим на следующий разряд:

    Теперь складываем десятые части 6+2=8 плюс единица, которая досталась от предыдущей операции, получим 9. Записываем цифру 9 в десятой части нашего ответа:

    Теперь складываем целые части 2+3=5. Записываем цифру 5 в целой части нашего ответа:

    Получили ответ 5,92. Значит значение выражения 2,65 + 3,27 равно 5,92

    2,65 + 3,27 = 5,92

    Пример 4.
    Найти значение выражения 9,5 + 2,8

    Записываем в столбик данное выражение

    Складываем дробные части 5 + 8 = 13. Число 13 не поместится в дробной часть нашего ответа, поэтому сначала записываем цифру 3, а единицу переносим на следующий разряд, точнее переносим её к целой части:

    Теперь складываем целые части 9+2=11 плюс единица, которая досталась от предыдущей операции, получаем 12. Записываем число 12 в целой части нашего ответа:

    Отделяем запятой целую часть от дробной:

    Получили ответ 12,3. Значит значение выражения 9,5 + 2,8 равно 12,3

    9,5 + 2,8 = 12,3

    При сложении десятичных дробей количество цифр после запятой в обеих дробях должно быть одинаковым. Если цифр не хватает, то эти места в дробной части заполняются нулями.

    Пример 5
    . Найти значение выражения: 12,725 + 1,7

    Прежде чем записывать в столбик данное выражение, сделаем количество цифр после запятой в обеих дробях одинаковым. В десятичной дроби 12,725 после запятой три цифры, а в дроби 1,7 только одна. Значит в дроби 1,7 в конце нужно добавить два нуля. Тогда получим дробь 1,700. Теперь можно записать в столбик данное выражение и начать вычислять:

    Складываем тысячные части 5+0=5. Записываем цифру 5 в тысячной части нашего ответа:

    Складываем сотые части 2+0=2. Записываем цифру 2 в сотой части нашего ответа:

    Складываем десятые части 7+7=14. Число 14 не поместится в десятой части нашего ответа. Поэтому сначала записываем цифру 4, а единицу переносим на следующий разряд:

    Теперь складываем целые части 12+1=13 плюс единица, которая досталась от предыдущей операции, получаем 14. Записываем число 14 в целой части нашего ответа:

    Отделяем запятой целую часть от дробной:

    Получили ответ 14,425. Значит значение выражения 12,725+1,700 равно 14,425

    12,725+ 1,700 = 14,425

    Вычитание десятичных дробей

    При вычитании десятичных дробей нужно соблюдать те же правила, что и при сложении: «запятая под запятой» и «равное количества цифр после запятой».

    Пример 1.
    Найти значение выражения 2,5 − 2,2

    Записываем в столбик данное выражение, соблюдая правило «запятая под запятой»:

    Вычисляем дробную часть 5−2=3. Записываем цифру 3 в десятой части нашего ответа:

    Вычисляем целую часть 2−2=0. Записываем ноль в целой части нашего ответа:

    Отделяем запятой целую часть от дробной:

    Получили ответ 0,3. Значит значение выражения 2,5 − 2,2 равно 0,3

    2,5 − 2,2 = 0,3

    Пример 2.
    Найти значение выражения 7,353 — 3,1

    В этом выражении разное количество цифр после запятой. В дроби 7,353 после запятой три цифры, а в дроби 3,1 только одна. Значит в дроби 3,1 в конце нужно добавить два нуля, чтобы сделать количество цифр в обеих дробях одинаковым. Тогда получим 3,100.

    Теперь можно записать в столбик данное выражение и вычислить его:

    Получили ответ 4,253. Значит значение выражения 7,353 − 3,1 равно 4,253

    7,353 — 3,1 = 4,253

    Как и в обычных числах, иногда придётся занимать единицу у соседнего разряда, если вычитание станет невозможным.

    Пример 3.
    Найти значение выражения 3,46 − 2,39

    Вычитаем сотые части 6−9. От число 6 не вычесть число 9. Поэтому нужно занять единицу у соседнего разряда. Заняв единицу у соседнего разряда число 6 обращается в число 16. Теперь можно вычислить сотые части 16−9=7. Записываем семёрку в сотой части нашего ответа:

    Теперь вычитаем десятые части. Поскольку мы заняли в разряде десятых одну единицу, то цифра, которая там располагалась, уменьшилась на одну единицу. Другими словами, в разряде десятых теперь не цифра 4, а цифра 3. Вычислим десятые части 3−3=0. Записываем ноль в десятой части нашего ответа:

    Теперь вычитаем целые части 3−2=1. Записываем единицу в целой части нашего ответа:

    Отделяем запятой целую часть от дробной:

    Получили ответ 1,07. Значит значение выражения 3,46−2,39 равно 1,07

    3,46−2,39=1,07

    Пример 4
    . Найти значение выражения 3−1,2

    В этом примере из целого числа вычитается десятичная дробь. Запишем данное выражение столбиком так, чтобы целая часть десятичной дроби 1,23 оказалась под числом 3

    Теперь сделаем количество цифр после запятой одинаковым. Для этого после числа 3 поставим запятую и допишем один ноль:

    Теперь вычитаем десятые части: 0−2. От нуля не вычесть число 2. Поэтому нужно занять единицу у соседнего разряда. Заняв единицу у соседнего разряда, 0 обращается в число 10. Теперь можно вычислить десятые части 10−2=8. Записываем восьмёрку в десятой части нашего ответа:

    Теперь вычитаем целые части. Раньше в целой располагалось число 3, но мы заняли у него одну единицу. В результате оно обратилось в число 2. Поэтому из 2 вычитаем 1. 2−1=1. Записываем единицу в целой части нашего ответа:

    Отделяем запятой целую часть от дробной:

    Получили ответ 1,8. Значит значение выражения 3−1,2 равно 1,8

    Умножение десятичных дробей

    Умножение десятичных дробей это просто и даже увлекательно. Чтобы перемножить десятичные дроби, нужно перемножить их как обычные числа, не обращая внимания на запятые.

    Получив ответ, необходимо отделить запятой целую часть от дробной. Чтобы сделать это, надо посчитать количество цифр после запятой в обеих дробях, затем в ответе отсчитать справа столько же цифр и поставить запятую.

    Пример 1.
    Найти значение выражения 2,5 × 1,5

    Перемножим эти десятичные дроби как обычные числа, не обращая внимания на запятые. Чтобы не обращать внимания на запятые, можно на время представить, что они вообще отсутствуют:

    Получили 375. В этом числе необходимо отделить запятой целую часть от дробной. Для этого нужно посчитать количество цифр после запятой в дробях 2,5 и 1,5. В первой дроби после запятой одна цифра, во второй дроби тоже одна. Итого две цифры.

    Возвращаемся к числу 375 и начинаем двигаться справа налево. Нам нужно отсчитать две цифры справа и поставить запятую:

    Получили ответ 3,75. Значит значение выражения 2,5 × 1,5 равно 3,75

    2,5 × 1,5 = 3,75

    Пример 2.
    Найти значение выражения 12,85 × 2,7

    Перемножим эти десятичные дроби, не обращая внимания на запятые:

    Получили 34695. В этом числе нужно отделить запятой целую часть от дробной. Для этого необходимо посчитать количество цифр после запятой в дробях 12,85 и 2,7. В дроби 12,85 после запятой две цифры, в дроби 2,7 одна цифра — итого три цифры.

    Возвращаемся к числу 34695 и начинаем двигаться справа налево. Нам нужно отсчитать три цифры справа и поставить запятую:

    Получили ответ 34,695. Значит значение выражения 12,85 × 2,7 равно 34,695

    12,85 × 2,7 = 34,695

    Умножение десятичной дроби на обычное число

    Иногда возникают ситуации, когда требуется умножить десятичную дробь на обычное число.

    Чтобы перемножить десятичную дробь и обычное число, нужно перемножить их, не обращая внимания на запятую в десятичной дроби. Получив ответ, необходимо отделить запятой целую часть от дробной. Для этого нужно посчитать количество цифр после запятой в десятичной дроби, затем в ответе отсчитать справа столько же цифр и поставить запятую.

    Например, умножим 2,54 на 2

    Умножаем десятичную дробь 2,54 на обычное число 2, не обращая внимания на запятую:

    Получили число 508. В этом числе нужно отделить запятой целую часть от дробной. Для этого необходимо посчитать количество цифр после запятой в дроби 2,54. В дроби 2,54 после запятой две цифры.

    Возвращаемся к числу 508 и начинаем двигаться справа налево. Нам нужно отсчитать две цифры справа и поставить запятую:

    Получили ответ 5,08. Значит значение выражения 2,54 × 2 равно 5,08

    2,54 × 2 = 5,08

    Умножение десятичных дробей на 10, 100, 1000

    Умножение десятичных дробей на 10, 100 или 1000 выполняется таким же образом, как и умножение десятичных дробей на обычные числа. Нужно выполнить умножение, не обращая внимания на запятую в десятичной дроби, затем в ответе отделить целую часть от дробной, отсчитав справа столько же цифр, сколько было цифр после запятой в десятичной дроби.

    Например, умножим 2,88 на 10

    Умножим десятичную дробь 2,88 на 10, не обращая внимания на запятую в десятичной дроби:

    Получили 2880. В этом числе нужно отделить запятой целую часть от дробной. Для этого необходимо посчитать количество цифр после запятой в дроби 2,88. Видим, что в дроби 2,88 после запятой две цифры.

    Возвращаемся к числу 2880 и начинаем двигаться справа налево. Нам нужно отсчитать две цифры справа и поставить запятую:

    Получили ответ 28,80. Отбросим последний ноль — получим 28,8. Значит значение выражения 2,88×10 равно 28,8

    2,88 × 10 = 28,8

    Есть и второй способ умножения десятичных дробей на 10, 100, 1000. Этот способ намного проще и удобнее. Он заключается в том, что запятая в десятичной дроби передвигается вправо на столько цифр, сколько нулей во множителе.

    Например, решим предыдущий пример 2,88×10 этим способом. Не приводя никаких вычислений, сразу же смотрим на множитель 10. Нас интересует сколько в нём нулей. Видим, что в нём один ноль. Теперь в дроби 2,88 передвигаем запятую вправо на одну цифру, получим 28,8.

    2,88 × 10 = 28,8

    Попробуем умножить 2,88 на 100. Сразу же смотрим на множитель 100. Нас интересует сколько в нём нулей. Видим, что в нём два нуля. Теперь в дроби 2,88 передвигаем запятую вправо на две цифры, получаем 288

    2,88 × 100 = 288

    Попробуем умножить 2,88 на 1000. Сразу же смотрим на множитель 1000. Нас интересует сколько в нём нулей. Видим, что в нём три нуля. Теперь в дроби 2,88 передвигаем запятую вправо на три цифры. Третьей цифры там нет, поэтому мы дописываем ещё один ноль. В итоге получаем 2880.

    2,88 × 1000 = 2880

    Умножение десятичных дробей на 0,1 0,01 и 0,001

    Умножение десятичных дробей на 0,1, 0,01 и 0,001 происходит таким же образом, как и умножение десятичной дроби на десятичную дробь. Необходимо перемножить дроби, как обычные числа, и в ответе поставить запятую, отсчитав столько цифр справа, сколько цифр после запятой в обеих дробях.

    Например, умножим 3,25 на 0,1

    Умножаем эти дроби, как обычные числа, не обращая внимания на запятые:

    Получили 325. В этом числе нужно отделить запятой целую часть от дробной. Для этого необходимо посчитать количество цифр после запятой в дробях 3,25 и 0,1. В дроби 3,25 после запятой две цифры, в дроби 0,1 одна цифра. Итого три цифры.

    Возвращаемся к числу 325 и начинаем двигаться справа налево. Нам нужно отсчитать три цифры справа и поставить запятую. Отсчитав три цифры мы обнаруживаем, что цифры закончились. В этом случае нужно дописать один ноль и поставить запятую:

    Получили ответ 0,325. Значит значение выражения 3,25 × 0,1 равно 0,325

    3,25 × 0,1 = 0,325

    Есть и второй способ умножения десятичных дробей на 0,1, 0,01 и 0,001. Этот способ намного проще и удобнее. Он заключается в том, что запятая в десятичной дроби передвигается влево на столько цифр, сколько нулей во множителе.

    Например, решим предыдущий пример 3,25 × 0,1 этим способом. Не приводя никаких вычислений сразу же смотрим на множитель 0,1. Нас интересует сколько в нём нулей. Видим, что в нём один ноль. Теперь в дроби 3,25 передвигаем запятую влево на одну цифру. Передвинув запятую на одну цифру влево мы видим, что перед тройкой больше нет никаких цифр. В этом случае дописываем один ноль и ставим запятую. В результате получаем 0,325

    3,25 × 0,1 = 0,325

    Попробуем умножить 3,25 на 0,01. Сразу же смотрим на множитель 0,01. Нас интересует сколько в нём нулей. Видим, что в нём два нуля. Теперь в дроби 3,25 передвигаем запятую влево на две цифры, получаем 0,0325

    3,25 × 0,01 = 0,0325

    Попробуем умножить 3,25 на 0,001. Сразу же смотрим на множитель 0,001. Нас интересует сколько в нём нулей. Видим, что в нём три нуля. Теперь в дроби 3,25 передвигаем запятую влево на три цифры, получаем 0,00325

    3,25 × 0,001 = 0,00325

    Нельзя путать умножение десятичных дробей на 0,1, 0,001 и 0,001 с умножением на 10, 100, 1000. Типичная ошибка большинства людей.

    При умножении на 10, 100, 1000 запятая переносится вправо на столько же цифр сколько нулей во множителе.

    А при умножении на 0,1, 0,01 и 0,001 запятая переносится влево на столько же цифр сколько нулей во множителе.

    Если на первых порах это сложно запомнить, можно пользоваться первым способом, в котором умножение выполняется как с обычными числами. В ответе нужно будет отделить целую часть от дробной, отсчитав справа столько же цифр, сколько цифр после запятой в обеих дробях.

    Деление меньшего числа на большее. Продвинутый уровень.

    В одном из предыдущих уроков мы сказали, что при делении меньшего числа на большее получается дробь, в числителе которой делимое, а в знаменателе – делитель.

    Например, чтобы разделить одно яблоко на двоих, нужно в числитель записать 1 (одно яблоко), а в знаменатель записать 2 (двое друзей). В результате получим дробь . Значит каждому другу достанется по яблока. Другими словами, по половине яблока. Дробь это ответ к задаче «как разделить одно яблоко на двоих»

    Оказывается, можно решать эту задачу и дальше, если разделить 1 на 2. Ведь дробная черта в любой дроби означает деление, а значит и в дроби это деление разрешено. Но как? Мы ведь привыкли к тому, что делимое всегда больше делителя. А здесь наоборот, делимое меньше делителя.

    Всё станет ясным, если вспомнить, что дробь означает дробление, деление, разделение. А значит и единица может быть раздроблена на сколько угодно частей, а не только на две части.

    При разделении меньшего числа на большее получается десятичная дробь, в которой целая часть будет 0 (нулевой). Дробная часть же может быть любой.

    Итак, разделим 1 на 2. Решим этот пример уголком:

    Единицу на два просто так нацело не разделить. Если задать вопрос «сколько двоек в единице»

    , то ответом будет 0. Поэтому в частном записываем 0 и ставим запятую:

    Теперь как обычно умножаем частное на делитель, чтобы вытащить остаток:

    Настал момент, когда единицу можно дробить на две части. Для этого справа от полученной единички дописываем ещё один ноль:

    Получили 10. Делим 10 на 2, получаем 5. Записываем пятёрку в дробной части нашего ответа:

    Теперь вытаскиваем последний остаток, чтобы завершить вычисление. Умножаем 5 на 2, получаем 10

    Получили ответ 0,5. Значит дробь равна 0,5

    Половину яблока можно записать и с помощью десятичной дроби 0,5. Если сложить эти две половинки (0,5 и 0,5), мы опять получим изначальное одно целое яблоко:

    Этот момент также можно понять, если представить, как 1 см делится на две части. Если 1 сантиметр разделить на 2 части, то получится 0,5 см

    Пример 2.
    Найти значение выражения 4: 5

    Сколько пятёрок в четвёрке? Нисколько. Записываем в частном 0 и ставим запятую:

    Умножаем 0 на 5, получаем 0. Записываем ноль под четвёркой. Сразу же вычитаем этот ноль из делимого:

    Теперь начнём дробить (делить) четвёрку на 5 частей. Для этого справа от 4 дописываем ноль и делим 40 на 5, получаем 8. Записываем восьмёрку в частном.

    Завершаем пример, умножив 8 на 5, и получив 40:

    Получили ответ 0,8. Значит значение выражения 4: 5 равно 0,8

    Пример 3.
    Найти значение выражения 5: 125

    Сколько чисел 125 в пятёрке? Нисколько. Записываем 0 в частном и ставим запятую:

    Умножаем 0 на 5, получаем 0. Записываем 0 под пятёркой. Сразу же вычитаем из пятёрки 0

    Теперь начнём дробить (делить) пятёрку на 125 частей. Для этого справа от этой пятёрки запишем ноль:

    Делим 50 на 125. Сколько чисел 125 в числе 50? Нисколько. Значит в частном опять записываем 0

    Умножаем 0 на 125, получаем 0. Записываем этот ноль под 50. Сразу же вычитаем 0 из 50

    Теперь делим число 50 на 125 частей. Для этого справа от 50 запишем ещё один ноль:

    Делим 500 на 125. Сколько чисел 125 в числе 500. В числе 500 четыре числа 125. Записываем четвёрку в частном:

    Завершаем пример, умножив 4 на 125, и получив 500

    Получили ответ 0,04. Значит значение выражения 5: 125 равно 0,04

    Деление чисел без остатка

    Итак, поставим в частном после единицы запятую, тем самым указывая, что деление целых частей закончилось и мы приступаем к дробной части:

    Допишем ноль к остатку 4

    Теперь делим 40 на 5, получаем 8. Записываем восьмёрку в частном:

    40−40=0. Получили 0 в остатке. Значит деление на этом полностью завершено. При делении 9 на 5 получается десятичная дробь 1,8:

    9: 5 = 1,8

    Пример 2
    . Разделить 84 на 5 без остатка

    Сначала разделим 84 на 5 как обычно с остатком:

    Получили в частном 16 и еще 4 в остатке. Теперь разделим этот остаток на 5. Поставим в частном запятую, а к остатку 4 допишем 0

    Теперь делим 40 на 5, получаем 8. Записываем восьмерку в частном после запятой:

    и завершаем пример, проверив есть ли еще остаток:

    Деление десятичной дроби на обычное число

    Десятичная дробь, как мы знаем состоит из целой и дробной части. При делении десятичной дроби на обычное число в первую очередь нужно:

    • разделить целую часть десятичной дроби на это число;
    • после того, как целая часть будет разделена, нужно в частном сразу же поставить запятую и продолжить вычисление, как в обычном делении.

    Например, разделим 4,8 на 2

    Запишем этот пример уголком:

    Теперь разделим целую часть на 2. Четыре разделить на два будет два. Записываем двойку в частном и сразу же ставим запятую:

    Теперь умножаем частное на делитель и смотрим есть ли остаток от деления:

    4−4=0. Остаток равен нулю. Ноль пока не записываем, поскольку решение не завершено. Далее продолжаем вычислять, как в обычном делении. Сносим 8 и делим её на 2

    8: 2 = 4. Записываем четвёрку в частном и сразу умножаем её на делитель:

    Получили ответ 2,4. Значение выражения 4,8: 2 равно 2,4

    Пример 2.
    Найти значение выражения 8,43: 3

    Делим 8 на 3, получаем 2. Сразу же ставим запятую после двойки:

    Теперь умножаем частное на делитель 2 × 3 = 6. Записываем шестёрку под восьмёркой и находим остаток:

    Делим 24 на 3, получаем 8. Записываем восьмёрку в частном. Сразу же умножаем её на делитель, чтобы найти остаток от деления:

    24−24=0. Остаток равен нулю. Ноль пока не записываем. Сносим последнюю тройку из делимого и делим на 3, получим 1. Сразу же умножаем 1 на 3, чтобы завершить этот пример:

    Получили ответ 2,81. Значит значение выражения 8,43: 3 равно 2,81

    Деление десятичной дроби на десятичную дробь

    Чтобы разделить десятичную дробь на десятичную дробь, надо в делимом и в делителе перенести запятую вправо на столько же цифр, сколько их после запятой в делителе, и затем выполнить деление на обычное число.

    Например, разделим 5,95 на 1,7

    Запишем уголком данное выражение

    Теперь в делимом и в делителе перенесём запятую вправо на столько же цифр, сколько их после запятой в делителе. В делителе после запятой одна цифра. Значит мы должны в делимом и в делителе перенести запятую вправо на одну цифру. Переносим:

    После перенесения запятой вправо на одну цифру десятичная дробь 5,95 обратилась в дробь 59,5. А десятичная дробь 1,7 после перенесения запятой вправо на одну цифру обратилась в обычное число 17. А как делить десятичную дробь на обычное число мы уже знаем. Дальнейшее вычисление не составляет особого труда:

    Запятая переносится вправо с целью облегчить деление. Это допускается по причине того, что при умножении или делении делимого и делителя на одно и то же число, частное не меняется. Что это значит?

    Это одна из интересных особенностей деления. Его называют свойством частного. Рассмотрим выражение 9: 3 = 3. Если в этом выражении делимое и делитель умножить или разделить на одно и то же число, то частное 3 не изменится.

    Давайте умножим делимое и делитель на 2, и посмотрим, что из этого получится:

    (9 × 2
    ) : (3 × 2
    ) = 18: 6 = 3

    Как видно из примера, частное не поменялось.

    Тоже самое происходит, когда мы переносим запятую в делимом и в делителе. В предыдущем примере, где мы делили 5,91 на 1,7 мы перенесли в делимом и делителе запятую на одну цифру вправо. После переноса запятой, дробь 5,91 преобразовалась в дробь 59,1 а дробь 1,7 преобразовалась в обычное число 17.

    На самом деле внутри этого процесса происходило умножение на 10. Вот как это выглядело:

    5,91 × 10 = 59,1

    Поэтому от количества цифр после запятой в делителе зависит то, на что будет умножено делимое и делитель. Другими словами, от количества цифр после запятой в делителе будет зависеть то, на сколько цифр в делимом и в делителе запятая будет перенесена вправо.

    Деление десятичной дроби на 10, 100, 1000

    Деление десятичной дроби на 10, 100, или 1000 осуществляется таким же образом, как и . Например, разделим 2,1 на 10. Решим этот пример уголком:

    Но есть и второй способ. Он более лёгкий. Суть этого способа в том, что запятая в делимом переносится влево на столько цифр, сколько нулей в делителе.

    Решим предыдущий пример этим способом. 2,1: 10. Смотрим на делитель. Нас интересует сколько в нём нулей. Видим, что там один ноль. Значит в делимом 2,1 нужно перенести запятую влево на одну цифру. Переносим запятую влево на одну цифру и видим, что там больше не осталось цифр. В этом случае перед цифрой дописываем ещё один ноль. В итоге получаем 0,21

    Попробуем разделить 2,1 на 100. В числе 100 два нуля. Значит в делимом 2,1 надо перенести запятую влево на две цифры:

    2,1: 100 = 0,021

    Попробуем разделить 2,1 на 1000. В числе 1000 три нуля. Значит в делимом 2,1 надо перенести запятую влево на три цифры:

    2,1: 1000 = 0,0021

    Деление десятичной дроби на 0,1, 0,01 и 0,001

    Деление десятичной дроби на 0,1, 0,01, и 0,001 осуществляется таким же образом, как и . В делимом и в делителе надо перенести запятую вправо на столько цифр, сколько их после запятой в делителе.

    Например, разделим 6,3 на 0,1. В первую очередь перенесём запятые в делимом и в делителе вправо на столько же цифр, сколько их после запятой в делителе. В делителе после запятой одна цифра. Значит переносим запятые в делимом и в делителе вправо на одну цифру.

    После перенесения запятой вправо на одну цифру, десятичная дробь 6,3 превращается в обычное число 63, а десятичная дробь 0,1 после перенесения запятой вправо на одну цифру превращается в единицу. А разделить 63 на 1 очень просто:

    Значит значение выражения 6,3: 0,1 равно 63

    Но есть и второй способ. Он более лёгкий. Суть этого способа в том, что запятая в делимом переносится вправо на столько цифр, сколько нулей в делителе.

    Решим предыдущий пример этим способом. 6,3: 0,1. Смотрим на делитель. Нас интересует сколько в нём нулей. Видим, что там один ноль. Значит в делимом 6,3 нужно перенести запятую вправо на одну цифру. Переносим запятую вправо на одну цифру и получаем 63

    Попробуем разделить 6,3 на 0,01. В делителе 0,01 два нуля. Значит в делимом 6,3 надо перенести запятую вправо на две цифры. Но в делимом после запятой только одна цифра. В этом случае в конце нужно дописать ещё один ноль. В результате получим 630

    Попробуем разделить 6,3 на 0,001. В делителе 0,001 три нуля. Значит в делимом 6,3 надо перенести запятую вправо на три цифры:

    6,3: 0,001 = 6300

    Задания для самостоятельного решения

    Понравился урок?
    Вступай в нашу новую группу Вконтакте и начни получать уведомления о новых уроках

    Умножение десятичных дробей
    происходит в три этапа.

    Десятичные дроби записывают в столбик и умножают как обыкновенные числа.

    Считаем количество знаков после запятой у первой десятичной дроби и у второй. Их количество складываем.

    В полученном результате отсчитываем справа налево столько же цифр, сколько получилось их в пункте выше и ставим запятую.

    Как умножать десятичные дроби

    Записываем десятичные дроби в столбик и умножаем их как натуральные числа, не обращая внимания на запятые. То есть 3,11 мы рассматриваем как 311 , а 0,01 как 1 .

    Получили 311 . Теперь считаем количество знаков (цифр) после запятой у обеих дробей. В первой десятичной дроби два знака и во второй — два. Общее количество цифр после запятых:

    Отсчитываем справа налево 4 знака (цифры) у полученного числа. В полученном результате цифр меньше, чем нужно отделить запятой. В таком случае нужно слева
    приписать недостающее число нулей.

    У нас не хватает одной цифры, поэтому приписываем слева один ноль.

    При умножении любой десятичной дроби
    на 10; 100; 1000 и т.д. запятая в десятичной дроби перемещается вправо на столько знаков, сколько нулей стоит после единицы.

  • 70,1 · 10 = 701
  • 0,023 · 100 = 2,3
  • 5,6 · 1 000 = 5 600
  • Чтобы умножить десятичную дробь на 0,1; 0,01; 0,001 и т.д., надо в этой дроби перенести запятую влево на столько знаков, сколько нулей стоит перед единицей.

    Считаем и ноль целых!

    • 12 · 0,1 = 1,2
    • 0,05 · 0,1 = 0,005
    • 1,256 · 0,01 = 0,012 56
    • Чтобы понять, как умножать десятичные дроби, рассмотрим конкретные примеры.

      Правило умножения десятичных дробей

      1) Умножаем, не обращая внимания на запятую.

      2) В результате отделяем после запятой столько цифр, сколько их после запятых в обоих множителях вместе.

      Найти произведение десятичных дробей:

      Чтобы умножить десятичные дроби, умножаем, не обращая внимания на запятые. То есть мы умножаем не 6,8 и 3,4, а 68 и 34. В результате отделяем после запятой столько цифр, сколько их после запятых в обоих множителях вместе. В первом множителе после запятой одна цифра, во втором — тоже одна. Итого, отделяем после запятой две цифры.Таким образом, получили окончательный ответ: 6,8∙3,4=23,12.

      Умножаем десятичные дроби, не принимая во внимание запятую. То есть фактически вместо умножения 36,85 на 1,14 мы умножаем 3685 на 14. Получаем 51590. Теперь в этом результате надо отделить запятой столько цифр, сколько их в обоих множителях вместе. В первом числе после запятой две цифры, во втором — одна. Итого, отделяем запятой три цифры. Поскольку в конце записи после запятой стоит нуль, в ответ мы его не пишем: 36,85∙1,4=51,59.

      Чтобы умножить эти десятичные дроби, умножим числа, не обращая внимания на запятые. То есть умножаем натуральные числа 2315 и 7. Получаем 16205. В этом числе нужно отделить после запятой четыре цифры — столько, сколько их в обоих множителях вместе (в каждом — по два). Окончательный ответ: 23,15∙0,07=1,6205.

      Умножение десятичной дроби на натуральное число выполняется аналогично. Умножаем числа, не обращая внимания на запятую, то есть 75 умножаем на 16. В полученном результате после запятой должно стоять столько же знаков, сколько их в обоих множителях вместе — один. Таким образом, 75∙1,6=120,0=120.

      Умножение десятичных дробей начинаем с того, что умножаем натуральные числа, так как на запятые не обращаем внимания. После этого отделяем после запятой столько цифр, сколько их в обоих множителях вместе. В первом числе после запятой два знака, во втором — тоже два. Итого, в результате после запятой должно стоять четыре цифры: 4,72∙5,04=23,7888.

      И еще пара примеров на умножение десятичных дробей:

      www.for6cl.uznateshe.ru

      Умножение десятичных дробей, правила, примеры, решения.

      Переходим к изучению следующего действия с десятичными дробями, сейчас мы всесторонне рассмотрим умножение десятичных дробей
      . Сначала обговорим общие принципы умножения десятичных дробей. После этого перейдем к умножению десятичной дроби на десятичную дробь, покажем, как выполняется умножение десятичных дробей столбиком, рассмотрим решения примеров. Дальше разберем умножение десятичных дробей на натуральные числа, в частности на 10, 100 и т.д. В заключение поговорим об умножении десятичных дробей на обыкновенные дроби и смешанные числа.

      Сразу скажем, что в этой статье мы будем говорить лишь об умножении положительных десятичных дробей (смотрите положительные и отрицательные числа). Остальные случаи разобраны в статьях умножение рациональных чисел и умножение действительных чисел
      .

      Навигация по странице.

      Общие принципы умножения десятичных дробей

      Обсудим общие принципы, которых следует придерживаться при проведении умножения с десятичными дробями.

      Так как конечные десятичные дроби и бесконечные периодические дроби являются десятичной формой записи обыкновенных дробей, то умножение таких десятичных дробей по сути является умножением обыкновенных дробей. Иными словами, умножение конечных десятичных дробей
      , умножение конечной и периодической десятичных дробей
      , а также умножение периодических десятичных дробей
      сводится к умножению обыкновенных дробей после перевода десятичных дробей в обыкновенные.

      Рассмотрим примеры применения озвученного принципа умножения десятичных дробей.

      Выполните умножение десятичных дробей 1,5 и 0,75 .

      Заменим умножаемые десятичные дроби соответствующими обыкновенными дробями. Так как 1,5=15/10 и 0,75=75/100 , то. Можно провести сокращение дроби, после чего выделить целую часть из неправильной дроби, а удобнее полученную обыкновенную дробь 1 125/1 000 записать в виде десятичной дроби 1,125 .

      Следует отметить, что конечные десятичные дроби удобно умножать столбиком, об этом способе умножения десятичных дробей мы поговорим в следующем пункте.

      Рассмотрим пример умножения периодических десятичных дробей.

      Вычислите произведение периодических десятичных дробей 0,(3) и 2,(36) .

      Выполним перевод периодических десятичных дробей в обыкновенные дроби:

      Тогда. Можно полученную обыкновенную дробь перевести в десятичную дробь:

      Если среди умножаемых десятичных дробей присутствуют бесконечные непериодические, то все умножаемые дроби, в том числе конечные и периодические, следует округлить до некоторого разряда (смотрите округление чисел
      ), после чего выполнять умножение полученных после округления конечных десятичных дробей.

      Выполните умножение десятичных дробей 5,382… и 0,2 .

      Сначала округлим бесконечную непериодическую десятичную дробь, округление можно провести до сотых, имеем 5,382…≈5,38 . Конечную десятичную дробь 0,2 округлять до сотых нет необходимости. Таким образом, 5,382…·0,2≈5,38·0,2 . Осталось вычислить произведение конечных десятичных дробей: 5,38·0,2=538/100·2/10= 1 076/1 000=1,076 .

      Умножение десятичных дробей столбиком

      Умножение конечных десятичных дробей можно выполнять столбиком, аналогично умножению столбиком натуральных чисел.

      Сформулируем правило умножения десятичных дробей столбиком
      . Чтобы умножить десятичные дроби столбиком, надо:

      • не обращая внимания на запятые, выполнить умножение по всем правилам умножения столбиком натуральных чисел;
      • в полученном числе отделить десятичной запятой столько цифр справа, сколько десятичных знаков в обоих множителях вместе, при этом если в произведении не хватает цифр, то слева нужно дописать нужное количество нулей.
      • Рассмотрим примеры умножения десятичных дробей столбиком.

        Выполните умножение десятичных дробей 63,37 и 0,12 .

        Проведем умножение десятичных дробей столбиком. Сначала умножаем числа, не обращая внимания на запятые:

        Осталось в полученном произведении поставить запятую. Ей нужно отделить 4 цифры справа, так как в множителях в сумме четыре десятичных знака (два в дроби 3,37 и два в дроби 0,12). Цифр там хватает, поэтому нулей слева дописывать не придется. Закончим запись:

        В итоге имеем 3,37·0,12=7,6044 .

        Вычислите произведение десятичных дробей 3,2601 и 0,0254 .

        Выполнив умножение столбиком без учета запятых, получаем следующую картину:

        Теперь в произведении нужно отделить запятой 8 цифр справа, так как общее количество десятичных знаков умножаемых дробей равно восьми. Но в произведении только 7 цифр, поэтому, нужно слева приписать столько нулей, чтобы можно было отделить запятой 8 цифр. В нашем случае нужно приписать два нуля:

        На этом умножение десятичных дробей столбиком закончено.

        Умножение десятичных дробей на 0,1, 0,01, и т.д.

        Довольно часто приходится умножать десятичные дроби на 0,1 , 0,01 и так далее. Поэтому целесообразно сформулировать правило умножения десятичной дроби на эти числа, которое следует из рассмотренных выше принципов умножения десятичных дробей.

        Итак, умножение данной десятичной дроби на 0,1 , 0,01 , 0,001 и так далее
        дает дробь, которая получается из исходной, если в ее записи перенести запятую влево на 1 , 2 , 3 и так далее цифр соответственно, при этом если не хватает цифр для переноса запятой, то нужно слева дописать необходимое количество нулей.

        Например, чтобы умножить десятичную дробь 54,34 на 0,1 , надо в дроби 54,34 перенести запятую влево на 1 цифру, при этом получится дробь 5,434 , то есть, 54,34·0,1=5,434 . Приведем еще один пример. Умножим десятичную дробь 9,3 на 0,0001 . Для этого нам нужно в умножаемой десятичной дроби 9,3 перенести запятую на 4 цифры влево, но запись дроби 9,3 не содержит такого количества знаков. Поэтому нам нужно в записи дроби 9,3 слева приписать столько нулей, чтобы можно было беспрепятственно осуществить перенос запятой на 4 цифры, имеем 9,3·0,0001=0,00093 .

        Заметим, что озвученное правило умножения десятичной дроби на 0,1, 0,01, … справедливо и для бесконечных десятичных дробей. К примеру, 0,(18)·0,01=0,00(18) или 93,938…·0,1=9,3938… .

        Умножение десятичной дроби на натуральное число

        По своей сути умножение десятичных дробей на натуральные числа
        ничем не отличается от умножения десятичной дроби на десятичную дробь.

        Конечную десятичную дробь умножать на натуральное число удобнее всего столбиком, при этом следует придерживаться правил умножения столбиком десятичных дробей, рассмотренных в одном из предыдущих пунктов.

        Вычислите произведение 15·2,27 .

        Проведем умножение натурального числа на десятичную дробь столбиком:

        При умножении периодической десятичной дроби на натуральное число, периодическую дробь следует заменить обыкновенной дробью.

        Умножьте десятичную дробь 0,(42) на натуральное число 22 .

        Сначала переведем периодическую десятичную дробь в обыкновенную дробь:

        Теперь выполним умножение: . Этот результат в виде десятичной дроби имеет вид 9,(3) .

        А при умножении бесконечной непериодической десятичной дроби на натуральное число нужно предварительно провести округление.

        Выполните умножение 4·2,145… .

        Округлив до сотых исходную бесконечную десятичную дробь, мы придем к умножению натурального числа и конечной десятичной дроби. Имеем 4·2,145…≈4·2,15=8,60 .

        Умножение десятичной дроби на 10, 100, …

        Довольно часто приходится умножать десятичные дроби на 10, 100, … Поэтому целесообразно подробно остановиться на этих случаях.

        Озвучим правило умножения десятичной дроби на 10, 100, 1 000 и т.д.
        При умножении десятичной дроби на 10, 100, … в ее записи нужно перенести запятую вправо на 1, 2, 3, … цифры соответственно и отбросить лишние нули слева; если в записи умножаемой дроби не хватает цифр для переноса запятой, то нужно дописать необходимое количество нулей справа.

        Умножьте десятичную дробь 0,0783 на 100 .

        Перенесем в записи дроби 0,0783 на две цифры вправо, при этом получим 007,83 . Отбросив два нуля слева, получаем десятичную дробь 7,38 . Таким образом, 0,0783·100=7,83 .

        Выполните умножение десятичной дроби 0,02 на 10 000 .

        Чтобы умножить 0,02 на 10 000 , нам нужно перенести запятую на 4 цифры вправо. Очевидно, в записи дроби 0,02 не хватает цифр для переноса запятой на 4 цифры, поэтому допишем несколько нулей справа, чтобы можно было осуществить перенос запятой. В нашем примере достаточно дописать три нуля, имеем 0,02000 . После переноса запятой получим запись 00200,0 . Отбросив нули слева, имеем число 200,0 , которое равно натуральному числу 200 , оно и является результатом умножения десятичной дроби 0,02 на 10 000 .

        Озвученное правило справедливо и для умножения бесконечных десятичных дробей на 10, 100, … При умножении периодических десятичных дробей нужно быть аккуратными с периодом дроби, которая является результатом умножения.

        Умножьте периодическую десятичную дробь 5,32(672) на 1 000 .

        Перед умножением распишем периодическую десятичную дробь как 5,32672672672… , это нам позволит не допустить ошибки. Теперь перенесем запятую вправо на 3 знака, имеем 5 326,726726… . Таким образом, после умножения получается периодическая десятичная дробь 5 326,(726) .

        5,32(672)·1 000=5 326,(726) .

        При умножении бесконечных непериодических дробей на 10, 100, … нужно предварительно провести округление бесконечной дроби до некоторого разряда, после чего проводить умножение.

        Умножение десятичной дроби на обыкновенную дробь или смешанное число

        Для умножения конечной десятичной дроби или бесконечной периодической десятичной дроби на обыкновенную дробь или смешанное число, нужно десятичную дробь представить в виде обыкновенной дроби, после чего провести умножение.

        Проведите умножение десятичной дроби 0,4 на смешанное число.

        Так как 0,4=4/10=2/5 и, то. Полученное число можно записать в виде периодической десятичной дроби 1,5(3) .

        При умножении бесконечной непериодической десятичной дроби на обыкновенную дробь или смешанное число, обыкновенную дробь или смешанное число следует заменить десятичной дробью, после чего провести округление умножаемых дробей и закончить вычисления.

        Так как 2/3=0,6666… , то. После округления умножаемых дробей до тысячных, приходим к произведению двух конечных десятичных дробей 3,568 и 0,667 . Выполним умножение в столбик:

        Полученный результат следует округлить до тысячных, так как умножаемые дроби были взяты с точностью до тысячных, имеем 2,379856≈2,380 .

        www.cleverstudents.ru

        29. Умножение десятичных дробей. Правила

        Найдем площадь прямоугольника со сторонами равными
        1,4 дм и 0,3 дм. Переведем дециметры в сантиметры:

        1,4 дм = 14 см; 0,3 дм = 3 см.

        Теперь вычислим площадь в сантиметрах.

        S = 14 3 = 42 см 2 .

        Переведем квадратные сантиметры в квадратные
        дециметры:

        д м 2 = 0,42 д м 2 .

        Значит, S = 1,4 дм 0,3 дм = 0,42 дм 2 .

        Умножение двух десятичных дробей выполняется так:
        1) числа перемножаются без учета запятых.
        2) запятая в произведении ставится так, чтобы отделить справа
        столько же знаков, сколько отделено в обоих множителях
        вместе взятых. Например:

        1,1 0,2 = 0,22 ; 1,1 1,1 = 1,21 ; 2,2 0,1 = 0,22 .

        Примеры умножения десятичных дробей в столбик:

        Вместо умножения любого числа на 0,1 ; 0,01 ; 0,001 ,
        можно разделить это число на 10 ; 100 ; или 1000 соответственно.
        Например:

        22 0,1 = 2,2 ; 22: 10 = 2,2 .

        При умножении десятичной дроби на натуральное число, мы должны:

        1) перемножить числа, не обращая внимания на запятую;

        2) в полученном произведении поставить запятую так, чтобы справа
        от нее было столько же цифр, сколько в десятичной дроби.

        Найдем произведение 3,12 10 . По указанному выше правилу
        сначала умножаем 312 на 10 . Получим: 312 10 = 3120 .
        А теперь отделяем запятой две цифры справа и получаем:

        3,12 10 = 31,20 = 31,2 .

        Значит, при умножении 3,12 на 10 мы перенесли запятую на одну
        цифру вправо. Если умножить 3,12 на 100 , то получим 312 , то есть
        запятую перенесли на две цифры вправо.

        3,12 100 = 312,00 = 312 .

        При умножении десятичной дроби на 10 , 100 , 1000 и т. д., надо
        в этой дроби перенести запятую вправо на столько знаков, сколько нулей
        стоит в множителе. Например:

        0,065 1000 = 0065, = 65 ;

        2,9 1000 = 2,900 1000 = 2900, = 2900 .

        Задачи на тему «Умножение десятичных дробей»

        school-assistant.ru

        Сложение, вычитание, умножение и деление десятичных дробей

        Сложение и вычитание десятичных дробей аналогично сложению и вычитанию натуральных чисел, но с определенными условиями.

        Правило. производится по разрядам целой и дробной части как натуральных чисел.

        При письменном сложении и вычитании десятичных дробей
        запятая, отделяющая целую часть от дробной, должна находиться у слагаемых и суммы или у уменьшаемого, вычитаемого и разности в одном столбце (запятая под запятой от записи условия до конца вычисления).

        Сложение и вычитание десятичных дробей
        в строку:

        243,625 + 24,026 = 200 + 40 + 3 + 0,6 + 0,02 + 0,005 + 20 + 4 + 0,02 + 0,006 = 200 + (40 + 20) + (3 + 4)+ 0,6 + (0,02 + 0,02) + (0,005 + 0,006) = 200 + 60 + 7 + 0,6 + 0,04 + 0,011 = 200 + 60 + 7 + 0,6 + (0,04 + 0,01) + 0,001 = 200 + 60 + 7 + 0,6 + 0,05 + 0,001 = 267,651

        843,217 — 700,628 = (800 — 700) + 40 + 3 + (0,2 — 0,6) + (0,01 — 0,02) + (0,007 — 0,008) = 100 + 40 + 2 + (1,2 — 0,6) + (0,01 — 0,02) + (0,007 — 0,008) = 100 + 40 + 2 + 0,5 + (0,11 — 0,02) + (0,007 — 0,008) = 100 + 40 + 2 + 0,5 + 0,09 + (0,007 — 0,008) = 100 + 40 + 2 + 0,5 + 0,08 + (0,017 — 0,008) = 100 + 40 + 2 + 0,5 + 0,08 + 0,009 = 142,589

        Сложение и вычитание десятичных дробей
        в столбик:

        Сложение десятичных дробей требует верхней дополнительной строки для записи чисел, когда сумма разряда переходит через десяток. Вычитание десятичных дробей требует верхней дополнительной строки для того, чтобы отметить разряд, в котором одалживается 1.

        Если справа от слагаемого или уменьшаемого не хватает разрядов дробной части, то справа в дробной части можно дописывать столько нулей (увеличивать разрядность дробной части), сколько разрядов в другом слагаемом или уменьшаемом.

        Умножение десятичных дробей
        производится так же, как и умножение натуральных чисел, по тем же правилам, но в произведении ставится запятая по сумме разрядов множителей в дробной части, считая справа налево (сумма разрядов множителей — это количество разрядов после запятой у множителей, вместе взятых).

        При умножении десятичных дробей
        в столбик первая справа значащая цифра подписывается под первой справа значащей цифрой, как и в натуральных числах:

        Запись умножения десятичных дробей
        в столбик:

        Запись деления десятичных дробей
        в столбик:

        Подчеркнутые знаки — это знаки, за которые переносится запятая, потому что делитель должен быть целым числом.

        Правило. При делении дробей
        делитель десятичной дроби увеличивается на столько разрядов, сколько разрядов в дробной его части. Чтобы дробь не изменилась, на столько же разрядов увеличивается и делимое (в делимом и делителе запятая переносится на одно и то же число знаков). Запятая ставится в частном на том этапе деления, когда целая часть дроби разделена.

        Для десятичных дробей, как и для натуральных чисел, сохраняется правило: на ноль десятичную дробь делить нельзя!

    Рассмотрим тему преобразования выражений со степенями, но прежде остановимся на ряде преобразований, которые можно проводить с любыми выражениями, в том числе со степенными. Мы научимся раскрывать скобки, приводить подобные слагаемые, работать с основанием и показателем степени, использовать свойства степеней.

    Yandex.RTB R-A-339285-1

    Что представляют собой степенные выражения?

    В школьном курсе мало кто использует словосочетание «степенные выражения», зато этот термин постоянно встречается в сборниках для подготовки к ЕГЭ. В большинства случаев словосочетанием обозначаются выражения, которые содержат в своих записях степени. Это мы и отразим в нашем определении.

    Определение 1

    Степенное выражение
    – это выражение, которое содержит степени.

    Приведем несколько примеров степенных выражений, начиная со степени с натуральным показателем и заканчивая степенью с действительным показателем.

    Самыми простыми степенными выражениями можно считать степени числа с натуральным показателем: 3 2 , 7 5 + 1 , (2 + 1) 5 , (− 0 , 1) 4 , 2 2 3 3 , 3 · a 2 − a + a 2 , x 3 − 1 , (a 2) 3 . А также степени с нулевым показателем: 5 0 , (a + 1) 0 , 3 + 5 2 − 3 , 2 0 . И степени с целыми отрицательными степенями: (0 , 5) 2 + (0 , 5) — 2 2 .

    Чуть сложнее работать со степенью, имеющей рациональный и иррациональный показатели: 264 1 4 — 3 · 3 · 3 1 2 , 2 3 , 5 · 2 — 2 2 — 1 , 5 , 1 a 1 4 · a 1 2 — 2 · a — 1 6 · b 1 2 , x π · x 1 — π , 2 3 3 + 5 .

    В качестве показателя может выступать переменная 3 x — 54 — 7 · 3 x — 58 или логарифм x 2 · l g x − 5 · x l g x
    .

    С вопросом о том, что такое степенные выражения, мы разобрались. Теперь займемся их преобразованием.

    Основные виды преобразований степенных выражений

    В первую очередь мы рассмотрим основные тождественные преобразования выражений, которые можно выполнять со степенными выражениями.

    Пример 1

    Вычислите значение степенного выражения 2 3 · (4 2 − 12)
    .

    Решение

    Все преобразования мы будем проводить с соблюдением порядка выполнения действий. В данном случае начнем мы с выполнения действий в скобках: заменим степень на цифровое значение и вычислим разность двух чисел. Имеем 2 3 · (4 2 − 12) = 2 3 · (16 − 12) = 2 3 · 4
    .

    Нам остается заменить степень 2 3
    ее значением 8
    и вычислить произведение 8 · 4 = 32
    . Вот наш ответ.

    Ответ:
    2 3 · (4 2 − 12) = 32 .

    Пример 2

    Упростите выражение со степенями 3 · a 4 · b − 7 − 1 + 2 · a 4 · b − 7
    .

    Решение

    Данное нам в условии задачи выражение содержит подобные слагаемые, которые мы можем привести: 3 · a 4 · b − 7 − 1 + 2 · a 4 · b − 7 = 5 · a 4 · b − 7 − 1
    .

    Ответ:
    3 · a 4 · b − 7 − 1 + 2 · a 4 · b − 7 = 5 · a 4 · b − 7 − 1 .

    Пример 3

    Представьте выражение со степенями 9 — b 3 · π — 1 2 в виде произведения.

    Решение

    Представим число 9 как степень 3 2
    и применим формулу сокращенного умножения:

    9 — b 3 · π — 1 2 = 3 2 — b 3 · π — 1 2 = = 3 — b 3 · π — 1 3 + b 3 · π — 1

    Ответ:
    9 — b 3 · π — 1 2 = 3 — b 3 · π — 1 3 + b 3 · π — 1 .

    А теперь перейдем к разбору тождественных преобразований, которые могут применяться именно в отношении степенных выражений.

    Работа с основанием и показателем степени

    Степень в основании или показателе может иметь и числа, и переменные, и некоторые выражения. Например, (2 + 0 , 3 · 7) 5 − 3 , 7
    и . Работать с такими записями сложно. Намного проще заменить выражение в основании степени или выражение в показателе тождественно равным выражением.

    Проводятся преобразования степени и показателя по известным нам правилам отдельно друг от друга. Самое главное, чтобы в результате преобразований получилось выражение, тождественное исходному.

    Цель преобразований – упростить исходное выражение или получить решение задачи. Например, в примере, который мы привели выше, (2 + 0 , 3 · 7) 5 − 3 , 7 можно выполнить действия для перехода к степени 4 , 1 1 , 3
    . Раскрыв скобки, мы можем привести подобные слагаемые в основании степени (a · (a + 1) − a 2) 2 · (x + 1)
    и получить степенное выражение более простого вида a 2 · (x + 1)
    .

    Использование свойств степеней

    Свойства степеней, записанные в виде равенств, являются одним из главных инструментов преобразования выражений со степенями. Приведем здесь основные из них, учитывая, что a
    и b
    – это любые положительные числа, а r
    и s
    — произвольные действительные числа:

    Определение 2

    • a r · a s = a r + s ;
    • a r: a s = a r − s ;
    • (a · b) r = a r · b r ;
    • (a: b) r = a r: b r ;
    • (a r) s = a r · s .

    В тех случаях, когда мы имеем дело с натуральными, целыми, положительными показателями степени, ограничения на числа a и b могут быть гораздо менее строгими. Так, например, если рассмотреть равенство a m · a n = a m + n
    , где m
    и n
    – натуральные числа, то оно будет верно для любых значений a , как положительных, так и отрицательных, а также для a = 0
    .

    Применять свойства степеней без ограничений можно в тех случаях, когда основания степеней положительные или содержат переменные, область допустимых значений которых такова, что на ней основания принимают лишь положительные значения. Фактически, в рамках школьной программы по математике задачей учащегося является выбор подходящего свойства и правильное его применение.

    При подготовке к поступлению в Вузы могут встречаться задачи, в которых неаккуратное применение свойств будет приводить к сужению ОДЗ и другим сложностям с решением. В данном разделе мы разберем всего два таких случая. Больше информации по вопросу можно найти в теме «Преобразование выражений с использованием свойств степеней».

    Пример 4

    Представьте выражение a 2 , 5 · (a 2) − 3: a − 5 , 5
    в виде степени с основанием a
    .

    Решение

    Для начала используем свойство возведения в степень и преобразуем по нему второй множитель (a 2) − 3
    . Затем используем свойства умножения и деления степеней с одинаковым основанием:

    a 2 , 5 · a − 6: a − 5 , 5 = a 2 , 5 − 6: a − 5 , 5 = a − 3 , 5: a − 5 , 5 = a − 3 , 5 − (− 5 , 5) = a 2 .

    Ответ:
    a 2 , 5 · (a 2) − 3: a − 5 , 5 = a 2 .

    Преобразование степенных выражений согласно свойству степеней может производиться как слева направо, так и в обратном направлении.

    Пример 5

    Найти значение степенного выражения 3 1 3 · 7 1 3 · 21 2 3 .

    Решение

    Если мы применим равенство (a · b) r = a r · b r
    , справа налево, то получим произведение вида 3 · 7 1 3 · 21 2 3 и дальше 21 1 3 · 21 2 3 . Сложим показатели при умножении степеней с одинаковыми основаниями: 21 1 3 · 21 2 3 = 21 1 3 + 2 3 = 21 1 = 21 .

    Есть еще один способ провести преобразования:

    3 1 3 · 7 1 3 · 21 2 3 = 3 1 3 · 7 1 3 · (3 · 7) 2 3 = 3 1 3 · 7 1 3 · 3 2 3 · 7 2 3 = = 3 1 3 · 3 2 3 · 7 1 3 · 7 2 3 = 3 1 3 + 2 3 · 7 1 3 + 2 3 = 3 1 · 7 1 = 21

    Ответ:
    3 1 3 · 7 1 3 · 21 2 3 = 3 1 · 7 1 = 21

    Пример 6

    Дано степенное выражение a 1 , 5 − a 0 , 5 − 6
    , введите новую переменную t = a 0 , 5
    .

    Решение

    Представим степень a 1 , 5
    как a 0 , 5 · 3
    . Используем свойство степени в степени (a r) s = a r · s
    справа налево и получим (a 0 , 5) 3: a 1 , 5 − a 0 , 5 − 6 = (a 0 , 5) 3 − a 0 , 5 − 6 . В полученное выражение можно без проблем вводить новую переменную t = a 0 , 5
    : получаем t 3 − t − 6
    .

    Ответ:
    t 3 − t − 6 .

    Преобразование дробей, содержащих степени

    Обычно мы имеем дело с двумя вариантами степенных выражений с дробями: выражение представляет собой дробь со степенью или содержит такую дробь. К таким выражениям применимы все основные преобразования дробей без ограничений. Их можно сокращать, приводить к новому знаменателю, работать отдельно с числителем и знаменателем. Проиллюстрируем это примерами.

    Пример 7

    Упростить степенное выражение 3 · 5 2 3 · 5 1 3 — 5 — 2 3 1 + 2 · x 2 — 3 — 3 · x 2 .

    Решение

    Мы имеем дело с дробью, поэтому проведем преобразования и в числителе, и в знаменателе:

    3 · 5 2 3 · 5 1 3 — 5 — 2 3 1 + 2 · x 2 — 3 — 3 · x 2 = 3 · 5 2 3 · 5 1 3 — 3 · 5 2 3 · 5 — 2 3 — 2 — x 2 = = 3 · 5 2 3 + 1 3 — 3 · 5 2 3 + — 2 3 — 2 — x 2 = 3 · 5 1 — 3 · 5 0 — 2 — x 2

    Поместим минус перед дробью для того, чтобы изменить знак знаменателя: 12 — 2 — x 2 = — 12 2 + x 2

    Ответ:
    3 · 5 2 3 · 5 1 3 — 5 — 2 3 1 + 2 · x 2 — 3 — 3 · x 2 = — 12 2 + x 2

    Дроби, содержащие степени, приводятся к новому знаменателю точно также, как и рациональные дроби. Для этого необходимо найти дополнительный множитель и умножить на него числитель и знаменатель дроби. Подбирать дополнительный множитель необходимо таким образом, чтобы он не обращался в нуль ни при каких значениях переменных из ОДЗ переменных для исходного выражения.

    Пример 8

    Приведите дроби к новому знаменателю: а) a + 1 a 0 , 7 к знаменателю a
    , б) 1 x 2 3 — 2 · x 1 3 · y 1 6 + 4 · y 1 3 к знаменателю x + 8 · y 1 2 .

    Решение

    а) Подберем множитель, который позволит нам произвести приведение к новому знаменателю. a 0 , 7 · a 0 , 3 = a 0 , 7 + 0 , 3 = a ,
    следовательно, в качестве дополнительного множителя мы возьмем a 0 , 3
    . Область допустимых значений переменной а включает множество всех положительных действительных чисел. В этой области степень a 0 , 3
    не обращается в нуль.

    Выполним умножение числителя и знаменателя дроби на a 0 , 3
    :

    a + 1 a 0 , 7 = a + 1 · a 0 , 3 a 0 , 7 · a 0 , 3 = a + 1 · a 0 , 3 a

    б) Обратим внимание на знаменатель:

    x 2 3 — 2 · x 1 3 · y 1 6 + 4 · y 1 3 = = x 1 3 2 — x 1 3 · 2 · y 1 6 + 2 · y 1 6 2

    Умножим это выражение на x 1 3 + 2 · y 1 6 , получим сумму кубов x 1 3 и 2 · y 1 6 , т.е. x + 8 · y 1 2 . Это наш новый знаменатель, к которому нам надо привести исходную дробь.

    Так мы нашли дополнительный множитель x 1 3 + 2 · y 1 6 . На области допустимых значений переменных x
    и y
    выражение x 1 3 + 2 · y 1 6 не обращается в нуль, поэтому, мы можем умножить на него числитель и знаменатель дроби:
    1 x 2 3 — 2 · x 1 3 · y 1 6 + 4 · y 1 3 = = x 1 3 + 2 · y 1 6 x 1 3 + 2 · y 1 6 x 2 3 — 2 · x 1 3 · y 1 6 + 4 · y 1 3 = = x 1 3 + 2 · y 1 6 x 1 3 3 + 2 · y 1 6 3 = x 1 3 + 2 · y 1 6 x + 8 · y 1 2

    Ответ:
    а) a + 1 a 0 , 7 = a + 1 · a 0 , 3 a , б) 1 x 2 3 — 2 · x 1 3 · y 1 6 + 4 · y 1 3 = x 1 3 + 2 · y 1 6 x + 8 · y 1 2 .

    Пример 9

    Сократите дробь: а) 30 · x 3 · (x 0 , 5 + 1) · x + 2 · x 1 1 3 — 5 3 45 · x 0 , 5 + 1 2 · x + 2 · x 1 1 3 — 5 3 , б) a 1 4 — b 1 4 a 1 2 — b 1 2 .

    Решение

    а) Используем наибольший общий знаменатель (НОД), на который можно сократить числитель и знаменатель. Для чисел 30 и 45 это 15 . Также мы можем произвести сокращение на x 0 , 5 + 1
    и на x + 2 · x 1 1 3 — 5 3 .

    Получаем:

    30 · x 3 · (x 0 , 5 + 1) · x + 2 · x 1 1 3 — 5 3 45 · x 0 , 5 + 1 2 · x + 2 · x 1 1 3 — 5 3 = 2 · x 3 3 · (x 0 , 5 + 1)

    б) Здесь наличие одинаковых множителей неочевидно. Придется выполнить некоторые преобразования для того, чтобы получить одинаковые множители в числителе и знаменателе. Для этого разложим знаменатель, используя формулу разности квадратов:

    a 1 4 — b 1 4 a 1 2 — b 1 2 = a 1 4 — b 1 4 a 1 4 2 — b 1 2 2 = = a 1 4 — b 1 4 a 1 4 + b 1 4 · a 1 4 — b 1 4 = 1 a 1 4 + b 1 4

    Ответ:
    а) 30 · x 3 · (x 0 , 5 + 1) · x + 2 · x 1 1 3 — 5 3 45 · x 0 , 5 + 1 2 · x + 2 · x 1 1 3 — 5 3 = 2 · x 3 3 · (x 0 , 5 + 1) , б) a 1 4 — b 1 4 a 1 2 — b 1 2 = 1 a 1 4 + b 1 4 .

    К числу основных действий с дробями относится приведение к новому знаменателю и сокращение дробей. Оба действия выполняют с соблюдением ряда правил. При сложении и вычитании дробей сначала дроби приводятся к общему знаменателю, после чего проводятся действия (сложение или вычитание) с числителями. Знаменатель остается прежним. Результатом наших действий является новая дробь, числитель которой является произведением числителей, а знаменатель есть произведение знаменателей.

    Пример 10

    Выполните действия x 1 2 + 1 x 1 2 — 1 — x 1 2 — 1 x 1 2 + 1 · 1 x 1 2 .

    Решение

    Начнем с вычитания дробей, которые располагаются в скобках. Приведем их к общему знаменателю:

    x 1 2 — 1 · x 1 2 + 1

    Вычтем числители:

    x 1 2 + 1 x 1 2 — 1 — x 1 2 — 1 x 1 2 + 1 · 1 x 1 2 = = x 1 2 + 1 · x 1 2 + 1 x 1 2 — 1 · x 1 2 + 1 — x 1 2 — 1 · x 1 2 — 1 x 1 2 + 1 · x 1 2 — 1 · 1 x 1 2 = = x 1 2 + 1 2 — x 1 2 — 1 2 x 1 2 — 1 · x 1 2 + 1 · 1 x 1 2 = = x 1 2 2 + 2 · x 1 2 + 1 — x 1 2 2 — 2 · x 1 2 + 1 x 1 2 — 1 · x 1 2 + 1 · 1 x 1 2 = = 4 · x 1 2 x 1 2 — 1 · x 1 2 + 1 · 1 x 1 2

    Теперь умножаем дроби:

    4 · x 1 2 x 1 2 — 1 · x 1 2 + 1 · 1 x 1 2 = = 4 · x 1 2 x 1 2 — 1 · x 1 2 + 1 · x 1 2

    Произведем сокращение на степень x 1 2
    , получим 4 x 1 2 — 1 · x 1 2 + 1 .

    Дополнительно можно упростить степенное выражение в знаменателе, используя формулу разности квадратов: квадратов: 4 x 1 2 — 1 · x 1 2 + 1 = 4 x 1 2 2 — 1 2 = 4 x — 1 .

    Ответ:
    x 1 2 + 1 x 1 2 — 1 — x 1 2 — 1 x 1 2 + 1 · 1 x 1 2 = 4 x — 1

    Пример 11

    Упростите степенное выражение x 3 4 · x 2 , 7 + 1 2 x — 5 8 · x 2 , 7 + 1 3 .
    Решение

    Мы можем произвести сокращение дроби на (x 2 , 7 + 1) 2
    . Получаем дробь x 3 4 x — 5 8 · x 2 , 7 + 1 .

    Продолжим преобразования степеней икса x 3 4 x — 5 8 · 1 x 2 , 7 + 1 . Теперь можно использовать свойство деления степеней с одинаковыми основаниями: x 3 4 x — 5 8 · 1 x 2 , 7 + 1 = x 3 4 — — 5 8 · 1 x 2 , 7 + 1 = x 1 1 8 · 1 x 2 , 7 + 1 .

    Переходим от последнего произведения к дроби x 1 3 8 x 2 , 7 + 1 .

    Ответ:
    x 3 4 · x 2 , 7 + 1 2 x — 5 8 · x 2 , 7 + 1 3 = x 1 3 8 x 2 , 7 + 1 .

    Множители с отрицательными показателями степени в большинстве случаев удобнее переносить из числителя в знаменатель и обратно, изменяя знак показателя. Это действие позволяет упростить дальнейшее решение. Приведем пример: степенное выражение (x + 1) — 0 , 2 3 · x — 1 можно заменить на x 3 · (x + 1) 0 , 2 .

    Преобразование выражений с корнями и степенями

    В задачах встречаются степенные выражения, которые содержат не только степени с дробными показателями, но и корни. Такие выражения желательно привести только к корням или только к степеням. Переход к степеням предпочтительнее, так как с ними проще работать. Такой переход является особенно предпочтительным, когда ОДЗ переменных для исходного выражения позволяет заменить корни степенями без необходимости обращаться к модулю или разбивать ОДЗ на несколько промежутков.

    Пример 12

    Представьте выражение x 1 9 · x · x 3 6 в виде степени.

    Решение

    Область допустимых значений переменной x
    определяется двумя неравенствами x ≥ 0
    и x · x 3 ≥ 0 , которые задают множество [ 0 , + ∞)
    .

    На этом множестве мы имеем право перейти от корней к степеням:

    x 1 9 · x · x 3 6 = x 1 9 · x · x 1 3 1 6

    Используя свойства степеней, упростим полученное степенное выражение.

    x 1 9 · x · x 1 3 1 6 = x 1 9 · x 1 6 · x 1 3 1 6 = x 1 9 · x 1 6 · x 1 · 1 3 · 6 = = x 1 9 · x 1 6 · x 1 18 = x 1 9 + 1 6 + 1 18 = x 1 3

    Ответ:
    x 1 9 · x · x 3 6 = x 1 3 .

    Преобразование степеней с переменными в показателе

    Данные преобразования достаточно просто произвести, если грамотно использовать свойства степени. Например, 5 2 · x + 1 − 3 · 5 x · 7 x − 14 · 7 2 · x − 1 = 0
    .

    Мы можем заменить произведением степени, в показателях которых находится сумма некоторой переменной и числа. В левой части это можно проделать с первым и последним слагаемыми левой части выражения:

    5 2 · x · 5 1 − 3 · 5 x · 7 x − 14 · 7 2 · x · 7 − 1 = 0 , 5 · 5 2 · x − 3 · 5 x · 7 x − 2 · 7 2 · x = 0 .

    Теперь поделим обе части равенства на 7 2 · x
    . Это выражение на ОДЗ переменной x принимает только положительные значения:

    5 · 5 — 3 · 5 x · 7 x — 2 · 7 2 · x 7 2 · x = 0 7 2 · x , 5 · 5 2 · x 7 2 · x — 3 · 5 x · 7 x 7 2 · x — 2 · 7 2 · x 7 2 · x = 0 , 5 · 5 2 · x 7 2 · x — 3 · 5 x · 7 x 7 x · 7 x — 2 · 7 2 · x 7 2 · x = 0

    Сократим дроби со степенями, получим: 5 · 5 2 · x 7 2 · x — 3 · 5 x 7 x — 2 = 0 .

    Наконец, отношение степеней с одинаковыми показателями заменяется степенями отношений, что приводит к уравнению 5 · 5 7 2 · x — 3 · 5 7 x — 2 = 0 , которое равносильно 5 · 5 7 x 2 — 3 · 5 7 x — 2 = 0 .

    Введем новую переменную t = 5 7 x , что сводит решение исходного показательного уравнения к решению квадратного уравнения 5 · t 2 − 3 · t − 2 = 0 .

    Преобразование выражений со степенями и логарифмами

    Выражения, содержащие с записи степени и логарифмы, также встречаются в задачах. Примером таких выражений могут служить: 1 4 1 — 5 · log 2 3 или log 3 27 9 + 5 (1 — log 3 5) · log 5 3 . Преобразование подобных выражений проводится с использованием разобранных выше подходов и свойств логарифмов, которые мы подробно разобрали в теме «Преобразование логарифмических выражений».

    Если вы заметили ошибку в тексте, пожалуйста, выделите её и нажмите Ctrl+Enter

    Вычисление значений степенных выражений(ЕГЭ №9)

    Степенные выражения — это выражения, содержащие степени.2=121

    В задании №9 так же могут пригодятся такие формулы:

    (a + b)2 = a2 + 2ab + b2 (квадрат суммы)

    (a — b)2 = a2 — 2ab + b2 (квадрат разности)

    a2 — b2 = (a + b)(a — b) (разность квадратов)

    (a + b)3 = a3 + 3a2b + 3ab2 + b3 (куб суммы)

    (a — b)3 = a3 — 3a2b + 3ab2 — b3 (куб разности)

    a3 + b3 = (a + b)(a2 — ab + b2) (сумма кубов)

    a3 — b3 = (a — b)(a2 + ab + b2) (разность кубов)

    Преобразование выражений с корнями и степенями

    Так же , часто в выражениях, в которыми требуется провести некоторые преобразования, вместе со степенями с дробными показателями присутствуют и корни. Чтобы преобразовать подобное выражение к нужному виду, в большинстве случаев достаточно перейти только к корням или только к степеням. Но поскольку работать со степенями удобнее, обычно переходят от корней к степеням

    Пример:

    На следующем указанном сайте, вы можете найти много примеров по заданию №9. Так же на этом сайте,вы сможете найти объяснение к номеру,если вы не знаете как решать его:

    https://math-ege.sdamgia.ru/test?theme=57

    Для проверки своих знаний,вы можете перейти по ссылке на тест:
    https://test-po-zadaniu-9-ege.testograf.ru

    Упрощение экспонентных выражений | Purplemath

    Purplemath

    Чтобы упростить работу с экспонентами, не думайте, что вам нужно работать только с правилами для экспонентов или прямо из них. Часто проще работать непосредственно с определением и значением показателя степени. Например:

    Правила говорят мне добавить экспоненты.Но когда я начал заниматься алгеброй, у меня были проблемы с соблюдением правил, поэтому я просто подумал о том, что означают показатели. « a 6 » означает «шесть копий a , умноженных вместе», а « a 5 » означает «пять копий a , умноженных вместе». Итак, если я умножу эти два выражения вместе, я получу одиннадцать копий на , умноженных вместе. То есть:

    MathHelp.com

    a 6 × a 5 = ( a 6 ) ( a 5 )

    = ( аааааа ) ( ааааа )

    = аааааааааа

    = а 11

    Таким образом:


    • Упростите следующее выражение:

    Правила экспоненты говорят мне вычесть экспоненты.Но предположим, что я снова забыл правила. «6 8 » означает, что у меня восемь копий из шести сверху; «6 5 » означает, что у меня есть пять копий из 6 внизу.

    Сколько у меня лишних шестерок и где они? У меня есть три лишних шестерки, и они на высоте. Тогда:

    Если в инструкциях вам также не сказано «оценивать», вы, вероятно, должны оставить такие проблемы с числовым показателем, как эта, в форме экспоненты.Если вы не уверены, можете добавить «= 216» на всякий случай.


    • Упростите следующее выражение:

    Сколько у меня дополнительных копий t и где они? У меня есть две лишние копии, сверху:

    Как только вы освоитесь с вопросом «сколько у меня статистов и где они?» рассуждая, вы обнаружите, что вам не нужно все записывать и устранять повторяющиеся факторы.Ответы станут для вас очевидными.


    • Упростите следующее выражение:

    Этот вопрос немного отличается, потому что больший показатель степени находится у члена в знаменателе. Но основная аргументация остается прежней.

    Сколько у меня дополнительных копий из 5 и где они? У меня есть шесть дополнительных копий, и они внизу:


    Примечание. Если вы примените правило вычитания, вы получите 5 3–9 = 5 –6 , что математически верно, но почти наверняка не является тем ответом, который они ищут.

    Независимо от того, учили ли вы отрицательные показатели степени, когда они говорят «упрощать», они имеют в виду «упростить выражение, чтобы оно не имело отрицательных или нулевых степеней». Некоторые студенты будут пытаться обойти эту проблему со знаком минус, произвольно переключая знак, чтобы волшебным образом получить «5 6 » сверху (а не ниже «1»), но это неверно.

    Давайте перейдем к более сложным выражениям.


    • Упростите следующее выражение:

    Я не должен забывать, что «5» и «3» — это просто числа.Поскольку 3 не делится на 5, я не могу отменить числа.

    И я не должен пытаться вычитать числа, потому что 5 и 3 в дроби «

    5 / 3 » совсем не то же самое, что 5 и 3 в рациональном выражении « x 5 / x 3 «. Числовая часть 5 / 3 остается без изменений.

    Для переменных у меня есть две дополнительные копии x сверху, поэтому ответ:

    Любой из ответов, выделенных фиолетовым цветом, должен быть приемлемым: единственная разница заключается в форматировании; они означают одно и то же.


    Это достаточно просто: все до нулевой степени равно 1.

    (–46 x 2 y 3 z ) 0 = 1


    Часть в скобках по-прежнему упрощается до 1, но на этот раз «минус» стоит перед скобками; то есть он выходит из-под власти, поэтому экспонента его не трогает.Итак, в данном случае ответ:

    .

    — (46 x 2 y 3 z ) 0 = –1


    • Упростите следующее выражение:

    Я могу исключить общий множитель 5 в числовой части дроби:

    Теперь мне нужно посмотреть на каждую из переменных.Сколько у меня лишних штук и где они? У меня есть два лишних и сверху. У меня есть один лишний b внизу. И у меня одинаковые числа c сверху и снизу, поэтому они полностью отменяются. Это дает мне:


    URL: https://www.purplemath.com/modules/simpexpo.htm

    Термины и выражения с показателями

    Результаты обучения

    • Определить компоненты термина, содержащего целые показатели
    • Вычислить выражения, содержащие целые показатели

    Lingua franca — это общий язык, используемый для обеспечения возможности общения между людьми, говорящими на разных языках.Математика, как общая идея, иногда рассматривается как пример общего языка, потому что формулы и уравнения не зависят от свободного владения конкретным языком.

    Но даже в математике нужен общий язык, чтобы ясно и эффективно передавать математические идеи. Экспоненциальное представление (помните, что это также может называться научным представлением) было разработано для более эффективной записи повторного умножения. Например, у живых организмов рост происходит за счет деления клеток.{12} [/ латекс]. Такое выражение — гораздо более эффективный и ясный способ выразить способы деления клеток.

    В этом разделе мы узнаем, как упростить и выполнять математические операции, такие как умножение и деление, над членами, имеющими показатель степени. Мы также узнаем, как использовать научную нотацию для представления очень больших или очень маленьких чисел и выполнять с ними математические операции.

    Анатомия экспоненциальных членов

    Мы используем экспоненциальную запись для записи повторного умножения.{2} = — 5 \ cdot {-5} = 25 [/ латекс]

    В следующем видео вам предоставлено больше примеров применения экспонент к различным основаниям.

    Вычислить выражения

    Оценка выражений, содержащих показатели степени, аналогична оценке линейных выражений из предыдущего курса. Вы подставляете значение переменной в выражение и упрощаете.

    Вы можете использовать порядок операций для оценки выражений, содержащих экспоненты.{2} \\ = \ left (-3 \ right) \ cdot \ left (-3 \ right) \\ = {9} \ end {array} [/ latex]

    Ключ к запоминанию — это соблюдение порядка операций. В первом выражении нет скобок, поэтому сначала нужно применить показатель степени к целому числу 3, а затем применить отрицательный знак. Второе выражение включает круглые скобки, поэтому, надеюсь, вы помните, что отрицательный знак также возводится в квадрат.

    В следующих разделах вы узнаете, как упрощать выражения, содержащие экспоненты.Вернитесь на эту страницу, если вы забыли, как применить порядок операций к члену с показателями, или забыли, что является основанием, а которое — показателем!

    В следующем видео вам представлены примеры вычисления экспоненциальных выражений для данного числа.

    Упрощение экспоненциальных выражений | Колледж алгебры

    Напомним, что упрощение выражения означает его переписывание, комбинируя термины или показатели; другими словами, проще написать выражение с меньшим количеством терминов.{2}} [/ латекс]

    Решение

    правил экспонент — ChiliMath

    Правила экспонент, также известные как «правила экспоненты», — это некоторые из правил по предмету алгебры, с которыми нам необходимо ознакомиться. Освоение этих основных правил экспоненты вместе с основными правилами логарифмов (также известными как «правила журнала») сделает ваше изучение алгебры очень продуктивным и увлекательным.

    Начнем с изучения частей экспоненциального числа.

    Показательное число или выражение состоит из двух частей.Первый компонент — это основание , которое «несет» показатель степени , который является вторым компонентом в его правом верхнем углу.

    Взгляните на иллюстрацию ниже.

    Части экспоненциального числа или выражения

    Например, как бы вы записали 2 \ cdot 2 \ cdot 2 \ cdot 2 \ cdot 2 в экспоненциальной записи?

    Число 2 — это число, которое многократно умножается, поэтому оно автоматически становится основой экспоненциального выражения.Обратите внимание, что это написано пять раз. Это значение указывает количество вхождений основания, таким образом, это должен быть показатель степени.

    Читается как «от 2 до 5 степени».


    Основание экспоненциального выражения также может быть буквой или переменной. Допустим, у нас

    Поскольку переменная x умножается в десять раз, мы можем записать это в компактной форме.

    Читается как «x в 10-й степени».


    Краткое изложение семи (7) правил экспонент


    Теперь давайте рассмотрим семь (7) основных правил экспоненты.0} = 1.

    • Упростите показанное ниже экспоненциальное выражение.

    Каждое выражение со скобкой, возведенной в степень нуля, 0, как в числителе, так и в знаменателе, будет просто заменено на 1. Обязательно уменьшите дробь до наименьшего члена.


    ПРАВИЛО 2: Свойство отрицательной экспоненты

    Любое ненулевое число, возведенное в отрицательную степень, не имеет стандартной формы. Нам нужно будет немного переставить. Переместите основание с отрицательной степенью в противоположную сторону дроби, затем сделайте показатель положительной.{- \, 4}}.

    Основание 2 имеет отрицательный показатель степени -4. Это можно исправить, переместив его в знаменатель и изменив знак экспоненты на положительный, используя отрицательное правило экспоненты.

    • Упростите экспоненциальное выражение.

    На этот раз основание с отрицательной экспонентой находится в знаменателе. Подведите его к числителю, сделав показатель степени положительным. 2}} \ right).2}} \ справа).

    После того, как мы умножим экспоненциальные выражения с тем же основанием, добавив их показатели, мы получим одну переменную с отрицательной экспонентой, а другую — с нулевой экспонентой.

    Не сомневайтесь, примените два выученных ранее правила, а именно Правило 1 и Правило 2, чтобы еще больше упростить это выражение.


    ПРАВИЛО 4: Факторное свойство экспоненты

    При делении экспоненциальных выражений с одинаковым основанием, где основание является ненулевым действительным числом, скопируйте общее основание, а затем вычтите верхний показатель степени на нижний показатель степени.Здесь мы должны предположить, что b \ ne 0 и оба m и n принадлежат множеству целых чисел.

    Примеры :

    • Упростите частное экспоненциальных выражений.

    Дробная черта означает, что мы собираемся делить. Имеет смысл применить правило деления экспоненты, то есть скопировать общее основание в числителе и знаменателе и вычесть верхний показатель степени на более низкий показатель.

    • Упростите экспоненциальные выражения.

    Сравнивая выражения в числителе и знаменателе, я вижу, что есть два общих основания, x и y. Примените правило деления к каждой переменной. После этого переменная x будет содержать отрицательную экспоненту, поэтому используйте отрицательное правило экспоненты, чтобы решить проблему.

    • Упростите экспоненциальные выражения.

    Один из способов упростить это — пока игнорировать отрицательные показатели. Сначала примените правило деления и посмотрите, появятся ли снова отрицательные показатели.3}.

    Это выражение имеет внутренний и внешний экспоненты. Правило «Степень к мощности» позволяет нам копировать основание и умножать показатели.


    ПРАВИЛО 6: Мощность свойства продукта экспоненты

    Когда произведение двух или более множителей возводится в степень, скопируйте каждый множитель, а затем умножьте его показатель на внешний показатель. Мы должны сделать это для каждого фактора в скобках, которые в данном случае являются a и b. Предположения: a \ ne 0 или b \ ne 0, а n — целое число.2}.

    Эта проблема очень похожа на предыдущую. Единственное отличие состоит в том, что есть три (3) множителя с показателями степени. Нам просто нужно распределить внешний показатель степени на каждый из внутренних показателей.


    ПРАВИЛО 7: Степень частной собственности экспоненты

    Когда частное возводится в степень, скопируйте множитель в числителе, затем умножьте его показатель на внешний показатель. Мы должны сделать то же самое с множителем в знаменателе, куда мы его копируем, а затем умножить его показатель на внешний показатель.Здесь также нужно предположить, что a \ ne 0 или b \ ne 0, а m — целое число.

    Пример:

    • Упростите экспоненциальное выражение.

    На самом деле, мы будем использовать здесь одновременно два свойства экспонент, чтобы полностью это упростить. В дополнение к Правилу 7 (Правило силы частного) нам нужно будет применить Правило 6 (Правило силы произведения). Проще говоря, просто обрабатывайте числитель и знаменатель отдельно при распределении путем умножения внутренних и внешних показателей для каждого фактора.


    Практика с рабочими листами

    Умножение экспонентов — объяснение и примеры

    Показатели — это степени или индексы. Показатель или степень обозначают, сколько раз число многократно умножается само на себя. Например, когда мы встречаем число, записанное как 5 3 , это просто означает, что 5 умножается само на себя три раза. Другими словами, 5 3 = 5 x 5 x 5 = 125.

    Экспоненциальное выражение состоит из двух частей: основания, обозначенного как b, и показателя степени, обозначенного как n.Общая форма экспоненциального выражения: b n .

    Как умножить экспоненты?

    Умножение экспонент является важной частью математики более высокого уровня, однако многим ученикам сложно понять, как это сделать. Хотя выражения, включающие отрицательные и множественные показатели, могут сбивать с толку.

    В этой статье мы собираемся изучить умножение экспонент и, следовательно, это поможет вам чувствовать себя более комфортно при решении задач с показателями.

    Умножение показателей включает в себя следующие подтемы:

    • Умножение показателей с одинаковым основанием
    • Умножение показателей с разными основаниями
    • Умножение отрицательных показателей
    • Умножение дробных показателей на показатели
    • Умножение дробных показателей
    • Умножение переменных с показателями степени
    • Умножение квадратных корней на показатели степени

    Умножение показателей степени с одинаковым основанием

    При умножении показателей степени с одинаковыми основаниями показатели степени складываются.Правило умножения для сложения показателей, когда основания одинаковы, можно обобщить следующим образом: a n xa m = a n + m

    Пример 1

    • m⁵ × m³ = (m × m × м × м × м) × (м × м × м)

    = м 5 + 3

    = m⁸

    • 3⁴ × 3² = (3 × 3 × 3 × 3) × (3 × 3) = 3 4+ 3 = 3⁶
    • (-3) ³ × (-3) ⁴ = [(-3) × (-3) × (-3)] × [(-3) × (-3) × (-3) × (-3)]

    = (-3) 3 +4

    = (-3) 7

    • 5³ × 5⁶
      = (5 × 5 × 5) × (5 × 5 × 5 × 5 × 5 × 5)
      = 5 3 + 6

    = 5⁹

    = [(-7) × (-7) × (-7) × (-7) × (-7) × (-7) × (-7) × (-7) × (-7) × (-7)] × [(-7) × (-7) × (-7) × (-7 ) × (-7) × (-7) × (-7) × (-7) × (-7) × (-7) × (-7) × (-7)].

    = (-7) ²²

    Умножение показателей степени с разными основаниями

    При умножении двух переменных с разными основаниями, но одинаковыми показателями степени, мы просто умножаем основания и ставим один и тот же показатель степени. Это правило можно резюмировать следующим образом:

    a n ⋅ b n = (a ⋅ b) n

    Пример 2

    • (x 3 ) * (y 3 ) = xxx * yyy = (xy) 3
    • 3 2 x 4 2 = (3 x 4) 2 = 12 2 = 144

    Если степень и основание различаются , затем каждое число вычисляется отдельно, а затем результаты умножаются.В этом случае формула имеет вид : a n b m

    Пример 3

    • Как умножить отрицательные показатели?

    Для чисел с одинаковым основанием и отрицательной степенью мы просто складываем экспоненты. В общем: a -n x -m = a (n + m) = 1 / a n + m .

    Пример 4

    • 2 -3 x 2 -4 = 2 — (3 + 4) = 2 -7 = 1/2 7 = 1 / (2 х 2 х 2 х 2 х 2 х 2 х 2) = 1/128 = 0.0078125

    Точно так же, если основания разные, а показатели одинаковы, мы сначала умножаем основания и используем показатель степени.

    a -n xb -n = (a x b) -n

    Пример 5

    • 3 -2 x 4 -2 = (3 x 4) -2 = 12 -2 = 1/12 2 = 1 / (12⋅12) = 1/144 = 0,0069444
    • Как умножить дроби на показатели?

    При умножении дробей с одинаковым основанием мы складываем экспоненты.Например:

    (a / b) n x (a / b) m = (a / b) n + m

    Пример 6

    • (4/3) 3 x (3/5) 3 = ((4/3) x (3/5)) 3 = (4/5) 3 = 0,8 3 = 0,8 x 0,8 x 8 = 0,512
    • (4/3) 3 x (4/3) 2 = (4/3) 3 + 2 = (4/3) 5 = 4 5 /3 5 = 4,214
    • (-1/4) -3 × (-1/4) -2
      (-1/4) -3 × (-1/4) -2
      = (4 / -1) 3 × (4 / -1) 2
      = (-4) 3 × (-4) 2
      = (-4) (3 + 2)
      = ( -4) 5
      = -4 5
      = -1024.
    • (-2 / 7 ) -4 × (-5 / 7 ) 2
      (-2 / 7 ) -4 × (-5 / 7 ) 2
      = (7 / -2) 4 × (-5/7) 2
      = (-7/2) 4 × (-5/7) 2
      = (-7) 4 /2 4 × (-5) 2 /7 2
      = {7 4 × (-5) 2 } / {2 4 × 7 2 }
      = {7 2 × (-5) 2 } / 2 4
      = [49 × (-5) × (-5)] / 16
      = 1225/16
    • Как умножить дробные показатели?

    Общая формула для этого случая: a н / м ⋅ b н / м = (a ⋅ b) н / м

    Пример 7

    • 2 3 / 2 x 3 3/2 = (2⋅3) 3/2 = 6 3/2 = √ (6 3 ) = √216 = 14.7

    Точно так же дробные показатели с одинаковыми основаниями, но разными показателями имеют общую формулу: a (n / m) xa (k / j) = a [(n / m) + (k / j)]

    Пример 8

    • 2 (3/2) x 2 (4/3) = 2 [(3/2) + (4/3)] = 7.127
    • Как умножить квадратный корень на показатель степени?

    Для показателей с одинаковым основанием можно сложить показатели:

    (√a) n x (√a) m = a (n + m) / 2

    Пример 9

    • (√5) 2 x ( 5) 4 = 5 (2 + 4) / 2 = 5 6/2 = 5 3 = 125
    • Умножение переменных на показатели

    Для показателей с одинаковым основанием мы можем сложить показатели:

    x n * x m = x n + m

    Пример 10

    • x 2 * x 3 = (x * x) ⋅ (x * x * x) = x 2 + 3 = x 5

    Практические вопросы

    1. Длина прямоугольник равен квадрату его ширины.Если площадь этого прямоугольника равна 64 квадратным единицам, найдите длину прямоугольника.
    2. Свету нужно 5 × 10 2 секунд, чтобы пройти от Солнца до Земли. Если скорость света 3 × 10 8 м / с, каково расстояние между Солнцем и Землей?

    Ответы

    1. 4 единицы
    2. 1,5 × 10 11 м

    Предыдущий урок | Главная страница | Следующий урок. = 1, это означает, что все, что возведено в нулевую степень, равно 1. Правило мощности (От полномочий к полномочиям): ( m ) n = mn , это говорит о том, что для возведения мощности в степень вам необходимо умножить степень. Есть несколько других правил, которые соответствуют правилу мощности, например, правило произведения к степеням и правило отношения к степеням. Правило отрицательной экспоненты :, это говорит о том, что отрицательные показатели в числителе перемещаются в знаменатель и становятся положительными показателями.Отрицательные показатели в знаменателе перемещаются в числитель и становятся положительными показателями. Перемещайте только отрицательные показатели. Правило произведения : a m ∙ a n = a m + n , это говорит о том, что для умножения двух степеней с одинаковым основанием вы сохраняете основание и складываете степени. Правило частных :, это говорит о том, что для деления двух степеней с одинаковым основанием вы сохраняете основание и вычитаете степени.Это похоже на сокращение фракций; когда вы вычитаете степени, поместите ответ в числитель или знаменатель в зависимости от того, где находится высшая степень. Если в знаменателе указана более высокая степень, поместите разницу в знаменатель и наоборот, это поможет избежать отрицательных показателей степени.

    Теперь, когда мы рассмотрели правила для экспонент, вот шаги, необходимые для упрощения экспоненциальных выражений (обратите внимание, что мы применяем правила в том же порядке, в котором они были написаны выше):

    Шаг 1 : Примените правило нулевой экспоненты.Измените все, что поднят до нуля, на 1.
    Шаг 2 : Примените правило мощности. Умножьте (или распределите) показатель степени за пределами круглых скобок с каждым показателем внутри скобок, помните, что если показатель степени не показан, то показатель степени равен 1.
    Шаг 3 : Примените правило отрицательной экспоненты. Отрицательные показатели в числителе перемещаются в знаменатель и становятся положительными показателями.Отрицательные показатели в знаменателе перемещаются в числитель и становятся положительными показателями. Перемещайте только отрицательные показатели. Обратите внимание, что порядок, в котором перемещаются объекты, не имеет значения.
    Шаг 4 : Примените правило продукта. Чтобы умножить два показателя степени с одинаковым основанием, вы сохраняете основание и складываете степени.
    Шаг 5 : Примените правило частного. Это похоже на сокращение фракций; когда вы вычитаете степени, поместите ответ в числитель или знаменатель в зависимости от того, где находится высшая степень.Если в знаменателе указана большая степень, поместите разницу в знаменатель и наоборот, это поможет избежать отрицательных показателей степени и повторения шага 3.
    Шаг 6 : Возвести каждый коэффициент (или число) в соответствующую степень, а затем упростить или уменьшить оставшиеся дроби.

    Пример 1 — Упростить:

    Пример 2 –Упростить:

    Щелкните здесь для практических задач

    Пример 3 –Simplify:

    Щелкните здесь для практических задач

    Пример 4 –Simplify:

    Щелкните здесь для практических задач

    Пример 5 –Simplify:

    Щелкните здесь для практических задач

    экспонент правил | Помощь с математикой

    Приведенные ниже объяснения и примеры правил экспоненты являются продолжением страницы Power (Exponents and Bases), с которой вы, возможно, захотите начать.

    Примеры упрощения экспоненциальных выражений показывают, когда и как используются приведенные выше правила экспоненты.

    Экспоненты: Правило продукта

    Правило произведения для экспонент гласит, что когда мы умножаем экспоненциальные выражения с одинаковым основанием, мы можем складывать показатели и оставлять основание неизменным.

    Правило продукта Если m и n — натуральные числа, а a — действительное число, то
    a m x a n = a m + n
    Пример Записываем 4 2 4 3 , используя единственное основание и показатель степени.
    Правило произведения утверждает, что a m xa n = a m + n
    Применяя правило к выражению и упрощая, получаем
    4 2 x 4 3 = 4 2 + 3 = 4 5
    4 2 = 4 x 4 и 4 3 = 4 x 4 x 4
    Подумайте об этом так, чтобы 4 2 x 4 3 = (4 x 4) x (4 x 4 x 4) = 4 5

    Ноль как показатель степени

    Согласно правилу нулевой экспоненты, любое ненулевое число, возведенное в степень 0, равно 1.

    Правило нуля Если a — любое ненулевое число, то
    a 0 = 1
    Пример Вычислите выражение (-2) 0
    Правило нулевой экспоненты утверждает, что 0 = 1
    Применяя правило к выражению, мы получаем
    (-2) 0 = 1

    Правило частного для показателей

    Мы используем правило частного при делении показателей с одинаковым основанием.Правило частного для экспонент гласит, что если мы разделим показатели с одинаковым основанием, то мы можем вычесть показатели и оставить основание неизменным.

    Правило отрицательной экспоненты

    Мы используем правило отрицательной экспоненты, чтобы заменить выражение с отрицательной экспонентой на эквивалентное выражение с положительной экспонентой. Правило гласит, что любое ненулевое число, возведенное в отрицательную степень, равно его обратному значению, возведенному в противоположную положительную степень. Другими словами, выражение, возведенное в отрицательную степень, равно 1, деленному на выражение с измененным знаком экспоненты.

    Правило отрицательной экспоненты Для любого ненулевого действительного числа b и любого целого числа m ,
    b -m = 1b m
    Пример Запишите выражение 6a -2 без отрицательной степени.
    Примените правило, чтобы переписать термин -2 , поскольку a возводится в отрицательную степень. Запишите обратную величину a и возведите ее в степень, противоположную –2, которая равна 2.

    Итак, 6a -2 , записанное без отрицательной степени, равно 6 / a 2

    Упрощение экспоненциальных выражений — все вместе

    Мы можем упростить экспоненциальные выражения, используя подходящую комбинацию правил и свойств, описанных выше.

    .

    Добавить комментарий

    Ваш адрес email не будет опубликован. Обязательные поля помечены *